100% found this document useful (1 vote)
411 views267 pages

E Maths Sec 42011

Uploaded by

Timothy Handoko
Copyright
© © All Rights Reserved
We take content rights seriously. If you suspect this is your content, claim it here.
Available Formats
Download as PDF, TXT or read online on Scribd
100% found this document useful (1 vote)
411 views267 pages

E Maths Sec 42011

Uploaded by

Timothy Handoko
Copyright
© © All Rights Reserved
We take content rights seriously. If you suspect this is your content, claim it here.
Available Formats
Download as PDF, TXT or read online on Scribd
You are on page 1/ 267

2011

Secondary School
Examination
Papers

Secondary Four Express


Mathematics
Paper 1 & 2

1 Anglican High School SA2


2 Cedar Girl’s Secondary School SA2
3 Crescent Girl’s School SA2
4 Dunman High School SA2
5 Dunman Secondary School SA2
6 Methodist Girl’s School SA2
7 Tanjong Katong Girls School SA2
8 Victoria School SA2
9 Xinmin Secondary School SA2
10 Anderson Secondary School SA2
ANGILICAN HIGH SCHOOL
PRELIMINARY EXAMINATION 2011
MATHEMATICS
PAPER 1
2

Answer (a) n= _ _ _ _ _ _ _ _ _ _[l]

(b) Simplify (p 2 )- 1 x (pq) 3 •

Answer (b) _ _ _ _ _ _ _ _ _ _[!]

5 A man bought a watch for $560. He made a profit of 15% of the cost price after selling the
watch at a discount of20% of the selling price. Find the actual selling price of the watch.

Answer
~------------
[2]

6 The model of a school is made to a scale of 1 : 50.


(a) If the height of the school in the model is 36 cm, find the actual height of the school in
metres.

[Turn Over
Anglican High School Preliminary Examinations 03 2011 Math (4016/01)

4
3

Answer (a) _ _ _ _ _ _ _ _ _ _ _[l]

{b) The actual floor area of the school hall is 700 m 2, find the corresponding floor area of
the hall of the model in cm2 .

Answer (b) _ _ _ _ _ _ _ _ _ _[1]

7 (a) Factorise 3x 2 -5x -2.

Answer (a) _ __ _ _ _ _ _ _ _ _,[ !]

(b) Hence, solve 3x 2 -5x -2 = 0.

Answer (b)x= _ _ _ _ _ _ or _ _ _ _ _[l]

8 Solve the inequalities -2~ 6-4x <6.


Show your solution on the number line below.

[Turn Over
Anglican High School Preliminary Examinations 03 2011 Math (4016/01)
5
4

Answer

-5 -4 -3 -2 -1 0 1 2 3 4 5
[2]

9 A gardener decides to water the plants every three days, trim them every four days and
fertilise them every eight days. If he perfonns these 3 tasks on Day 1, list the days that the
gardener will carry out at least 2 tasks over a 30-day period.

[2)

(Turn Over
Anglican High School Preliminary Examinations 03 2011 Math (4016/01)

6
5

B ,,,
10 In the diagram, A, B, C, D, Elie on a
circle, centre 0. Angle AED = 110°.
·""-c
-~
\

Find

(a) LACD,

Answer (a) LACD =---..,..----_..1]

(b) LABD,

Answer (b) LABD = _ _ _ ___,__ _..l]

(c) obtuse LAOD.

Answer (c) LAOD = _ _ _ _ _ _ _[l]

[Tum Over
Anglican High School Preliminary Examinations 03 2011 Math (4016/01)
6

11 A bag contains I 0 coloured balls of which 3 are grey, 5 are blue and 2 are red.
(a) Find the probability of picking either a grey or blue ball from the bag.

Answer (a) _ _ _ _ _ _ _ _ _ __ _ _ [I]

(b) When n grey balls are added into the bag. the probability of picking a grey ball
becomes 0.5. Find the value of n.

Answer (b) n = __________[2]

12 In a class of 40 students, x of them had a Sewing lesson and 18 had a Cooking lesson. Every
student needs to attend only one of the two lessons and 2 students were absent on that day.
(a) Draw a Venn diagram to illustrate the given information.

[I]

(b) From the diagram, find the number of pupils that had a Sewing lesson.

Answer {b) _ _ __ _ _ _ _ __ _ _[2]

(Turn Over
Anglican High School Preliminary Examinations 03 2011 Math (4016/01)

8
7

13 Mr Tan's monthly savings (S) is directly proportional to the square root of his monthly
income(/). His income in January and February 2011 is $3 600 and $2 500 respective!}
His savings in January is $80 more than in February. Find the amount he saved in January
2011.

[3]

14 It is given that ad+c=b+ae.


(a) Find e when a= 5, b = 3. c = -2 and d = -1.

Answer (a) e = __________[; I]

(b) Express a in terms of b, c, d and e.

Answer (b) a = ------~--[2]

[Turn Over
Anglican High School Preliminary Examinations 03 201 I Math (40I6/01)
8

1.5 T'he follo,\ing table shows the amount of $2, $10 and $50 notes that Amy. Bala and Owen
have.

$2 $10 $50
Amy 3 I I
Bala 2 0 2
Owen 3 2 0

(a) Given that M ~ [~


I
0
2
n
evaluate M[ ;~)

Answer (a) - - -- -- - -- - -- [I]

(b) Explain what the answer in (a) represents.

Answer (b) - - - -- - - -- - - -- -- - - - - - - - -- -- (1)

(c) Evaluate (1 1 1) M.

Answer (c) _ _ __ _ _ _ _ _ _ _ [I]

(d) Explain what the answer in (c) represents.

Answer (d) [I ]

(e) Write down, but do not evaluate, a product of three matrices which will give the total
amount of money that Amy, Bala and Owen have altogether.

Answer (e) - - - - - - - - - - -- -- - - - - - - - - - - - - [2]

(Turn Over
Anglican High School Preliminary Examinations 03 2011 Math (4016/01)

10
9

16 The graph shows Peter' s journey from home to market. Find


Speed (mis)
6

0 IO 20 30 40 50 60 70 80
Time (s)
(a) the acceleration for the first part of the journey,

Answer (a) _ _ _ _ __ __ _ m!s2 (I]

(b) the speed after 3 seconds of the journey,

Answer (b) - - - - - - -- --mis [2]

(c) the total distance travelled,

Answer (c) _ __ __ _ __ _ _ m[l]

(d) the average speed for the whole journey.

Answer (d) _ __ _ __ _ _ _ m/s[l]

(Turn Over
Anglican High School Preliminary Examinations 03 2011 Math (4016/01)
1
10

17 In a survey, a group of 14 people were asked the time taken to travel to work each day.
The time taken, recorded in minutes, are shown below.

31 22 40 55 11 29 37

28 29 37 46 14 29 35

The first column of the above data has already been entered on the stem and leaf diagram in
the answer space.

Complete the stem and leaf diagram and state the median.

Stem and Leaf Diagram [2]

2 2 8

Answer Median= _ __ _ _ minutes [ 1]

[Turn Over
Anglican High School Preliminary Examinations 03 2011 Math (4016/0 l)

12
11

18 (a) The cwve y = k + 5x - x 2 , where k is a constant, cuts the x-axis at (1,0) and (4,0).

(i) Find the value of k.

Answer (a)(i) k= _ __ _ _ _ _ _,[l]

(ii) Hence, sketch the graph of the cwve on the answer space below. [2]
y

•( l, I)

0
x

(b) Sketch the graph of y = (x - 3)2 - 2 on the answer space below, indicating the line
of symmetry clearly. y [3)

•(I , I)

[Turn Over
Anglican High School Preliminary Examinations 03 2011 Math (4016/01)
13
12

19 The graph of.:_+ .Y = I cuts the x-axis at A and the y-axis at B.


2 5
Find
(a) the coordinates of A and B.

Answer (a) A ( _ _ __ )[1]

B( _ __ _ _ _ )[l]

(b) the length AB,

Answer (b) _ _ __ _ _ __ _ _ _[!]

(c) the gradient of AB.

Ansll'er (c) _ _ _ _ _ _ _ _ __ _ [!]

20 The diagram shows three circles with diameters 2r, 4r and 6r respectively. Find the ratio of
the area of the shaded region to that of the unshaded region.

Anglican High School Preliminary Examinations 03 2011 Math (4016/01)

14
13

[3]

21 Two similar claypots have volumes 240cm3 and 810cm3 respectively.


(a) Find the ratio of the depth of the smaller claypot to that of the larger claypot.

Answer (a) _ _ _ _ _ _ _ _ _ _ _ [!]

(b) If the base area of the larger claypot is 72cm2, find the base area of the smaller claypot.

Answer (b) _ _ _ _ _ _ _ _ _ _ cm2 (2)

22 In the diagram, ABCD is a square, ADE and BFE are E


straight lines.
(a) Show that MEB is similar to t:JJEF.
(b) If DE= 1.5 cm and DF= 1 cm, find the area
of the square ABCD.

Answer (a)

[2]

[Turn Over
Anglican High School Preliminary Examinations 03 2011 Math (4016/01)
14

Answer (b) _ _ _ _ _ _ _ _ _ cm2 [2]

23 (a) Study and complete the table below: [1]

Number Pattern Sum of Number Pattern, S S+2


·-
2 2 4
2, 3, 2 7 9
2, 3, 4, 3, 2
2, 3, 4, s, 4, 3, 2

(b) What are the numbers in the S + 2 column commonly known as?

Answer (b) _ _ _ _ _ _ _ _ __ _[!]

(c) Find the value of S from the sum of 2, 3, 4, S, 6, 7, 8, 9, 8, 7, 6, 5, 4, 3, 2.

Answer (c) _ _ _ _ _ _ _ _ _ _[!]

(d) If S = 2 + 3 + 4 + 5 ..;_ ... + (n- 1) + n + (n- 1) + ... + 5 + 4 + 3 + 2 == 527,


find the value of n.

Answer (d) n = _ _ _ _ _ _ _ _ _[1]

(e) Write a formula connecting S ton.

Answer (e) _ _ _ _ _ _ __ _ _ _[1]

(f) Hence, find the sum of


2, 3, 4, ... '99, 100, 101, 100, 99, ... '4, 3, 2.

[Turn Over
Anglican High School Preliminary Examinations 03 2011 Math (4016/0 l)

16
15

Answer (f) _ _ _ _ _ _ _ _ _ _ _[2]

24 (a) In the answer space below, draw a quadrilateral ABCD in which the base

AB= 10 cm, LBAD = 82°, AD= 7.2 cm, LABC = 105° and LADC = 75°. [3]

(i) Construct the perpendicular bisector of A and D. [1]

(ii) Construct the angle bisector of angle BCD. [1]

(b) These two bisectors meet at P. Complete the statement below.

[Turn Over
Anglican High School Preliminary Examinations 03 2011 Math (4016/01)
Answer {b) The point P is equidistant from the lines ................. .
and .. .. .. .. .. .. .. .. .. .. and equidistant from the points
..................... and ........... ........... . [1]

--- THE END ---


Answer Scheme to (4016/01) 2011:

1 (a) 682 000

(b) 0.4 x o.73 = 2.92 x i 0·3


12 (a)

I
E

~~ =~--~
(
- -- \ !
·-.-- -- ~~
'
\
'
I
2 0.01482 I
x

'
I
I

\
18
l
/

I
: .-
3 4a 3 b(a-3b)(a + 3b) - ---- - - --- '

(b) 20
4 (a) n =2 13 $480
pq3

5 $805 14 (a) e = -2
b-c
6 (a) 18m (b) a=--
d-e

(b) 2800 cm2


(3x+ IXx-2)

[1~:]
7 (a) 15
l (a)
(b) x =--or
.., 2
..)

(b) The matrix states that Amy has $66, Bala has
8 O(x~2
$104 and Owen has $26.
9
(c) [8 3 3)
9 9th, 13th, 17th, 25th
(d) The matrix represents the total number of
-

[Turn Over
Anglican High School Preliminary Examinations 03 2011 Math (4016/0 l)

18
17

110 (a) L.ACD= 70° $2,$10 and $50 notes that


Amy, Bala and Owen has respectively.
(b) L.ABD= 70°

(c) L.AOD = 140°


(e) [l 1 I) [~ ~ ~] [I~]
3 2 0 50
11 4 16 (a) 0.5 mis"
(a) -
5
(b) 1.5 mis
(b) n=4
(c) 200m
6
(d) 2- mis
- 7

17 I I 4
2 2 8 9 9 9
3 I 5 7 7
4 0 6
5 5

Median: {29 +31) +2=30

18 (a)(i) k= -4
y

[Turn Over
Anglican High School Preliminary Examinations 03 2011 Math (4016/01)
18

-
y
\'
(b)

(~
..
(

v r

(3,-2)

19 (a) A is at (2,0) Bis at (0,5)

(b) AB ::.; 5.39 cm

(c) gradient of AB = - ~
2

20 Ratio= 4: 5

21 (a) 2:3
(b) Area= 32 cm2

22 ABCD = 9cm1
23 (a) 14, 16
23,25

(b) Perfect square numbers

(c) S = 79

(d) n2 =527 + 2

n =.J529 = 23

(e) n2 =S + 2
S:: n2 -2

(f) 10199

(Turn Over
Anglican High School Preliminary Examinations 03 2011 Math (4016/01)
20
19

24

END
Anglican High School Preliminary Examinations 03 2011 Math (4016/02)
Answer all the questions.

1. (a) In the diagram, the straight line ABC is parallel to ED and BF is parallel to CD.
LABF= 55°, LDEF= 115° and LBDC = 65°. State with reasons, the value of

A B c
(i) LBCD [I]

(ii) LABD [I]

(iii) LBDE [l]

(iv) LBFE [2]

(b) One of the interior angles of a polygon is 140°. The remaining interior angles are each
equal to 160°. Find the number of sides of the polygon. [2]

2. The line of the equations ax +by= 3 and ex - dy = 5 intersect at the point (2, I).
(a) Show that 2a + b - 2c + d= -2. [3]
(b) If a =-c and b = 2d, show that 4c - 3d= 2. [2]

7 3-x
3. (a) Express as a single fraction - [3]
2(5-x) (x + tXx-5)"

2
(b) Using completing the square method, solve the equation x -3=5x. [3]

Anglican High School Prelim 3 Examinations 2011 Mathcma1ics Paper 2 [Tum Over

s
Anglican High School Preliminary Examinations 03 2011 Math (4016/02)

4. A car travelled from town X to town Y, which were 320 km apart, at an average speed of x
km/h. A van travelled by the same route from town X to town Y at an average speed which
was 5 km/h slower than the speed of the car.

(a) Write down an expression in tenns of x, for the time taken by the car to travel
from town X to town Y. [1]

(b) Write down an expression in tenns of x, for the time taken by the van to travel
from town X to town Y. [l]

(c) Given that the difference between the time taken is 15 minutes, fonn an equation
in x and show that it reduces to x 2 - 5x - 6400 =o. [3]

(d) Solve the equation x2 - Sx - 6400 =O, giving your answer correct to 2 decimal
places. [3]

(e) Hence, find in hours and minutes, the time it took the van to travel from town
Xto town Y. [1]

5. (a) A man deposited $5 000 in a bank at a certain simple interest rate per annum. After two
years, he found that his deposit with interest was $5 080.

(i) Calculate the simple interest rate that the bank pays per annum. [2]

(ii) If the man deposits the same amount of money in another bank which gives a
compound interest rate of 0.5% per annum, how much money, in dollars and cents,
will the man receive after three years. [2]

(b) The income tax to be paid from a man's earned income is calculated as follows:
the first $10 000 of earned income is tax free,
the next $10 000 is taxed at 3.5%,
the next $5 000 is taxed at 6%,
and the remainder is taxed at 8%.

(i) If the man earns $60 000 in a year, calculate his income tax. [2]

(ii) The man's wife pays a total of$ l 280 in income tax, how much does she earn in a
year. [3]

Anglican High School Prelim 3 Examinations 2011 Mathematics Paper 2 [Tum Over
Anglican High School Preliminary Examinations 03 2011 Math (4016/02)

6. (a) The diagram below shows a pyramid with a horizontal rectangular base ABCD and a
vertex Vvertically above A. If AB= 5cm, BC= ?cm ~nd VC = 20 cm.
Calculate,

(i) the length of AC (2]


(ii) the length of VA [l]

(b) The diagram below shows a signboard suspended by three metal poles AB, AC and AD.
AC = 2.5 m and AD = 3 .5 m. Pole AC makes an angle of of 64° with the horizontal pole
AB. Calculate,

(i) the angle of depression of D from A (3)


(ii) cos LACD (2)

(c) Swimmers A and B left island X at the same time. Swimmer A swam 30 km on a bearing
of 130° from X and swimmer B swam 25 km on a bearing of285° from X. Calculate,

(i) the distance between the two swimmers, [2]


(ii) the bearing of swimmer A from swimmer B. [2]

Anglican High School Prelim 3 Examinations 2011 Mathematics Paper 2 [Tum Over
Anglican High School Preliminary Examinations 03 2011 Math (4016/02)

7. The probability that Kennie leaves his house for school before 6.30am is 0.35. The
probability of him reaching school on time at 7.25am or earlier is 0.85. If he leaves his house
after 6.30am, then the probability of him reaching school late is 0.8.

(a) Draw a probability tree diagram to show all possible outcomes. [2]
(b) Find the probability that Kennie will be late for school. [I]

8. In the diagram, t:.ABC is an equilateral triangle which has a circle inscribed in it. Wand X are
points of tangency on the circle. WY and XZ are the diameters of the circle.

(a) Find, (

(i) LCAB [I]


(ii) LXOW [2]
(iii) LWYZ [2)
(iv) LWYX [2]

(b) Prove that WX and YZ are parallel. [I]


Anglican High School Preliminary Examinations 03 2011 Math (4016/02)

9. The cumulative frequency curve below illustrates the daily wages of 76 workers in Factory A.

Number of workers
Factory A
80 ..........,..............,...................................................................................................................................................,..,..,l"'T"I"...............................................,..,..,~

c so
=
~
=
c:r
~

""'
~
~ 40
>
:c
cs
=6=
u 30

0 10 20 30 40 so 60 70 80

Daily wages (S)

(a) Use the graph to find

(i) the interquartile range, [2]

(ii) the percentage of workers earning $35 or less a day. [2]

Anglican High School Prelim 3 Examinations 2011 Mathematics Paper 2 [Tum Over
Anglican High School Preliminary Examinations 03 2011 Math (4016/02)

9. (b) (i) Copy and complete the grouped frequency table of the daily wages of the workers in
Factory A. [2]

Daily Wages (w) Mid value (x) Number of Workers(/) fx


10 < w::: 20 15 9 135
20 < w::: 30 25 13 325
30 <w :S 40 24
40 < w::: 50 17
50 < w::: 60 7
60 < w ::: 76 68

(ii) Show that the estimated mean of daily wages is approximately $37.61, correct
to 2 decimal places. [ 1]

(iii) Calculate the standard deviation of daily wages. [2]

9. (c) The daily wages of another group of 76 workers in Factory Bis illustrated by the box and
whisker diagram below.
Factory B
I I I I I

~ ~+ l- .•

0 10 20 30 40 50 60 70 80
Daily wages ($)

Compare and comment on the daily wages of the two groups of workers. [2]
Anglican High School Preliminary Examinations 03 2011 Math (4016/02)

- -
10. In the diagram, OABC is a parallelogram and Xis the midpoint of BC. OC is produced to D
such that OC =CD. OA = 4p and AB= 2q.

(a) Express the following vectors in tenns ofp and q

(i) OB
(ii) AX
-- [l]
[l]
(iii) BJj [1]

(b) Given that OX is produced to Ysuch that 30Y= 40X,- -


- -
(i) Show that BY= k BD, where k is a constant to be determined. [3]
(ii) Find the position vector P such that BYXP is a parallelogram. [2]

( ... . I va Iue o f Area 6..BXD


m) p·md the numenca [2]
Area 6.0XD

(c) The coordinates of point Xis (-6,-4) and XY = - (-lJ 1


-
. Find I OY I· [2]

Anglican High School Prelim 3 Examinations 2011 Mathematics Paper 2 {Turn Over
Anglican High School Preliminary Examinations 03 2011 Math (4016/02)

11. A toy manufacturer cuts off a top part of a hollow cone to make a frustum so that a sphere
can be placed on top of it as shown in Figure 2.

Figure I Figure 2

Figure 3 shows the cross-section of the object cut vertically through the centre. The circle
with centre 0 passes through points A, Band C. OA is produced such that OA =AX and OC is

produced such that OC =CY. The radius of the circle is 8 cm and LAOC = f( rad .
3

,, ,,0... ...
,, ,, ... ...
,, ,, ' ...

Figure 3

Calculate,

(a) the area of the shaded region as shown in Figure 3. [3]

(b) the volume of the frustum in Figure I. [5]

Anglican High School Prclim 3 Examinations 2011 Mather.iatics Paper 2 (Tum Over
Anglican High School Preliminary Examinations 03 2011 Math (4016/02)
12. Answer the whole ofthis question on a sheet of graph paper.

The variables x and y are connected by the equation y = _!_ x 2 (3- x).
2
Some corresponding values of x and y, corrected to 1 decimal place where necessary, are
given in the following table.

x -1.5 -1.3 -1 -0.5 0 0.5 l 1.5 2 2.5 3 3.3 3.5


y 5.1 3.6 2 0.4 0 0.3 1 1.7 2 1.6 0 -1.6 -3. l

(a) Using a scale of 2 cm to 1 unit on each axis, draw a horizontal x -axis for
-1.5 ~x~3.5 and a verticaJy-axis for -3.5 ~y ~ 5.5. On your axes,
plot the points given in the table and join them in a smooth curve. [3]

(b) By drawing a tangent, find the gradient of the curve at the point (2.5, 1.6). [2]

(c) Use your graph to find the largest solution of _!_ x 2 (3 - x) == 1 . [I]
2

(d) On the same axes, draw the graph of 3y + 2x == 6 for -1.5 ~ x ~ 3.5. [l]

(i) Write down thex-coordinates of the points where the two graphs intersect. (1]

(ii) These values of xis a solution of the equation Ax3 + Bx 2 + Cx + D == 0.


Find the value of A. B, C and D. [2]

END OF PAPER

A::zfican High School Prelim 3 Examinations 2011 Mathematics Paper 2 [Tum Over
Anglican High School Preliminary Examinations 03 2011 Math (4016/02)

Answers to AHS 2011 (4016/02)

1 (a) (i) LBCD = 55° 6 ( b) (i) angle of depression ~ 7 l.8v ( 1 dp)

(b) (ii) -0.899 (3sf)


(ii) LABD = 120°

(iii) LBDE = 60° (c) (i) distance ~ 53.7 km (3 sf)

(iv) LBFE = 120° (c) (ii) bearing of A from Bis 118.7° (ldp)

(b) 17

2 N.A 7
O~to school

3 5x+13 Leavohomo ~-"

-s:
before 6.30 a.a:i
Oot to school
(a) 2(x + ()(5 - x) late

4
(b) x =5.54 or -0.541 s
Leave home
after 6.30 am
~-
o~
.
Oct to school

late
to ac;hool
(a) 320
x (b) P (kennie will be late)= 0.5725.
(b) 320
x-5
(c) N.A

(d) x = 82.5 or x = - 77.539 (n.a)


(3sf)

(e) time taken = 4 hr 8 min


(nearest min)

5 (a) (i) 0.8 % 8 (i) LCAB = 60°

(a) (ii) $5075.38 (ii)LXOW= 120°

(b) (i) $3 450 (iii)L WYZ = 30°

(b) (ii) $32 875 (iv)LWYX= 60°

6 (a) (i) AC ~ 8.60m

(a) (ii) VA~ 18.I m (3sf)

Anglican High School Prelim 3 Examinations 2011 Mathematics Paper 2 [Tum Over

32
Anglican High School Preliminary Examinations 032011 Math (4016/02)

9 (a) (i)$ l 7 12 (b) Gradient= -1.875

(a) (ii) 44.7% (c) x = 2.7

(b) $14.24 (2dp)


(d) (ii) x = -1.15, 1.15, (±0.05) and 3.
(c) The median of Factory B ($41) is
higher than the median of Factory A (d) (ii) A= 3, B =-9, C = -4 and D = 12.
($37).
The interquartile range of
Factory B ($25) is wider than the
interquartile range of Factory A
($17).

10 (a) (i) OB =4p + 2q


(ii) AX=2q-2p
~
(iii) BD = -4p+2q·
(b)(i) k = 113
C) 10 •
11 3P + ]<l

C.)1
Ill i

(c) 7.62 units.

11 (a) 77.3 cmL


(b) 813 cm3
CEDAR GIRLS' SECONDARY SCHOOL
Preliminary Examination 2
Secondary Four

CANDIDATE
NAME ( ) 4/---
CENTRE INDEX
NUMBER I I I I I NUMBER I I
MATHEMATICS 4016/01
Paper1 15 August 2011

2 hours
Candidates answer on the Question Paper.

READ THESE INSTRUCTIONS FIRST

Write your Centre number, index number and name on all the work you hand in.
Write in dark blue or black pen on both sides of the paper.
You may use a pencil for any diagrams or graphs.
Do not use staples, paper clips, highlighters. glue or correction fluid.

Answer all questions.


If working is needed for any question, it must be shown with the answer.
Omission of essential working will result in loss of marks.
Calculators should be used where appropriate.
If the degree of accuracy is not specified in the question. and if the answer is not exact, give the answer to
three significant figures. Give answers in degrees to one decimal place.
For n, use either your calculator value or 3.142, unless the question requires the answer in tenns of 7t.

At the end of the examination, fasten all your work securely together.
The number of marks is given in brackets [ ] at the end of each question or part question.
The total number of marks for this paper is 80.

For Examiner's Use

Th is document consists of 17 printed pages

[Turn over

lC 3Ccdar Girls' Secondary School 4016/0 I/S4/ Prelim 2/20 I I


2

Mathematical Formulae

Compound interest

Total amount = P(l +~)n


100

Mensuration

Curved surface area of a cone = 1frl

Surface area of a sphere = 41fr 1

1
Volume of a cone= - 1fr 1 h
3

4
Volume of a sphere= - 1fr 1
3

Area of triangle ABC = ..!. ab sin C


2

Arc length = r B, where B is in radians

Sector area = ..!. r 2 B, where B is in radians


2

Trigonometry

a b c
--=--=--
sin A sin B sin C

a 2 =b 2 +c 2 -2bccosA

Statistics

Mean= 'L.fx
'LI

Standard deviation =
'L.fxl
- -- -- (I.Jr:li
Lf I.!

Cedar Girls' Secondary School 4016/0J/S4/Prelim 2/20 11


11
3
For Answer alJ the questions. For
Ezmnfntr's Examiner's
Use Use

1
1 (a) Express 2 - as a percentage.
40

(b) Find the fraction which is halfway between -~ and~.


7 7

Answer (a) % [l]

(b) [1]

. . 5 -4x x-3
2 (a) SoIvet he mequa 11ty ----.S- ~ - - + 1
2

(b) l lence write doVv1l the smallest perfect square which satisfies
5-4x x-3
--~--+I.
5 2

Answer (a) [2]

(b) [1]

Cedar Girls' Secondary School 4016/0l/S4/Prelim 2/2011 !Turn on:r


105
4
For For
&cominer's E:xaminer's
Use Use
3 Factorise fully

(a) x 2 -16a 2 ,

(b) x 2 -16a2 -2xy+8ay.

Answer (a) [1]

(b) [2]

4 (a) Simplify (2a)2 -:- 12a3 .


Sbc 10b 2

(b) Given that 311 + 9 11- 2 =I, find n.

Answer (a) [2]

(b) n= -- - (1)

S A polygon has n sides. Two of its exterior angles are 90° and 60°, and all
the remaining interior angles are 166° each. Calculate the value of n.

Answer n= (2J

Cedar Girls' Secondary School 4016/0 I/S4/Prelim 2/2011 106


5
FOT FIX
Exam~r 's Exairt1MT J
0

Us1 Us.
6 (a) Given thatx = 7 is a root of the quadratic equation x 2 +kx= 14,
find the other root of the equation.

(b) Solve the simultaneous equations

2 3
- + -= -9,
x y

.!.- s=.!..
x y

Answer (a) The other root is [2]

{b) x=

y= [21

Cedar Girls' ScconJal) School 40 I 6/0 l/S4/Prelim 2120 I I !Turn over


! 07
rw }
Exammer's E::wn
Vu C
7 The figure below shows a quadratic graph with y-intercept - 4 .
It has a minimum point (-1, - 7).
Find the equation of the graph in the form y = ax 2 +bx+ c , where a, b and c
are real numbers.

(- 1, -7)

Answer y= [2}

8 The first four terms in a sequence are 2, 6, 18, 54.

(a) Write down the next two terms in the sequence.

(b) Find an expression, in terms of n, for the nth term of the sequence.

(c) Explain why the number 50 285 is not a term of the sequence.

Cedar Girls ' Secondary School ·1016/0l/S4/Prelim 2/2011


7
For For
U.-tner 's Exam mu's
Uu Use
9 A triathlete swims 1 500 m at an average speed of 1.5 mis, cycles 40 km at an
average speed of 20 km/h and runs l 0 km in 72 minutes.

Find

(a) the time, in minutes, the triathlete takes to swim the 1 500m,

(b) the average speed for the total distance of the race.

Answer (a) _ _ _ _ _ minutes [ 1]

(b) km/h [2]

10 (a) Express 4 200 as the product of its prime factors.

(b) Find the smallest integer n such that 4 200n is a perfect square.

(c) Written as products of their prime factors, the HCF and LCM of the
numbers 4 200 and x are given below.

HCF = 2 2 x3x5 and


LCM = 2 3 x 33 x5 2 x7x11

Express x as the product of its prime factors.

Answer (a) [1]

(b) ll = [l]

(c) - - - - - - - - [l]

Cedar Girls' Secondnry School 1 1016/0l /S4/ Prclim 2/201 I (Turn over
:s
8
For For
Ezammtr's ~mine
Un Use
11 A nature reserve of area 225 km2 is represented on a map by an area of
36 cm2 •

Find

(a) the scale of the map, in the form I : n,

(b) the area of the nature reserve, in cm2, on another map whose scale is
l cm to 5 km.

Answer (a) l : [1]

(b) cm2 [2]

12 (a) r is inversely proportional to the square of s.


Find the percentage change in r \.vhen s decreases by 20%.

(b) If 17 men can complete 85% of a construction project in 8 days,


how many men are required to complete the entire project in 2 days?

lnswer (a) % [2)

(b) nwn ll I

Cedar Girls· ScCl: ndar: School 401610 l/S4/Prclim 212011 110


9
For For
Examinu's E'xaminer's
Use Use
13 (a) On the Venn diagram shown in the answer space, shade the set Au B'.

[1]

(b) E ={x : x is a positive integer less than 10 }


M = { x : - 6 < - 2x + 1 }
N ={x : x 2 + x - 6 =0}
P = {x: xis a multiple of 4}

(i) Illustrate the relationship of sets M, N and P in a Venn diagram


and indicate the elements of each set clearly.

[I]

(ii) List the eJement(s) of the set Mn N.

(iii) Find n (Nu P').

Answer (b) (ii) - - --- - [I]


(' .. ) [ 1]
lll - -- - - - - · - - -

Cedar Girls' Secondary School 4016/0 l/S4/Prelim 2/2011 !Turn over


:1
10
For For
Examiner's E:xominer's
Use Use
14 The following bar graph is taken from an advertisement.

Sales of soft drinks on


l 5t July 2011
Number of
bottles sold

35 -· - - - -
30
25
20
15
10
5

A B c D Brand of soft drinks


* Data was obtained from the canteen of a secondary school in Singapore

(a) Express the sales of Brand Bas a percentage of the total sales.

(b) The same information is shown on a pie chart. Find the ang1e of
the sector which represents the sales of Brand D.

(c) The advertisement made the claim that 'Brand C is the most
popular soft drink in Singapore in 20 11 '.
Is the claim valid? Give a reason to support your ans\.ver.

Answer (a) % [l]


0
(b) [ l]

Ans wer (c)

[2]

Cedar Girls' Secondary School 4016/01 /S4/Prelim 2/2011 112


11
For For
~r's Exam/nu's
Uu
15 The following announcement was featured in a newspaper article.

Launch of The Sapphire


By Tim Ang
timang@abnews.com.sg

LBS group announced on Tuesday that its latest condominium


development The Sapphire at Potong Pasir will go on sale from
Thursday. Consisting of 220 studio apartments and 280
two-bedroom apartments. The Sapphire is situated near reputable
schools like Cedar Girls' Secondary and it will come with first-class
facilities and luxurious interiors.
LBS group had earlier in the year launched The Emerald at Bishan
which comprises 130 studio apartments and 440 two-bedroom
apartments. All units were snapped up within a month ...

Selling Price Floor Area per apartment (square feet)


Condominium (per square foot)
($) Studio Two-Bedroom
The Sapphire 800 500 800
The Emerald 850 420 680
The Garner x 600 740

Use the information in the article and the table above to answer the questions.

(a) Calculate the total selling price of all apartments in The Sapphire.
Give your answer in standard form.

(b) Find the exact difference in the total selling price of all apartments in The
Sapphire and the total selling price of all apartments in The Emerald.
Give your answer in million dollars.

(c) The selling price of a studio apartment in The Garnet is 110% more than
the selling price of a studio apartment in The Sapphire.
Find x, the selling price per square foot for an apartment in The Garner.

Answer (a) $ - -- - - - -- - ll]


(b) million dollars [2]

(c} x = [2]

Cedar Girls' Secondary School 4016/0 I/S4/Prelim 2/20 11 !Turn over


13
12
For
Examiner's
Use
16 The table shows the number of hours spent on Facebook by a group of pupils
in a day.

Number of hours 0 1 2 3 4 5 6

Number of pupils 4 x 7 6 5 4 2

(a) If the mode is 2, write down an inequality which must be satisfied by x.

(b) If the median is 3, find the largest possible value of x.

(c) If the mean number of hours spent by each pupil is 2.75, find the value
of x.

Answer (a) _ _ _ _ [I]

(b) [I]

(c) [2]

17 In the figure, the area of Lriangk OA Bis 8 square units and OA OB.
y

x
0 A
Find
(a) e.
(b) the coordinates of A and R.

(c) tht! equation of the line AR.

0
;f nswer (a) 0 =- [l]

(b) A( _ _ , _ ) [I]

IJ( _ _ , _ ) [I ]

(c} l 1J

Cedar Girls' Secondary School 4016/0 l/S4/Prelim 212011 l


13
For
Examiner's
Use
18 A hollow container is made by joining a hemisphere of radius r cm to an open
cylinder of radius r cm and height h cm.

hem

(a) If the ratio of the capacity of the hemisphere to the capacity of the
cylinder is I : 4, show that h = ~ r .
3

{b) If the total capacity of the container is SO.?r cm3, find the radius r.
3

(c) Find the area of the metal sheet used to make the container.
Leave your answer in terms of 1T.

Answer (a)

(1 J

Answer (b) cm [2]

(c) cm2 [2]

Cedar Girls' Secondary School 4016/0 J/S4/Prelim 2/2011 !Turn over


5
14
For For
Examilll!r's E:xamlllu~
Use Uu
19 Two geometrically similar containers made of the same material, when
completely filled with water, have masses 240 g and 3.75 kg.

(a) Find, in the simplest integer form, the ratio of the length of the
smaller container to the larger container.

(b) The base area of the smaller container is 9 cm2• Find the base area
of the larger container.

Answer (a) [l]


~

(b) cm· [2]

20 ABCD is a rhombus. AX is perpendicular to BC and intersects BD at L.

A .....___

D--------~ C

(a) Prove that triangle ALD is congruent to triangle CLD.

(h) Hence, find L LCD.

Answer (a)

[21

Answer (hJ [I J

Cedar Girls' Secondary School 401610 l/S4/Prelim 212011 1


15
For 21 The diagram below shows a container which is made up of a frustum,
Examiner's
Use a cylinder and a hemisphere. Water is poured into the container to its brim at a
constant rate.
On the axes in the answer space, sketch the graph showing how the depth of
water in the container varies with time.
Depth of water

__.-- - ··--..-......

Time [2]

22 The diagram shows the cross-section of a component of a car engine.


The circular arc BC is part of a circle with centre at A, radius 13 cm and
LBAC = 0.96 radians. The centre of the semi-circle ADB is 0.
c

_4 _ _.__ _ _ _o.,._ _____ B


13 cm

Find

(a) the total perimeter of the cross-section ACBD,

(b) the total area of the cross section ACBD.

Answer (a) cm [2]


- - - - -·---
(b) cm2 [2]

Cedar Girls' Secondary School 401610 l /S4fPrelim 2120 I I !Turn over


l!7
16
For For
Examfner'.J C'xaminer i
Use u~
23 The relative positions of four islands A, B, C and D are in the shape of a
quadrilateral. B is 8 km due east of A, the bearing of C from B is 030°, the
distance between B and C is 5 km and D is 9 km from A and from C.

(a) Using a scale of l cm to 1 km, complete the construction of the


quadrilateral ABCD in the answer space below.

(b) The navy police is trying to locate a distressed ship, E which is at a


point equidistant from AB and AD, and equidistant from A and D.
By drawing suitable construction lines~ mark the location of E.

(c) Measure the length CE.

Answer (a), (b)

North

[31

Answer (c) cm [I]

Cedar Girls' Secondary School 4016/01/$4/Prelim 2/2011 1


17
For FCtT
Examiner's
24 The diagram below shows the distance-time graph for the first 50 seconds of f:JCDlllUltlt:T '1
Use the journey of a car. cm
The car travels at constant acceleration in the first 20 seconds, reaching a
speed of v m/s which it maintains for the next 15 seconds. For the last
15 seconds, the car travels at constant acceleration until it comes to rest.

Distance (m)

200

0 20 35 50 Time (t seconds)

(a) Find

(i) v,

(ii) the acceleration of the car for the last 15 seconds.

(b) On the grid in the answer space, draw the speed-time graph of the same
journey.

Answer (a) (i) [1]

(ii) m/s2 [1]

(b) (2]
Speed (mis)

0 20 35 50 Time (t seconds)

End of Paper

Cedar Girls' Secondary School 4016/0l /S4/Prelim 2/201 I [Turn over


-~
CEDAR GIRLS' SECONDARY SCHOOL
SECONDARY 4 MATHEMATICS
Answer Key for 2011 Preliminary Examination 2

PAPER 4016/1
l(a) 202.5 % 13(b)(ii) Mr.N={2}
3 13(b)(iii) 7
l(b) -
14 14(a) 25 %
2 14(b) 67.5°
2(a) x;::: l -
13 The claim is not valid
2(b) 4 - Small sample size (data taken
3(a) (x + 4a)(x - 4a) 14(c) from only one secondary school
- Data based on only one day (not
3(b) (x-4a)(x + 4a-2y) representative of the entire year)
2b
4(a) - lS(a) $( 2.672 x I 0 8 )
3ac
4(b) 4 15(b) 33.53 million dollars
5 17 15(c) I 400
6(a) -2 16(a) O~x~6
I 1 16(b) 5
6(b) x=- and y=--
3 5 16(c) 4
2
7 y=3x +6x-4 17(a) 45°
8(a) 162 and 486 l 7(b) A ( 4, 0 ) B ( 0. 4 )
8(b) 2 (3) 111
- 17(c) y=-x+4
18(b) 2cm
It is not a multiple of 21
8(c) I
It is not an even number 18(c) 29-1l' cm2
3
9(a) 16~ minutes
19(a) 2:5
3 19(b) 56.25 cm2
9(b) 14.8 krn/b AD =DC (sides of rhombus)
lO(a) 2 x3x5 2 x7
3 LADB = LCDB( diagonal of
lO(b) n= 42 20(a) rhombus bisects LADC)
DL is common side
lO(c) 2 2 x3 3 x5xll :. tMLD = LJCLD (SAS)
ll(a) 1 : 250 000 20(b) 90°
ll(b) 9cmt Depth
12(a) 56.25 %
12(b) 80 men

'm:J
21
13(a)

Time

'W 9G
22(a) 45.9 cm
p
13(b)(i) ~cp 22(b) 147 cm2
-

5 6 7 23(c) 6.2 i. 0.1 cm


l21
CEDAR GIRLS' SECONDARY SCHOOL
SECONDARY 4 MATHEMATICS
Answer Key for 2011 Preliminary Examination 2

PAPER 4016/1
l(a) 202.5 % 13(b)(ii) Mr1N={2}
3 13(b)(iii) 7
l(b) -
14 14(a) 25%
2 14(b) 67.5°
2(a) x~ l -
13 The claim is not valid
2(b) 4 - Small sample size (data taken
3(a) (x + 4a)(x -4a) 14(c) from only one secondary school
- Data based on only one day (not
3(b) (x-4a)(x + 4a-2y) representative of the entire year)
2b
"(a) - lS(a) $(2.672x 10 8 )
3ac
4(b) 4 15(b) 33.53 mill ion dollars
5 17 lS(c) 1 400
6(a) -2 16(a) O~x~6
I 1 16(b) 5
6(b) x=- and y= - -
3 5 16(c) 4
2
7 y = 3x +6x-4 17(a) 45°
8(a) 162 and 486 l 7(b) A ( 4, 0 ), B ( 0, 4 )
8(b) 2(3)"-I 17(c) y=-x+4
18(b) 2cm
It is not a multiple of 2 /
8(c) I
It is not an even number 18(c) 29-.1l" cm2
3
9(a) 1 6~ minutes
19(a) 2:5
3 19(b) 56.25 cm2
9(b) 14.8 km/h AD =DC (sides of rhombus)
lO(a) 2 3 x3x5 2 x7 LADB = LCDB( diagonal of
lO(b) n =42 20(a) rhombus bisects LADC)
DL is common side
lO(c) 2 2 x3 3 x5xl l : . MLD = LJCLD (SAS)
ll(a) 1 : 250 000 20(b) 90°
ll(b) 9cmJ. Depth
12(a) 56.25 %
J2(b) 80 men

'm:J
21
13(a)

T ime

13(b)(i)
w /:J
f,

s 6 7
p
22(a)

22(b)

23(c)
45.9 cm

147 cm2

6.2 ± 0.1 cm
CEDAR GIRLS' SECONDARY SCHOOL
Preliminary Examination 2
Secondary Four

MATHEMATICS 4016/02
Paper 2 16 August 2011

2 hours 30 minutes
Additional Materials: Answer Paper
Graph paper (1 sheet)

READ THESE INSTRUCTIONS FIRST

Write your Centre number. index number and name on all the work you hand in.
Write in dark blue or black pen on both sides of the paper.
You may use a pencil for any diagrams or graphs.
Do not use staples, paper clips, highlighters, glue or correction fluid.

Answer all questions.


If working is needed for any question it must be shown w ith the answer.
Omission of essential working will result in loss of marks
Calculators should be used where appropriate.
If the degree of accuracy 1s not specified in the question, and if the answer is not exact. give the
answer to three significant figures. Give answers in degrees to one decimal place.
For 1t, use either your calculator value or 3.142, unless the question requires the answer in terms of 1t.

At the end of the examination, fasten all your work securely together.
The number of marks is given in brackets [ ] at the end of each question or part question.
The tot al n umber of marks for this paper is 100.

This document consists of 12 printed pages.

[Turn over

3
Mathematical Formulae

Compound interest

Total amount= P(l + ~)n


100

Mensuration

Curved surface area of a cone = 1t rl

Surface area of a sphere= 4nr 1

Volume of a cone - !._1tr 2 h


3

4
Volume of a sphere - -;rr 1
3

Area of triangle ABC- !..ab sin C


.... 2

Arc length = rO. where 8 is in radians

Sector area = _!_ r 2 B , where B is in radians


2

Trigonometry

a b c
--=--=--
sin A sin B sin C

a2 =b
2
+ c2 - 2bc cos A

Sratistics

l. ·fr·
Mean=-
l.f

L:fx
Standard deviation = - --
Lf
2
('-I./x)
-
2..f
2
3

Answer all the questions.

3
1 ( a) .
G iven th at y = k - x ,

express x m tenns o f y, k and 1r. [2]
3
7CX +1

1 3
(b) Express as a single fraction in its simplest form - [2]
2(x-2) 2 x(x-2)

(c) (i) Express x 2 -~x-5 inthefonn (x+a) 2 +b. [l]


4

(ii) Hence solve the equation x 2 -~x-5 = 0, giving your answers correct to
4
two decimal places. [3]

2 Mr Goh bought a home entertainment system during the Year-End Sale on hire
purchase. He paid a deposit of 20% of the selling price. The balance of the payment
was $8 000.

(a) Calculate the selling price of the system. [1]

(b) Mr Goh paid the balance of the payment in monthly instalments of $380,
charged at x % per annwn simple interest for 2 years. Find the value of x. [3]

(c) Mr Goh could have taken a loan to pay the outstanding balance. lf the finance
company charged him a compound interest of 6.5 % per annum with repayment
period of 2 years, would you recommend that he takes up the loan instead of
paying in instalments? State your reason clearly. [3]

Cedar Girls' Secondal) School 4016/02tS4/Prelun 2/2011 [Turn over


'5
4

3
M

Three points, A, Band C, lie on a horizontal field. A is due North of Band the
bearing of C from A is 076°. M is vertically above A. Angle J,/BA = 43°, AB = 12 m
and AC 20 m.

(a ) Calculate

(i) MA. [2]

(ii) the angle of depression of C from ,\{ (2)

(iii} RC. [2]

(iv) the area of triangle ABC [2]

(b) P is a point on BC such that the nngle of elevation of M from P is the greatest.
Calculate the angle of elevation of \f from P. [3 J

Cedar Girls' Secondary School 40 16/02tS4 'Prclim :Y2011


12
s
4 A car travels x kilometres on each litre of petrol when it is driven on rough terrain.
It can travel 2 more kilometres on each litre of petrol when it is driven on an
expressway.

(a) Write down an expression, in terms of x, for the number of litres of petrol used
when the car is driven on rough terrain for 400 km. [ 1]

(b) Write down an expression, in terms of x, for the number of litres of petrol used
when the car is driven on an expressway for 400 km. [l]

(c) The car uses 5 litres less of petrol when it is driven on an expressway for
400 km compared to when it is driven on rough terrain. Write down an
equation in x to represent this information, and show that it reduces to
x 2 + 2x - 160 = 0 . (3]

(d) Solve the equation x 2 + 2x- l60 = 0, giving both answers correct to
two decimal places. [3]

(e) Calculate the number of litres of petrol used when the car is driven on an
expressway for 100 km . [2)

Cedar Girls' Secondary School 40 I 6/02/S4/Prelim 2/2011 fTuro over


!27
6

5 (a) It is given that AB is ( ~5).

(i) Calculate j2AB'. [1 ]

(ii) Calculate the gradient of the line AB. (1]

(iii) Given that A is the point ( - 5, 2), find the coordinates of B. [l]

(b)
p

S ~-----------R.___ _ __..__ __. 9.


a

In the diagram, QR = a and ST= b. R is the point on QS such that QS = 3QR.


U is the point on RP such that RP= 4R U. T is the midpoint of SP.

(i) Express, as simply as possible, in terms of a and/or b,

(a) RS, [ l]

(b) RP, [l]

(c) UQ, [2]

(d) TQ. [l]

(ii) Write down two facts about the points T, U and Q. [2)

(iii) Calculate the value of

area of triangle PQU


(a) area of trianglePQR ' [I]

area of triangleRUQ
(b) area of triangle PQS [2]

Cedar Girls' Secondary School 4016/02/S4/Prelim 2/2011


l
7

6
A

In the diagram, DE is a tangent to the circle \Vilh centre 0. ABC and AOF are
straight Jines and L.CBE = 61°.

(a) Giving your reasons, find

(i) angle AOE, [2]

(ii) angle AFE. [2]

(iii) angle AEO, [2]

(iv) angle AED. [2]

(b) Are the lines AE and DC parallel? Give a reason for your answer. (2)

Cedar Girls' Secondary School 40 I 6/02/S4/Prchm 2/20 I I (Turn over


8

7 (a) A four-sided unbiased die and a six-sided unbiased die are thrown at the same
time. The score of each die is noted.

(i) Draw a possibility diagram to show the outcomes of the throw. (1]

(ii) Find, as a fraction in its simplest form, the probability that

(a) the sum of the scores is 7, [l]

(b) the product of the scores is a multiple of 3. [1]

(b) Box A contains 3 cups of chocolate ice-cream and 5 cups of strawberry


ice-cream. Box B contains 2 cups of chocolate ice-cream, 4 cups of strawberry
ice-cream and 3 cups of vanilla ice-cream. A cup of ice-cream is selected at
random from box A. It is then placed in box B before a cup of ice-cream is
selected at random from box B.

(i) Draw a tree diagram to show the probabilities of the possible outcomes. (2]

(ii) Find, as a fraction in its simplest form, the probability that

(a) the two cups of ice-cream selected are of the same flavour, [2]

(b) the second cup of ice-cream selected is strawberry. (2]


9

8 Ashley, Ben and Celia are salespt:rsons at a travel agency. The sales thal they
obtained for each of the three tour packages in May and June are shown in the table
below.

May June
Ashley Ben Celia Ashley Ben Celia
Package 1 5 8 6 9 14 15
Package 2 7 4 5 13 10 9
Package 3 4· 3 5 12 11 10

The infonnation for the monlh of May can be represented by the matrix

M= 7
5 48 5.
6)
(4 1 5
(a) The information for the monlh of June is represented by the matrix J .

(i) Write do\vn the matrix J . [ I]

(ii) Evaluate J - M
Stale \\.hat Lhe clements of J - M represent. [2]

(b) Write down an appropriate matrix X and use it to evaluate the matrix product
M X ~hich gives the total number of sales of each package for the month of
May. 12]

(c) The prices of packages I. 2 and 3 are $1 000. $1 500 and$ 3 000 respecti\ ely.
The sales commission for each of the packages is 8% of the price.
This information can be represented by a row matrix C.

(i) Write do\vn the matrix C. 11 J


(ii) E'valuate the matnx product CM
State what the clements of C M represent. 12]

Cedar Girls' Sccondal) "ichool -1016 021S.t Prclirn 212011 (Turn over
31
10

9 The cumulative frequency graph below shows the distribution of marks of 300
students from School A in an English examination.

Cumulative Frequency
. ~__;__ I
~--
.~ .. +
r - - - ---
+ • +

--- I ~t:=~~I -1 .- - -

____:!__vv . - . . . - -·-j--1· -
- I . +
~
>-

-.·-·
- - --
..
--"j"-
.. . ·---
~!. .- . -
._ . I·. - -
..
-... . .
- -----
-
+
,_
~,.;e ,,

--. f--,----
-
~

-- p::::.:: --·-
-.-..- -"~""

-- ..
• + +

.. . -- i--.-·-
. I
I - -- ·--·- -~-

- --. --
+-
- -. . ...
I
+
.-
- - -- .--
- -
[

.._ -
H-00 - -
- ...
.
1
..__ - ~

--
- - ,.·- ~ ~

I
. -1
/- -- - ... ~

I
-1 -

- -l
I
- 1--50 -- --- . - .
-
.
I
I './ - --- ·-
..
- f--
-
._ + ..
Ii -
-YOO - --
J
f
.
t
I I .
.
.. .
.
I
' ..
--- --- - · -
---:n.; -
~--1
I

/.
Y· i
-
.
- +

..
.
I
I
.
.
--

t.
i.
..• I•
I

- - --
-1 +
.. . ...--·
+
.

~ ...- .
......... r
~ ..
t +
'
+ ..
' t -- -- --
L---"" ' I
i

I 0 ---- '0
.
1-· "'a rn .
. H)
I

1no • +

Marks
I
--
(a) Find the median mark. (1)

(b) Find the interquartile range. (2]

(c) The top 20% of the students obtain an Al grade in the examination. Find the
lowest mark a student must score to obtain an Al grade. [ ll

300 students from School B took the same examination. The table below shows the
information for this school.

Median 65
Interquartile range 37

(d) Compare briefly, the results for the two schools. (2)

Cedar Girls' Secondary School 40 I 6i02/S4/ Prcl im 2120 I I


11

(e) Copy an~ complete the grouped frequency table of the marks scored by the 300
students m School A.

Marks 0 <x $ 20 20 <x $ 40 40 <x < 60 60 <x < 80 80 <x < 100
Frequency 30 125 [2]
(f) Using your grouped frequency table, calculate an estimate of

(i) the mean mark,


[2]
(ii) the standard deviation.
[2]

12

10 Answer the whole of this question on a sheet of graph paper.

The variables x and y are connected by the equation y = 3 + x - ~.


x

The table below gives some values of x and the corresponding values of y.

x 0.3 0.4 0.5 0.6 0.8 I 1.3 1.6


y -7.81 - 2.85 -0.5 p 2.23 3 q 4.21

(a) Find the value of p and of q. [1]

(b) Using a scale of 1 cm to represent 0.1 unit, draw a horizontal x-axis for
0.3 $ x $ 1.6.
Using a scale of 1 cm to represent I unit, draw a vertical y-axis for -10 Sy S 5.
On your axes, plot the points given in the table and join them with a smooth
curve. [3]

(c) Use your graph to find the value of x in the range 0.3 $ x $ 1.6 for which
l
x-- < - 8. [2]
x2

(d) Find the x-coordinate of the point on the curve at which the gradient of the
tangent is 3. [2]

(e) (i) On the same axes, draw the graph of y = -5x + 4. (1]

(ii) Write down the x-coordinate of the point at which the two graphs
intersect. [ 1]

1
(iii) Find the equation, in the form - 6x + ax 2 + b = 0, which is satisfied
by the value of x found in (e)(ii). (2]

End of Paper
CEDAR GIRLS' SECONDARY SCHOOL
SECONDARY 4 MATHEMATICS
Answer Key for 2011 Preliminary Examination 2

Mathematics 4016/2
Sbic - 3 1
x=v~-:;
la UQ= - - a--b
2 2

lb 12-Sx Sb id -
TQ = -3 a - b
2x(x-2) 2
Sbii - = -1 TQ
- . T, U and Q are collinear
(x -~)'
lei UQ
_5_2_ 2
8 64
U is the midooint of TQ
lcii x = 2.64' x = -1.89 (2 d.p.) Sb iii a 3
-
4
2a $10 000 Sbiiib l
-
12
2b x=7 6ai L.AOE = 122°
2c Total amount after interest from 6aii LAFE= 6 1°
finance company= $9 073.80 6aiii LAEO- 29°
Yes. l [e should take up the loan from 6aiv LAED = 61°
the finance company as the interest is 6b Yes. LEDC = LAED (alternate angles,
lower. AE and DC are oarallel).
3ai .\l-1 l l.2 rn 7ai

3aii L of depression= 29.2° (1 d.p.) 4 x x x x x x


3aiii BC 25.7 m 3 x x x x x x
3aiv 116 mz
3b 51.0°(1 d.p.) 2 x x x x x x
..aa N o. ot·1·1trcs ot- pelrol = -400 I x x x x x x
x
4 I 2 3 4 5 6
4b
No. of litres of petrol = 00
x+2
4c 400 ·100 -5 I
- ---- 7aiia -
x x+2 6
4d x I 1.69, x = - 13.69 (2 d.p.)
..ae No. of litres of petrol · 7.31 I I
7a iib -
2
5ai 11.7 units 7bi On next page

Sa ii 5 7hiia 17
- -
3 40
Sa iii B(-2,- 3) 7biib 37
-
-
80
Sbia --
RS =2a
Sui 9 14
.._
J [1 3 10 1:]
Sbib -RP 12 11 10
'~' t ~h
8aii 9c 81 marks

J-M=(: : :J
8 8 5
9d Pupils from school A performed better
as their median is higher. The results o~
The elements represent the difference pupils from school A are more
in the number of sales by each consistent as the interquartile range
salesperson between the months of is lower.
June and Mav for each package.
Sb 9fi Mean = 64.3 marks
9fii Standard Deviation = 21.9 marks
lOa p = 0.82, q = 3.71
Sci C=(80 120 240) lOc Drawy =- 5
0.3::;; x < 0.35 (± 0.02)
8cii CM = (2200 1840 2280) lOd x=l
The elements represent the sales lOeii x = 0.61(± 0.02)
commission each salesperson
received in the month of May. lOeiii -6x 3 + x 2 + 1 = 0
9a 70 marks I

9b 27 marks
7bi

9e Marks 0 <x ~ 20 20 < x ~ 40 40 < x $ 60 60 < x $ 80 80 < x $ 100


Frequency 15 30 55 125 75

I Ob -
'
: . ·' • , ; ~ ;
! : : T. :· : ; .
: ! :
r: ....
: : :
;· : : t
:. •
I
·· i . '
! ;:
: :· ·:
:
..

••n•·•i~(. .; .. C..., ,:-J!!•~: ,.. •1 • 1• ,.f•• ~ •(

~~
'.; ~~~l:j.~i ,: · I :
t 2 • i , · : · .' , : ·1 • • ,Y ·' 3 +,· x :..,..i. : . .
~- :: ~~ :
1
i c : <s ,, :; A- ·'

~ . ____________.:- ~ . 7~ ---·__ : . :~ _: ~ .
: io o. 1 · o. a ~ o.l .;·•r
; o. :;> ~ o. e 0. 1 • o. e ' ~ 1. 1 "' 1.t · • > ' · " : i..s 1. 1
t -2 • ' ~ ~:· ~ ~-
: ; : ~ ~ : ~ t : ~ ~ Y. = -?x +~ 4
: _: : : ~ ~ = : • :

::::t:~~ .:::;:·::~. :::j:::~:::r::;:::::;··:.~::.:j::.·;::::;: : ~::-:'. .:;~ ~~:·~::::(:i::::;:: :i. . ;: .: ~ .:::t::.[•.::I :..,...·;·: :;: : :~.···~· ·::::::.!:::.~::::

I[-=!-J~t-[1' i~HJI!:i~~ ~II~J=:J}: [Et-:~~~ ~:;:_i=~-i~[":.;:t~


••·••i-·•• ,, ...,••••• f.,,,.;,.,.j,.,.~ .. (.. , .; "•·••• i• - '"' ... , .... ~. ·•I•• · •••"'•••• ·••'''''' I •••0:·•• •(• •.:••· l•••••l····t·· i·•• C.• ~· •4' •t ••·•Jo'•••• ··•••• · ..... , •••· v•·-
·• ..::12 • L . ..:... t· .. : . ~- ~ ...~ ... ~- _, .. ~.. : . . ......... :· .. :... ..~ __ ;__ -~ .....~ ..~h - ! .....L .....
Name Register Number Class

CRESCENT GIRLS' SCHOOL


SECONDARY FOUR
2011 PRELIMINARY EXAMINATION

MATHEMATICS 4016/01
Paper 1
15AUG2011
Candidates answer on the Question Paper 2 hours

READ THESE INSTRUCTIONS FIRST

Write your name and register number on all the work you hand in.
Write in dark blue or black pen.
You may use a pencil for any diagram~ or graphs.
Do not use staples, paper clips, highlighters, glue or correction fluid.

Answer all questions.


If working is needed for any question, it must be shown in the space below that question.
Omission of essential working will result in loss of marks.
You are expected to use a scientific calculator to evaluate explicit numerical expressions.
If the degree of accuracy is not specified in the question, and if the answer is not exact, .
give the answer to three significant figures. Give answers in degrees to one decimal place.
For 1t, use either your calculator value or 3.142, unless the question requires the answer in
terms of 1t.

At the end of the examination, fasten all your work together.


The number of marks is given in brackets [ ] at the end of each question or part question.
The total of the marks for this paper is 80.

This document consists of 12 printed pages, including the cover page.

rTnrn nv"r
3

Answer all the questions.


1 Evaluate

3 S.8+84.07
(a)
7.0SX7.003

(b) (7.13 x 103 )-:- (27.l x 10- 4 - 0.147 x 10- 2 ). Give your answer in standard form.

Answer(a) ........................................ (I]

(b) ... ............... .................. ......... [l]

2 Two maps are drawn of a new town. On the first map, a school appears with an area of 3 cm2 and on the
second map with an area of 12 cm2 . Given that the scale of the first map is 1:80 000, find the scale of the
second map in the form I :n.

Answer ..................................... [2]

(Turn over
CGS Prehm Exam 2011 4016/01
4

3 A company charges $530.40 for transporting goods weighing 48 kg for 36 km. If the charge is 35cents
per kg per km for the first Skm, how much less is the charge per kg per km for every subsequent km?

Answer ........................... ......... cent [2]

4 The diagram shows a quadrant of a circle, centre 0 and radius 28 cm.Mis the midpoint of OB and a
semi-circle is drawn with MB as diameter. The other semi-circle in the quadrant has diameter OA.
Find the perimeter of the shaded region. Give your answer in the form a+ brr.

28

Answer: .................... .cm [2]

CGS Prelim Exam 2011 4016/0 I

3E
5

5 (a) Find the range of values of x for which 1 - 3x ;:::: 4x - 2 and -2x < 7 and then represent the
solution set on the number line.

Answer (a) .................................. [1]

(1]

) Solve the simultaneous equations.


1
2x + y = 3
1 2
-x - -y=6
4 s

Answer (b) x = ................................ .

y = ................................. [3]

(Turn over
(GS Prehm Exam 2011 4016101
6

~<.>
.....
0
...
0
.0
§
z

During a traffic survey, the number of people in each car passing through a checkpoint was noted.
The results of the survey are shown in the bar chart.
(a) Explain why this bar chart might be considered misleading.
(b) Express the total number of cars which had at least 2 people as a fraction of the total number of cars.
Give your answer in its lowest terms.
(c) The same information is to be shown in a pie chart.
Find the angle of the sector which represents cars with three people.

Answer (a) ....................................................................... .. ..................................... .

......................... ......... ......... ..... .. ... ........... .............. ... ................. ............. (1)

Answer (b) .. .. . .. . .. .. .. .. .. .. .. . .. . . .. [I]

(c) ............ .... .... .. .... ... [ l]

It
7

- A flash drive which Wendy bought has 20 gigabytes of memory space. The average size of a digital file
is 4.5 megabytes.
(a) 20 gigabytes can be written as k trilllon bytes. Find k.
{b) Find the number of complete digital files the flash drive can store.
(c) Given that 680 digital files have been stored, find the size of the free memory space in bytes. Give
your answer in standard form.

Answer (a) k = ........................................ [l]

(b) ............................................ [l]

(c) ................................... byte~ [I]


AB, BC and CD are adjacent sides of a regular polygon. Given that LCAB = 7.5°, calculate
(a) the exterior angle of the polygon,
1b) the number of sides of the polygon,
(c) LAYD.
c

Answer (a) ... ........................................... [l]

(b) .... ................ ······ ...... ...... [I]

(c) LAYD = ...... ...... ............. [I]


[Turn over
aiS Prtl1m Exam 2011 4016/01
8

9 The braking distance of a car is directly proportional to the square of its speed.
When the speed is p metres per second, the braking distance is 6 m.
When the speed is increased by 250%, find
(a) an expression for the speed of the car,
(b) the braking distance,
(c) the percentage increase in the braking distance.

Answer (a) ......................................... mis (1]


(b) ... ...... ...... ........................... m [1]

(c) ... ...... ................................... % (1]


10 The height of I 0 pupils, in cm, are as shown below.
152, 168, 176, 156, 163, 180, 158, 170, 175, x
(a) If the mode of the height of the pupils is 158 cm, state the value of x and hence find the median
height of the I 0 pupils.
(b) · If the median height is t 69 cm, find the range of values of x.

Answer (a) x = ............................................. [I]


Median height = ................... cm [I]
(b) ................................................. [l]

CGS Prclim Exam 2011 4016/01


9

11 The vertices of triangle PQR are (-2, -2), (2, 4) and (2, 1) respectively.Sis the point (2, -2) and
Tis the point (2, 6).
(a) Find the length of PQ.
(b) Find the area of t:.PQR.
(c) Expressing your answer as a fraction in its lowest terms, write down the values of
(i) tanLPQT,
(ii) COSLPRQ.

'I

T
e ><

4 Q

3 4 5 x

p
-3
Answer (a) ........................ units [I]

{b) ........................ units 2 [1]

(c)(i) .... .... .......... ........ [l]

(ii) .......................... [I]

12 (a) Factorise completely 9a2 - b 2 - 2bc - c 2


(b) Factorise 2x 2 +ax+ 6 completely. Hence, express 286 as the product of three prime factors.

Answer (a) ............................................. [2]

{b) .............................................. [I]

............................................... (1)

[Turn over
10

13 The following table shows the nutrition contents (in grams) of a packet of milk and a serving of
instant noodles.
Protein Fat Carbohydrate
Milk 6 7 10
Noodles 11 22 59

A boy drinks 3 packets of milk and has 2 servings of instant noodles.


(a) Find (3
6
11 2
~~).
2) ( J
(b) Explain what your answer to (a) represents.
(c) Write down a matrix C such that (3 2) (
6
11
J2 ~~)c gives the total amount of nutrients
taken by the boy.
(d) If the boy drinks only 3 packets of milk and the 2 servings of instant noodles are taken by his

friend, write down the matrix A such that A(


6
11 2
;~) represents the amount of protein, fat J
and carbohydrate that the boy and his friend has taken respectively.

Answer (a) .............................. (11

(b) ......... . ............................................................................................................ .

. . . . ... . . . . . . . . . . . . . . . . . . . . . . . . . . . . . . . . . . . . . . . . . . . . . . . . . . . . . . . . . . . . . . . . . . . . . . . . . . . . .. . . . . . . . .. . . . . . . . . . . . . . .. . . . . . . . . . . .. [l]

(c) . . . .. . . . .. . .. . .. . ... .. . .. . ... .. [I]

(d) ................................ (1)


14 (a) (i) Express 90 as the product of its prime factors.
(ii) The lowest common multiple of 6, IS and xis 90.
Find the two possible values of x which are odd numbers.
(b) The highest common factor of 90 and an integer y is 5. Write down the least possible integer value of
y larger than 5.

Answer (a) (I} .............................. [I]


(ii) x = ......... or ............ [2]
(b) .. .. .... .. ....... ... ...... .. ... [l]

CGS Prelim Exam 2011 4016/01


11

15 Given that n(~)=55, n(A)=28,n(B)=x and n(A('IB)=5.


(a) Find n(A' U B).
(b) Express, in terms ofx, n(AuB) and n(A' ('I B').
}Ience or otherwise, find the greatest and the smallest possible values of x.

Answer (a) ............................... [I]

(b) n(AuB) .................. [I]

n(A' ('\ B') ..................... [I]


Greatest x = ............... [I]
Smallestx = ............... [I]

16 In the diagram. ABCD is a parallelogram. The AT:TD = l :2 and BTP is a straight line. CT meets BD at
point Q and is parallel to DP. Find the value of
Area of APTD
(a)
Area of ATBC .·
Area of llPTD
(b)
Area of ADTC
TQ ;;
(c)
CQ

Answer (a) ............................... [I]

{b) ................................ [!]

(c) ........................ ........ [I]

[Turn over
12

17 The diagram is the distance-time graph for the first 25 seconds of a car's journey. The car started from
rest.

_J
Distance
400 f-
(metres) l
I

10 15 2.5
'
. J
Time (t seconds)

(a) Find the speed of the car when t = 25.


(b) The car moved with a constant acceleration for the first 10 seconds. Find this acceleration.
(c) On the grid in the answer space, draw the speed-time graph for the same journey.

30
1 t
-i
Speed
(metres per second) 20 .. !
I' I
I
!
~o
I' I I I
I

i -t t
..
'

0 5 1 10 15 2P 25

Time (t seconds)
(2)

Answer (a) ........................ mis [l]

(b) ........................ m/s2 [l]


13

Two similar right pyramids shown below, A and B, have base areas of 28 cm2 and 63 cm2 .
The density of pyramid A is thrice the density of pyramid B.
(a) Find, in its simplest integer form, the ratio of the height of pyramid A to the height of pyramid B.
(b) The total surface area of the triangular faces of pyramid A is 68 cm 2 •
Find the total surface area of the triangular faces of pyramid B.
(c) The volume of pyramid A is 336 cm3, find
(i) the height of pyramid A,
(ii) the volume of pyramid B.
(d) The mass of pyramid Bis 189 g. Find the mass of pyramid A.

Pyramid A

Pyramid B

Answer (a) ............................ [I]

(b) ....................... cm2 [I]

(c){i) ..................... cm [l]

(ii} .................... cm3 [I]

(d) ........................... g [2]

[Turn over
14

19 Town X and Town Y are 36 km apart. A cyclist left Town X for Town Y at 08 00, cycling at 16 km/h.
A motorist left Town Y for Town X at the same time as the cyclist, travelling at 24 km/h.
(a) Draw the distance-time graphs of the cyclist and the motorist on the same axes.
(b) Use your graph to find
(i) the time the cyclist passed the motorist,
(ii) the distance from Town Y when the cyclist passed the motorist,
(iii) how much longer the cyclist took to complete the journey compared to the motorist,
(iv) the time when the cyclist and the motorist were 26 km apart.

Answer (a) [2]

Answer (b) (V.... ........ . ... . ........ ... . .. [1]

(ii) ..................................... km [J]

(iii) ........................ minute [l]

(iv)................................ [1]

CGS Prelim Exam 2011 4016/01


15

... The scale drawing in the answer space below shows the positions of three shops A, Band C. A is due
North of B.

(a) Find the bearing of A from C.

Answer (a) .. .............................................. (l]

(b) On the map, the shop Dis 6 cm from Con a bearing of057°. Find and label the position of shop D.
(c) A new shop Eis to be built equidistant from AB, BC and CD. By constructing bisectors, find and
label the position of the new shop E.
(d) Given that BC= 1600 m, calculate the actual distance from D to the new shop E .

.. er (b) and (c)

[3]

Answer (d) ..... . .. . . . ................ .m [l]


(Turn over
CXiS Prelim Exam 2011 4016/01
16

____.. .-., ._. 2 ---+ - 3-


21 In the diagram, AB= 16p, AD= 4p + 15q, BE= 3BD and DC= 2AB.
(a) Express, in the form op + bq,
(i) 875,
(ii) IfC.
(iii) Af.
(b) Use your answers to parts (a)(ii) and (a)(iii) to explain\\ hy AE is parallel to BC.
(c) Show that llABE is similar to llCDB
(d) Hence find the area of llCDB if the area of llABE is 36 cm~.

D
c

4p + 15q

A 16p 8

Answer {a) (i) Bi5 = .. ... ...... ....... ... ..... [I)

(ii) BC = ......................... [I]

(iii) lfE = .. ....................... [I)

(b) ..................................................................................................................... [ll


(c) In MB E and llC DB,

[2]
(d} ................. ............ cm 2 [I]
COS Prehm Exam 2011 4016/01
er Key 4 2 2
16 (a) - (b) - (c) -
9 3 3
I a) 1.22 (b) 5.75 x 106 17 (a) 20 (b) 2
- l : 40 000
5
14 + 35rr
1
(a) -3-
2
< x <- -37
·1

i 1 t• "l l h l

·2 ·1 0 .) 1
·3.5 18 (a) 2:3 (b) 153 (c)(i) 36 (ii) 1134 (d) 168
1"
I 'q
(b) X = 16 y = -5 I
·~· ·- -f 1
(a) The bar chart might be considered
misleading as the vertical axis does not start
from zero. Hence the proportion of the length of
the chart is not accurate.
19 (a)
(b) £20 (c) 144°
(b) (i) 0854 (ii) 21.5 (iii) 45 (iv) 08 15
- (a) 0.02 (b) 4444 (c) I.694xl0 10 20 (a) 317° (d) 1429 i
' (a) 15° (b)24 (c) 165°
(a) 3.5p (b) 73.5 (c) 1125% (b) , (c)
0 (a) 158, median height= 165.5 cm

(b) x ~ 170

11 (a) 7.2 1 (c)(i) -~ (ii) -~


(b) 6
3 s
l l (a) (3a - b - c)(3a + b + c)

(b) 2(x+ l)(x +3)


0
(c)2x I I x 13
13 (a) (40 65 148)
c
(b) The amount of protein, fat and carbohydrate
21 (a)(i) lSq - 12p (ii) 12p + lSq
respectively in 3 packets of milk and 2 servings
of instant noodles.
(iii) 8p + 10q (b) 2BC = 3AE; Since
(c) G) (d) (~ ~) AE = kBC or AE is a scalar multiple of BC, AE
2
14 (a) (i) 2 x 3 x 5 (ii) 9 or 45 (iii) 25 is parallel to BC.
15 (a) 32 (b) n(AUB) = 23 + x (c) In aABE and ClCDB,
LABE = LCDB (alt.Ls ;AE//BC)
n(A'nB)=32-x,
LAEB = LCBD (alt.Ls ;AE//BC)
Greatest x = 32 :. MBE is similar to t.CBD (d) 8 1
~m:. l lP.<:t Y = S
Mathematics Paper 2
4

3 Two friends John and James recorded their time travelled from Town A to Town B.
The cumulative frequency curve below shows the time travelled by James on 50 trips .
• ~ ! I. ~ ili:
~++ - -~~ ~.L~•.;:~-+,_,.+..,..,__-t-
..........i---t----""'i-1-'

(i) Copy and complete the grouped frequency table of the time travelled by James. (2]
Times, x (min) frequency,/
130 <x ~ 135
135 < x ~140

140 <x ~ 145


145 <x ~ 150
150<x ~155

155 <x ~ 160

(ii) Find the mean and standard deviation. [3]


(iii) The average time travelled by John on the 50 trips was 146.5 minutes and with a
standard deviation of 4.93 minutes, comment on the travel time spent by John and
James in two different ways. [2]

CGS Prelim Exam 2011 4016/02


5

4 A trader bought some paraffin for $400. He paid $x for each litre of paraffin.
(a) Find, in terms of x,
(i) the number of litres of paraffin he bought, [1]
(ii) the selling price per litre, if he lost 3 litres of paraffin and sold the remainder
of the paraffin for 80 cents per litre more than he paid for it. [l]
(b) He made a profit of $36. Write down an equation in x to represent the information,
and show that it reduces to 15x 2 +192 x-1600 =0. (3]
(c) Solve the equation t 5 x 2 + 192 x - 1600 = o, giving your answers correct to two
decimal places. [2]
(d) Find, correct to whole number, the number oflitres of paraffin he bought. [l]

Fig. 1 Fig. 2 Fig. 3

Figure Total number of Total number oflines Number of small


dots joining the dots right-angled triangles
1 4 5 2
2 9 16 8
3 16 33 18
4 u v w

n x z y

(a) (i) Draw the next expected pattern for Fig.4. [1]
(ii) Find the values of u, v and w. [3]
(b) Find the number of small right-angled triangles in Fig.5. [l]
(c) Find the total number of dots needed to form Fig.IO. [I]
(d) Express x, y and z in terms of n respectively. [3]

CGS Prelim Exam 2011 4016/02

5
6

6 In the diagram, TA and TB are tangents to the circle at the points A and B respectively.
TBS is a straight line. The straight lines AB and QP intersect at Y. LQAB = 27°,
LPBT = LPQB = 40° and LATB = 56°.

~~~==-~~.C::::::..~~~~~~~~-'-~~r

B
Calculate and state the reasons clearly [1]
(a) L PAY, [l]
(b) L QBP, [1]
(c) L TAP, [2]
(d) LAYP, [2]
(e) LACQ. [2]

7 (a) Mr Fong left behind $252 000 to his wife and three sons. His wife got 35% of the
money and the rest was to be shared among his children, Ann, May and Chris, such
that Ann got two fifth as much as Chris and Chris got thrice as much as May.
How much did Chris get? (3]
(b) May bought a book on-line for S$52.20 which included a Value Added Tax(VA1)
of7.5%. The exchange rate then was £1 = S$2.02, calculate the VAT paid in
Pounds. [2]
(c) Ann bought a multi-function colour printer by paying zero downpayment and monthly
instalments of $177. 75 over 24 months. If the cash price of the colour printer was
$3950, find the rate of the interest per annum. [3]
(d) Chris puts all his money in fixed deposit with a bank that offered a compound interest
of 2.4% per annum compounded half yearly. Calculate the amount he would have at
the end of three years. [2)

CGS Prelim Exam 2011 4016/02


7

E
E. F and Gare points on level ground as shown in the diagram. EG = I 02.5 m, FG = 80 m,
LEFG = 112° and bearing of F from Eis 312°. A vertical clock tower GT stands at G, a man
walks aJong EH. Calculate
(a) (i) the bearing of F from G, [2]
(ii) the bearing of E from G. [4]
(b) Given further that angle of depression of F from Tis 35°, find
(i) the height of the clock tower, [I]
(ii) the greatest angle of elevation of the top of the clock tower from the man's view
point and how far he has walked from his starting point E to this view point. (3]

9 (a) c

In the above figure, the sector CAB has centre C and radius 8 cm. CD bisects LA CB

and 0 is the midpoint of CD. An arc, centre 0, LEOF = 2tr and radius 4 cm is drawn
5
to meet CA and CB at E and F respectively.

(i) Find in terms of 7r

CGS Prehm Exam 2011 4016/02


8

(a) the angleACB, [l]


(b) the length ofarcADB, [l]
(c) the area of the sector CAB. [ l]

(ii) Find the area of the shaded region ADBFE, correct to 2 significant figures. [3]
Q
(b)

8m

Sm B

In the diagram, rectangle ABCD is on horizontal ground. P is vertically above A


while Q is vertically above C. Given that AB= 5 m, LPBQ = 73° , AB= AP and
BC= 6 m, find
(i) the length of PQ, (4]
(ii) the angle AQC. [2]

10

l.6m

A container which is made by a square pyramid of height 1.2 m and a cylinder of radius

2 m and height 1.6 m is shown in the diagram above. The vertices of the square base of

the pyramid lie on the circumference of the circle with centre 0.

CGS Prelim Exam 201 I 4016/02


9

(a) Prove that the length of the square base of the pyramid is 2.J2 m. [2]

(b) Calculate the total volume of the container. [3]

(c) Calculate the total surface area of the container, correct to one decimal place. (4]

11 Answer the whole of this question on a sheet of graph paper.

The variables x and y are connected by the equation

2 3
y=x +--5.
x
The table below shows some corresponding values of x and y. The values of y are given correct

to one decimal place.

x
y
I o.5
1.3
1
-1
1.5
k
2
0.5
2.5
2.5
3
5
4
11.5
5
20.6
6
31.5

(a) Calculate the value of k. [l]


(b) Using a scale of 2 cm to 1 unit, draw a horizontal x-axis for 0 $ x $ 6.

Using a scale of2 cm to 5 units, draw a vertical y-axis for -10 $ y $ 40.

On your axes, plot the points given in the table and join them with a smooth curve. [3]
(c) Find the x-coordinate of a point on the curve with a gradient 5. [2]
( d) Showing your method clearly, use your graph to find the value of x in the range 2 $ x $ 6
for [1]
3
2
x +--30 =o.
x

(e) On the same axes, draw the graph of the straight line y = ~x - 4. [I]
(f) (i) State the x-coordinate of the points at which the two graphs intersect. [2]

(ii) Find the equation, in the form 2x3 + ax2 + bx + c =-0, which is satisfied by the values

of x found in part (f)(i). [1]

End of Paper

CGS Prelim Exam 20 I I 4016/02


2

?0 11 S4 EM Prelim P2 Answer 5 (a)(ii) u = 25, v = 56, w = 32


3 (b) 50
lj a) 6a3 b 2 (b) = 3qr
p 8q) +,) (c) 121
2
(d) x = (n+l)2 y = 2n2 z =3n + 2n
(c) x = 2 + ..fi (d) 5y
4 x 6(a) 40° (b) 113° (c) 22°

2 (i) (d) 49° (e) 89°


Difft 2 3 5 7 8
renee 7(a) $94500 (b) £ 1.80 (c) r = 4 °/o
2 - 1 3 5 6
(d) $ 101511.42
3 1 - 2 4 5
5 3 2 - 2 3 8(a)(i) 064° (ii) 085.6°
7 5 4 2 - 1
(b)(i) 56.0 m (ii) 70.7 m
8 6 5 3 1 - 3 2
9(i)(a) 1f (b) 1- n- (c) 6-n-
(ii) ~ (b) 2- (c) ~ s 5 5
5 10 10
(ii) 0.65 m
3
(b)(i) 10.4 m (ii) 44.3°
Times, x (min) Frequency,/

130<xS135 3 lO(a) 2J2. m (b) 23.3 m 2 (c) 47.7 m


2

135 <x S 140 Il


140 <x $145 ll(a) k = - 0.75 m
I(
(c) x=2.7
145 <x $150 1t (d) x =S.45
150 <x S 155 -, (f)(i) x =0.8 or 4.55
155 < x $160 3 (ii) 2x J - 9x2 - 2x
Mean = 144.2 mins S.D = 6.45 mins
MeaDJame<MeaDJohn, James spend less time o the
trips. However the travel time by John on the
trips are more consistent as S.DJ1111es>S.DJoho

.t(a)(i) 4 00 (ii) $(x+~ )


x 10
(c) 5.75 (d) No of litres = 70

CGS Prelim Exam 20 I I 4016/02


IName: I tndex Number: IClass: j 4A

·•·..
DUNMAN HIGH SCHOOL
Preliminary Examination
• 0
Year 4 SAP

Mathematics PAPER 1
4016/01 15 Sep 2011
2 hours

READ THESE INSTRUCTIONS FIRST


Write your name, index number and class on all the work you hand in.
Write in dark blue or black pen on both sides of the paper.
You may use a soft pencil for any diagrams or graphs.
Do not use staples, paper clips , highlighters, glue or correction fluid/tape.

Answer all the questions.


If working is needed for any question, it must be shown with the answer.
Omission of essential working will result in loss of marks.
Calculators should be used where appropriate.
If the degree of accuracy is not specified in the question, and if the answer is not exact, give the answer
to 3 significant figures. Give answers in degrees to one decimal place.
For 1t , use either your calculator value or 3.142, unless the question requires the answer in terms of 7t .

At the end of the examination, fasten all your work securely together.
The number of marks is given in brackets [ ] at the end of each question or part question.
The total number of marks for this paper is 80.

For examiner's use:


I Q2 Q3 Q4 Q5 QS Q7 Q8 Q9 Q10 Q11 Q12 Q13 Q14 Q15 Q16 Q17 Q18 Q19 Total

I
This document consists of 17 printed pages and 1 blank page.

3
[Turn over
© rJHS ?011
3

Answer all the questions.

l Evaluate

(1
~Jx0.2-12
(a) 3
to 1 decimal place.
10

(b) l.26xl0° +3.57x10°- 1, leaving your answer in standard form.

Answer (a) [1]

(b) [2]

2 (a) Factorise 2x 2 + 5x-3.

Answer (a) . . . . . . . . . . . . . . . . . . . . . . . . . . . . . . . . . [l]

(b) Hence, solve 2x 2 + 5x-3 = - (2x + 6).

Answer (b) .. . . . . . . . . . . . . . . . . . . . . . . . . . . . . . . . [2]

DHS 2011 Preliminary Sec 4 SAP Mathematics Paper 1(4016101) (Turn over

e OHS 2011
4

3
. i·i:..
Stmp IJ.y°
V- sa b x o.sa- b
3
/,
2 2

2avb

Answer (3]

4 The distribution of the teachers' age group in Dunman High School is shown in the table.
(a) Calculate the mean age of the teachers.
(b) The distribution is to be shown in a pie chart. Calculate the angle representing the age grou
of 40 ~ x < 50.

Age Group Number of


Teachers
20 s x < 30 118

30 s x < 40 46

40 s x < 50 22

50 s x < 60 10

Answer (a) ..................... years old [2

(b) 0
[l

DHS 201 l Preliminary Sec 4 SAP Mathematics Paper l (4016101)


6
©OHS 2011
5

Mr Kwong, who runs an IT shop, bought some ipads at $x each. He intends to sell each ipad at
double the cost price. However, sales has been poor and he decides to give a 20% discount.
Calculate his profit percentage for each ipad sold.

Answer . . . . . . . . . . . . . . . . . . . . . percent [3]

Alex deposited $.54,000 at LHL bank which pays an interest of 4.8% per annum. Given further that
the interest is compounded monthly, calculate the amount of money he will get after 3 years.

Answer $ ................ ''. ············ [2]

[Turn over
6

7 A map is drawn to a scale of 1 : n.


(a) The actual distance of Bedok Town from Tampines Town is 5.4 km and is represented b)
9 cm on the map. Find the value of n.
(b) Singapore covers an area of 704 square kilometres. Find, in square centimeters, the area
representing Singapore on the map, giving your answer correct to the nearest 3
significant figures.

Answer (a) [1]


2
(b) ......... ....... . .... .. ... . cm [2J

8 Solve the inequality - 3 + 4x < x < 5 - 2x.

Answer [3]

DHS 101 /Preliminary Sec .f SAP Mathematics Paper I (4016101)


7

(a) Given that 24n is a perfect cube. where n is a non-zero integer, find the smallest possible
value of n.

Answer (a) [2]

(b) Find the lowest common multiple of 15. 20 and 24.

Answer (b) [1]

JO Two similar Pepperoni pizza slices are sold in the Pizza Restaurant. The radii and the prices of the
pizza slices are as shown below, which pizza is more value for money? Justify your answer with
clear working.

Smaller pizza Larger pizza


Radius: 15 cm Radius: 20 cm
Price: $3.00 Price: $5.50

Answer [3]

DllS 201 I Preliminary Sec 4 SAP Mathematics Paper 1(4016101) [Turn over

~ OHS 2011
8

11 = 4 cm. A line is drawn from R to M such that M is f


In the diagram, PQR is a triangle where PR
midpoint of PQ where PM = 3 cm and MR = 5 cm.

(a) Explain why MPR is a right angle. R


(b) Find
(i) angle RQP,
4cm
(ii) cos RMQ, without the use of calculator.

Q M p
3cm

Answer (a) .................................................................................................. .

0
Answer (b)(i)

(b)(ii)

DHS 2011Preliminary Sec 4 SAP Mathematics Paper I (4016101)

©OHS 2011
9

The diagram below is made up of a square, a regular hexagon and part of a regular n-sided
polygon. Given that the sides of the square, regular hexagon and the regular regular n-sided
polygon are of equal length and vertex A is joined to vertex C, find
(a) the value of y,
(b) the value of n,
AB A
(c) the value of - .
AC

Answer (a) [I]

(b) [2]

(c) [3]
DHS 2011 Preliminary Sec 4 SAP Mathematics Paper 1(4016101) (Turn over

C OHS 2011
10

13 (a) On the Venn Diagram shown below, shade the set (Av B ') n C.
Answer (a) ~

(b) ~ ={x : x is an integer such that 0 $ x $ 10 }

A= { x: xis a prime number}


B = { x : x is a perfect square}
C = { x : x is a factor of 36}
(i) Find n (B').

(ii) List the element(s) of the set (An C).

(iii) Which of the following is not a subset of (B n C)?

{ }. {1}. {1, 4, 9}, {o, 9}

Answer (b)(i) . . ... . . . . . . . . . . .... . . . . . . .. . . . . . ... [1]

(b)(ii) ................................. (I]

(b)(iii) ................................. (1)

DJJS 201 /Preliminary Sec 4 SAP Mathematics Paper I (4016101)

©OHS 2011
- 11

fn the diagram. A is a point on the y-axis and BC is a horit.ontal line. Gi\·cn that the coordinates of
B is (6. I) and the gradient of AB is - f. find
la) the length of AB.
(b) the coordinates of C gi\'en that the line AC is parallel to the line J = -:!x,
(c) the perpendicular distance from C to 'he line. IB.

B (6, l)

An.nrer (a) .. ......... .................. units [3 J


(b} (.................................. ) [I)
(c) ............................. units [3]

· f/S 2011 Preliminary Sec 4 SAP Matltemntics Paper 1(4016101) [Turn over

C OHS 2011
12

15 A car and a motorcycle travelled from Bugis to Kallang and the speed-time graph of their journe
is shown below. Both vehicles left Bugis together and took the same route.

motorcycle

Speed
(m/s)
20

0 2 4
Time
(seconds)

Find
(a) the initial acceleration of the car,
(b) the distance travelled by the motorcycle in the first 4 seconds,
(c) the time tat which the motorcycle overtook the car. (You may assume that t > 2.)

Answer (a) ............................. m/s2

(b) ································ m
(c) .......................... second

DHS 2011 Preliminary Sec 4 SAP Mathematics Paper 1 (4016101)


13

Miss Fang found a bag of gold and she decided to bury it in a hole T, which is found within a
triangular field PQR as shown. It was given that Tis equidistant from PQ and QR and equidistant
from P and Q.
.t!nswer (a) and (b)

(a) By constructing suitable lines, locate and label the position of T [3]
(b) Mr Tan found the treasure and decided to bury it at a point S such that PQRS forms the
shape of a kite, where Sis outside the field. Mark out the point Son the diagram. (1]

DHS 2011 Preliminary Sec 4 SAP Mathematics Paper 1(4016101) [Turn over

0 OHS 2011
14

17 ABCD is a rectangle and Eis the midpoint of DC. The line BE intersects AC at G and the line AIJ
produced meets BE produced at F.

F
(a) Show that triangles GEC and OBA are similar.
Answer (a) In 6s GEC and GBA ................................................................. .

. . . . . . . . . . . . . . . . . . . . . . . . . . . . . . . . . . . . . . . . . . . . . . . . . . . . . . . . . . . . . . . . . . . . . . . . . . . . . . . . . . . . . . . ... [2J
(b) Name two triangles that are congruent.
Answer (b) ............... and ............ [11
(c) Find
Areaof 6GEC
(i)
Area of 6GBC '

Answer (c}(i) ................................. [I

Areaof !1FDE
(ii)
Area of A BCD '

Answer (c)(i)
DJJS 201 !Preliminary Sec 4 SAP Matltemalics Paper I (4016101)
7f

©OHS 2011
15

2
(a) (i) Express x 2 + 2x-6 in the form k(x+ a) +b.

Answer (a)(i) (1]

(ii) Hence, sketch the graph of y = x 2 + 2x-6, indicating clearly the coordinates of the
x-intercept(s) and y-intercept, if any.
Answer (aii)

(3]

DHS 1011 Preliminary Sec 4 SAP Mathematics Paper 1(4016101) [Turn over

COHS 2011
16

(b) The point (I, I) is marked on each diagram in the answer space. On these diagrams, skctd
the graph of

(i) y = x 3 +1.
(ii) Y =l.y •

Answer (b)(i) y

• (L l)

[2]

(b)(ii) y

• (l. l)

[I J)

DHS 201 !Preliminary Sec 4 SAP Mathematics Paper I (4016101)

©OHS 2011
17

D
19

In the diagram above, OAB is a triangle. C is a point on AB such that AC : CB= 1 : 3. The side OB is
produced to point D such that OB : BD = 3 : 2. It is given that OA =a and OB= b. Express, as simply as
possible, in terms of a and/or b,

(a) (i) AB,


(ii) oc'
(ill) OD.

(b) Show that CD = .!2.(~ b - ~a) .


20 3 17
---+ - 15
( c) It is given that E is the point on OA such that OE= - a . Express, as simply as
17
possible, in terms of a and b, the vector ED.
(d) Show that D, C and E are collinear.
Answer (a)(i) AB= ........................... [l]

(a)(ii) OC =.... .. .. . . ...... ... .. . . .. . [2]

(a)(iii) OD=.......................... [1]

Answer (b)
................................................................................................................... [1]

(c) ED=........................... [1]

Answer (d)
.................................................................................................................. [1]

DBS 2011 Preliminary Sec 4 SAP Mathematics Pflper 1 (4016101)

l @DHS2011
: l&l\ · 2011YEAR4 SAP
MY~yna PRELIMINARY EXAMINATION
d eport n MATHEMATICS Answer Key

Paper 1

fI (a) - 1.2 _{Pi) 7


(b) 1.617x 10° (bii) 2,3
")
(a) (2x- l)(x + 3) (bi ii) {0,9}

(b) I 14 (a) 2Jl3


x=-3or x=--
2
..,
.) II
- (b) (2, 1)
-b6
11 --
2
2a

.i (a) 31. l years old (c) 2.22 units


11
(b) 40.4° 15 (a) 10 mis
11

5 60% (b) 40m


6 $62345.83 (c) 6.83 seconds
7 (a) 60000 17 (b) Triangles BAC and DCA
(b) 1960 (ci) I
-
2
8 x<l (cii) I
-
4
9 (a) 9 18 (ai) (x+l) 2 -7
(b) 120 (aii)
'
) ll fJ•~t-6

• \
-v7-1\
I
C 1../1-1
.- ..

\_;
1-1. -1)
-6

- 10
l (bi)
(bii)
Smaller pizza
33.7°
19 (ai)
(aii)
(aiii)
b -a
1.a
4
~b
+ lb
4

--3 3
5
12 (a) 120 (c) 1b-ila
3 17 I
(b) 12
(c) 0.518
(
13 (a)

~
16
p

18(bi) (bii)

Y y = x 3 +1
y

y=;
(1, 1)

I
2
4
(b) The cumulative frequency curve illustrates the marks obtained, out of l 00, by 400
DHS students in the recent Year 4 Mathematics Alternative Assessment.
Number of students
with this mark or
less 400 I I l I 1

360

I I
320
_I, I I II
I
I I
_l_l I

I ..
280 I I! I

240

200

160

120

80

40

20 40 "50 60 70 80 90

DHS 2011Sec4 SAP Mathematics Preliminary Examinations Paper 2 (401612)

CDHS20 11
s

(i) Use the graph to find

(a) the median mark, [I]

(b) the interquartile range, [2]

(c) the seventieth (i.e. 701h) percentile [l]

(ii) Given that 65% of the students passed the test, use the graph to find the passing
mark. (2)

(iii) A candidate is awarded a "B" grade if he/she scores x marks, where


67 < x S 75. The "A" grader would have to score more than 75 marks.

Two candidates were selected from the 400 students at random. Calculate the
probability that

(a) both were awarded "B", [2]

(b) only one of them was awarded ''A". (2]


0

3 (a) The table below shows part of a utility bill received by Miss Fang in the month of Jun 2005.

Amount
Water Services by Public Utilities Board Usage Rate($) ($)

Water 21.0 CuM x 24.57

Waterborne Fee 21.0 CuM 0.30 6.30

Sanitary Appliance Fee 2 Fittings 3.00 6.00

Water Consumption Tax $24.57 30% y

Goods & Services Tax (G.S.T.) $31.94 zO/o 1.60

(i) Show thaty = 7.37. (1]

(ii) Find the values of x and z. [l]

(iii) In the year 2012, the water consumption tax is reduced to 90% of the value in
2005 but G.S.T. is increased to 7%. Calculate the amount of G.S.T. that Miss
Fang has to pay. (You can assume Miss Fang still uses the same amount of
water and retains 2 fittings of sanitary appliances and the rest of the rates
remain unchanged.) [3]

(b) ,; = {x: x is an integer such that 0 < x < 20}

A= { x: J; is an integer}

B ={x: x =2k, where k is an integer}

(i) Draw a Venn Diagram to illustrate this information. [2]

(ii) Write down n(B ' n A). (I]


7

4 The diagram below shows a circle with centre 0 and radius OB. AC is a tangent to the
circle at point B. Given that OB= 7 m, AC= (12 + 2.x) m and OA = OC = (10- 2.x) m,

(i) explain why angle ABO is a right angle, [1]

(ii) find, in terms of x, the length of AB, [1]

(iii) form an equation in x and show that it reduces to

3x2 - 52x + 15 = 0. (3)

(iv) Solve the equation 3x2 - 52x + 15 = 0, giving both answers correct to four decimal
places. (3)

(v) Hence, find the length of AC, giving your answer correct to the nearest centimetres. [2]

- - --(12+2.x)m---

0
8

5 The diagram below shows the cross-section of a regular paper weight in the shape of a
circle of radius 8 cm. A regular octagon with side AB is inscribed in the circle and another
regular octagon with side IJ is inside the circle with OJ= OJ= 6 cm.

(i) Show that angle AOB = ~ radians. [1)

(ii) Hence, calculate the area of the segmentAQB. [3]

(iii) Calculate the perimeter of the shaded region IAQBJ. [5]

(iv) The portion of the paper weight with the segment AQB as its cross-section is filled
up gold dust water. Calculate the volume of gold dust water required if the paper
weight has a height of 10 cm. [2)

Q B
.....
-,
..
),olo ~
A
-,
. ___/
..
.
L

....
........
..
.,,i.; "~
~
.. ~

Section (shaded) of the paper


weight filled with gold dust
water

Cross-sectional or plan view of


the paper weight

DHS 2011Sec4 SAP Mathematics Preliminary Examinations Paper 2 (401612)

C :>HS 2011
9

6 In the diagram below, a circle with centre 0 is shown. Given that angle BCE =109° ,
angle CEF =110°. AB = AE and BE is parallel to CD, calculate, showing your reason(s)
clearly,

(i) angle BAE, (1)

(ii) angle ABE, [I 1

(iii) angleAOE, [1]

(i'\') angle ADE, [l]

(v) angle CBE, [2]

(vi) angle DCE, and [2]

(\ii) angle CDE. [l]

F
lU

7 To improve the situation of the Mass Rapid Transport (MRT), a team was commissioned
by the new Minister for Transport to do some research.

The following data was collected from the team studying the East-West Line:

During peak hours (i.e. from 7 to I 0 am and 5 to 8 pm) of every weekday, each carriage of
the MRT carries an average of 120 adult passengers and 40 children.
During off-peak hours of every weekday, each carriage carries an average of 56 adults and
56 children.
(i) Express the above information as a 2 x 2 matrix, C. [ 1]

On Saturdays and Sundays, each carriage carries an average of 144 adults and 46 children
during the peak hours (i.e. from 10 am to 1 pm and 4 to 7 pm).
Each carriage carries an average of 60 adults and 18 children during the off-peak hours on
Saturdays and Sundays.
(ii) Represent the above information as a 2 x 2 matrix D. [I]

It is also noted that there are 8 carriages per train journey.

(iii) Evaluate M = 5 (8C) + 2 (8D). [2]


(iv) State what the elements of M represent. [2]
{v) Given that the average frequency of the MRT is 3 minutes during the peak hours (i.e.
6 hours) and that there are 180 train rides that serves the East-West Line during the
off-peak hours, express the total numbers of the train journeys for the peak and off-
peak hours as a 1 x 2 matrix, F. [2]
(vi) By using a matrix multiplication of 3 matrices, calculate the total number of MRT
commuters who commute via the East-West Line in a week. [2]
11

8 The diagram below shows the maximum number of intersections obtained from 2, 3 and 4
lines respectively.

(i) Copy and complete the 4lh and 5th rows of the table. (4)

Number of line, Maximum Maximum Maximum number


N number of line number of of regions, R
segments, E points of
intersection, P

1 1 0 2= 1 +1

2 4 - 2{1) 4=1+3
1--
2

3 9 - 3(2) 7= 1 +6
3--
2

4 6 11=1+ 10

(ii) Hence, copy and complete the klh row of the table, leaving your a nsw er in term s of
k only. p:
DHS 2011Sec4 SAP Mathematics Preliminary Examinations Paper 2 (401612) (Turn over

<ODHS2011
9 In the diagram (not drawn to scale), ABCD is a rectangular piece of land on horizontal
ground where AC and BD intersect at a point 0. It is given that AD= 200 m, CD = 100 m
and C is due North and due East of D and B respectively.
(a) Calculate

(i) OC, [2]

(ii) the bearing of C from 0, [1]

(iii) the bearing of D from B. [2]

(b) A vertical pole is erected at point 0 and a tent is set up so that ABCDP forms a
pyramid.

(i) Given that PD = 113 m, calculate the angle of depression of point D from the
top of the pole, P. [2]

(ii) Calculate, correct your answer to one decimal place, the volume of the tent,
ABCDP. [3]

[Note: The Volume of a pyramid = t x base area x height.]


p

A 200m

iflS 201 I Sec 4 SAP Mathematics Preliminary Examinations Paper 2 (401612)


13

10 Answer the whole of this question on a sheet of graph paper.

To test out a Physics' concept on Kinematics, Nicholas stood at the edge of the Sky Park,
which is at the top of Marina Bay Sands Hotel.

He threw a paper clip vertically upwards.

The height, S metres, of the paper clip above Sky Park at a time t seconds after it was
thrown is given by the formula

s = 3.6t-4.9t 2
The table below show some values of I and the corresponding values of S, correct to the
nearest whole number.

t 0 0.367 1.85 2.85 3.89 4.9 5.91 6.67

s 0 p - 10 -3 0 -60 -100 - 150 - 194

(a) Find the value of p.

(b) Using a scale of 2 cm to represent 1 second, draw a horizontal t - axis for 0 ~ t ~ 7.

Using a scale of 2cm to represent 40 metres, draw a vertical S -axis for


-200~s~10.

On your axes, plot the points given in the table and join them with a smooth curve. [

(c) Use your graph to estimate

(i) the time when the paper clip was next at the starting position of being thrown
up, r
(ii) how far below the Sky Park Jerome was if he saw the paper clip 4 seconds
after Nicholas threw it vertically upwards. (

(d) (i) By drawing a tangent, find the gradient of the graph at ( 4.9, -100). [

(ii) Explain what your answer to ( d)(i) tells you about the motion of the paper clip
at t = 4.9. (

(e) Given that the paper clip stopped moving 6.67 seconds after being throv.TI vertically
upwards from the Sky Park, explain the significance of the value S = - 194 when
t =6.67.
--- Encl of Paper ----
DHS 201 I Sec 4 SAP Mathematics Preliminary Exammations Paper 2 (401612)

0 DHS201 l
Paper 2

1 (a) ±25 3 (b)(ii) 2


9
(b) 3s+5 4 (i) Tangent is perpendicular to the radius at
2q the point of contact B
(c)(i)
(x-~)2 _5} (ii) AB= (6 + x) m or )r-(1-0---2-x)-2---4-9 m

(c)(ii) -1.14 or 6.14 (iv) 17.0399 or 0.2934


2 (a)(i) 2008 : 25.65 cents per kWh (v) 1259 cm (to nearest centimetres
2011 : 25.65 cents per kWh
(a)(ii) 2008: 3.23, 2011: 1.30 5 (ii) 2.51 cm.£
(a)(iii) There is a greater spread of the (iii) 14.9 cm
tariff rates in 2008, i.e. more
inconsistent
(b)(ia) 64 marks (iv) 10(8n-16.J2) ~ 25.1 cm3
(b)(ib) 25 marks 6 (i) = 71°
(b)(ic) 76 marks (ii) = 54.5°
(b)(ii) 59 marks (iii) =109°
(b)(iiia) 47
3325
(iv) = l oio = 54.50
(b)(iiib) 4352 (v) = 55.5°
9975
3 (a)(i) x= 1.17,z=S.01% (vi) = 15.5°
(a)(ii) $2.18 (vii) = 124.5°
(b)(i) ~ 7 (i)
120 40J or (120 56J
_JJ ( 56 56 40 56

2 6 8
I i 9 I 4
10 12
16
14 18
- -
3 5 7 11 13
15 17 19
7 (ii) 10
144 46) or ( 144 60)
( 60 18 46 18
- '"I,,-
0 -..i •1::::~ H -- , 1'"1"" "i
1 ·n
H f- rr-- -:'- ·1~ t '~ .·· -· •·rr
• I .... t· , ",...I·
t-.t- t:i..
(iii) 7104 2336) or (b)
( 3200 2528
7104 3200)
( 2336 2528
(iv) Total number ofMRT commuters
who commute via the East-West
Line in a week in categories of
age group and operating hours
(v) (6x 63 ° 180)=(120 180)
(vi) = (2163 840)
8 (i) 6, 25 , 10, 16
(ii) 2 k(k-1) (k+I)k
k , - 2- , l + - 2-
9 (a)(i) 112 m
(a)(ii) 063.4°
( a)(iii) 116.6°
(b)(i) 8.30
11 109341.5 m3
10 (a) 1
(c)(i) 0.7 to 0.8 s
( c)(ii) 60 to 68 (64 is the accurate
answer)
( d)(i) -44.42
(d)(ii) The paper clip is travelling with a
speed of 44.42 m per second in
the downwards direction. (i.e.
opposite to the initial direction of
going up)
(d)(iii) 194 mis the height of the Marina
Bay Sands building or ground
zero is 194 m below the S Park
I
Candidate Name: I Class: IIndex No:

DUNMAN SECONDARY SCHOOL

Where ...... discemmenr, discipline, daring, determinat1011


& duty bec:ome a part oflife.

PRELIMINARY EXAMINATION 2011


SECONDARY 4 EXPRESS I 5 NORMAL (ACADEMIC)
ELEMENTARY MATHEMATICS 4016/1

2H 17 AUG 2011

READ THESE INSTRUCTIONS FIRST

Write your name, index number and class in the spaces provided at the top of this page
Write in dark blue or black pen.
You may use a pencil for any diagrams or graphs.
Do not use staples, paper clips, highlighters, glue or correction fluid.

Answer all questions.


Write your answers in the spaces provided on the question paper.
Omission of essential working will result in loss of marks.
Calculators should be used where appropriate.
If the degree of accuracy is not specified in the question, and if the answer is not exact, give the answer
to three significant figures Give answers in degrees to one decimal place.
For n, use either your calculator value or 3.142, unless the question requires the answer in terms of n.

Hand in Question 1 to Question 10 and Question 11 to Question 24 separately.


The number of marks is given in brackets [ ] at the end of each question or part question
The total of the marks for this paper is 80.
CALCULATOR MODEL

This question paper consists of 16 printed pages including the cover page.

MdmC Tan [Turn over


Mathematica/ Formulae

Compound Interest

Total Amount= P(l + ~)n


100

Mensuration
Curved surface area of a cone= 7rrl .

Curved surface area of a sphere= 41l'r 2


1
Volume of a cone= -nr 2h .
3
4
Volwne of a sphere= -nr 3
3

Area of triangle ABC= .!..bcsin A


2
Arc length = r8 , where 8 is in radians .
1
Sector area= - r 28 , where 8 is in radians .
2

Trigonometry

a b c
--=--=--
sinA sinB sinC
a 2 = b2 + c2 - 2bc cos A .

Statistics

Mean = i1.
Standard Deviation= Ifx-i
- --
IJ
(I__/!_
IJ
2
)

201 JI Preliminary Exam/ Elementary Mathematics/ Sec 4£5N Paper I Dunman Secondary School

c;
3

Answer all the questions.

1. (a) 8.0lx25.0l
Use a calculator to evaluate
1.999
Write down all the figures shown on your calculator display.

Answer (a) [l]


(b) Correct your answer in part (a) to 3 significant figures.

Answer (b) [1]

2.
Consider the following numbers: ../65, -3.14, rr, o.4s, (-0.2)3, (-3)2 , ~-64, 3.!..
7
Write down the
(a) positive numbers,

Answer (a) [1]


(b) irrational numbers.

Answer (b) [1]

3. 1 4 1 ·
x+-
So lve - - =- · the fiorm
, eavmg your answer m a±.Jb , w here a, b and c are integers.
·
2x+3 6 c .

Answer x= ....... .. ... . ... ..... ...... ....... ..... ........... ...... [2]

201 JI Preliminary Exam/ Elementary Mathematics/ Sec 4ESN Paper I Dunman Secondary School
4

4. (a)
Given that ( i J= 025, find the value of k.

Answer (a) k = ... . .. ... .. . . . . . . . ... . . . . . ............ [I]

(b) Simplify 8x~ +..!.x-2 , giving your answer in positive indices.


4

Answer (b) [I]

2
5. Factorise fully 4 + 2ab - a2 - b .

Answer (2}

6. The scale of map P is 1 : p and the scale of map Q is 1 : q. If the same distance is represented
as 7.5 cm on map P and 11 .25 cm on map Q, calculate the ratio p: q.

Answer [2]

10111 Prelimmary Exam/ Elementary Mathematics/ Su 4E5N Paper I Dunman &condary School

98
5

7. Xin Ee is n years old, Yao Sen is 3 years younger than Xin Ee and Zhan Hong is the youngest
at 6 years old. Find an expression, in terms of n in its simplest form, for
(a) the (positive) difference in age between Xin Ee and Zhan Hong two years ago,

Answer (a) ............................................ years [l}


(b) the mean age of the three children in five years' time.

Answer (b) ............................................ years [l)

8. The radius of a circle is increased by 10%.


(a) Express the original circumference, as a fraction in its simplest form, of the increased
circumference.

Answer (a) [1]


(b) Find the percentage increase in the area.

Answer (b) ................................................. % [I]

20111 Pn//m/nary Exam/ EkmentaryMathemallc.sl Sec 4EJN Paper I Dunman S.condary School
6

9. In the diagram, 0 is the centre of the circle and


A, B, C, D, E and Fare points on the circle.
FD and AE pass through 0 and LABC = 130°.

(a) FindLCDE.
Ans1.ier (a) LCDE= ..................... 0 [I]
(b) Given further that CD = DE, find
(i) LAED,
Answer (b)(i) LAED= .................. 0 (JJ
(ii) LABF.
Answer (b)(ii) LABF= ................... 0 ll I

10. Written as the product of its prime factors, 270 = 2 x 33 x 5.


(a) Express 396 as the product of its prime factors.

Answer (a) 396 = ........... . .. .... ....... ..... [l)


(b) Hence find
(i) the smallest positive integer m for which 270m is a multiple of 396,

Answer (b)(i) m= .. . .. . .. ... . . . . .. . ... .. . .. . . . . [l)


(ii) the smallest positive integer n such that V396n is a whole number.

Answer (b)(ii) n= .... .. .. .. .. .. ... .. ... ..... .... [I)

20111 Prelimlllary &am/ Elementary Mathematics/ Ste 4E5N Paper I Dunman Secondary School
Candidate Name: Class: Index No:

11. Three unbiased dice are thrown. Find the probability that
(a) the three dice show different numbers,

Answer (a) (1)


(b) the sum of the three numbers shown is more than 16,

Answer (b) [I]


(c) the product of the three numbers shown is a prime number, given that the first die shows
a'two'.

Answer (c) [I]

12. It is given thats= {xis an integer: 50 ~ x ~ 100}, A= {x: 55 < x < 100} and

B= {x: ~ isapositive integer}. Find


(a) n(B),

Answer (a) [I]

(b) AnB,

Answer (b) [I]

(c) A' UB.

Answer (c) [1]

201 II Preliminary Exam/ Elementary Mathematics/ Sec 4E5N Paper I Dunman Secondary School
8

13. The force of attraction between two magnets is F newtons. This force is inversely proportionaJ
to the square of the distance, d cm, between the magnets.
(a) When the magnets are 6 cm apart, the force is 0.5 Newtons. Find the distance between
the magnets when the force is 0.72 Newtons.

Answer (a) ......................................... .......... cm [2]


(b) When the magnets are a certain distance apart, the force is 10 Newtons. What is the force
when this distance is doubled?

Answer (b) ......................................... Newtons (11

14. (a) Solve the inequality l 2x - 31 ~ 24 - 13x < 50.


Show your solution on the number line below.

Answer (a) I I (2]


-2 -1 0 2 3

(b) Hence state the least integer value of x which satisfies the inequality in part (a).

Answer (b) [1]

20111 Preliminary Exam/ Elementary Mathemat1cs/ Sec 4£5N Paper I Dunman Secondary School

102
9

15. Dunman Confectionery produces large, medium and small cakes. The table below shows the
inputs to produce each type of cak'!.
Labour (minutes) Flour (k~) Utility (units)
Large 45 0.6 5
Medium 15 0.3 3.5
Small 12 0.2 2
Labour costs $6 per hour, flour costs $2 per kg and utility costs 10 cents per unit.

3~5) ~ ).
(a) 0.75 0.6
Let A = 0.25 0.3 and B =( Given that C =AB, find C.
(
0.2 0.2 2 0.10

Answer (a) C= ............................... . [2]


(b) Explain what your answer to part (a) represents.

Answer (b)
[l]

16. The possible scores for each time a game is played are 1, 2, 5 or 10. The frequency table below
shows the scores for a certain number of games played.
1 2 5 10
Score
Frequency n+l n+3 n+4 n+3

(a) Find the value of n if the mode and mean scores are the same.

Answer (a) n= . .. . .. ... . . . ..... .. .. .. . .. ............ (2]


(b) Hence write down the median score.
Answer (b) [1]

101 JI PrelimlflQl')I Exam/ Elementary Mothernotlc.JI Sec 4ESN Poper I Dunman Secondary School
10

17. (a) s· lify 3a 10c3


unp 5c2 x7.

Answer (a) [

(b) l+x 3 2-x


Express as a single fraction in its simplest form 2
+- - - 2 •
x - x - 6 2x - 6 x + 2x

Answer (b)

20/ JI Preliminary £:.corn/ Elementary Mathematics/ Sec 4£5N Paper I Dunman Secondary School

104
11

18. The diagram below shows the speed-time graph of a journey.


Speed (m/s) .&
60 ·-----------.

- - - - - - - - - _ _ _ _ _..,. Time (sec)


40 100 130

(a) Find the speed after 25 seconds.

Answer (a) .................................................. mis [1]


(b) Find the total distance travelled.

Answer (b) ....................................................m [1)


(c) On the axes below, sketch the corresponding distance-time graph for the journey.

Answer (c)
Distance (m)

[2]
40 100 130

20111 Preliminary Exam/ Elementary Mathematics/ Sec 4E5N PaJHr I Dunman StcondlJry School
12

19. (a)
On the diagram below, sketch the graph of y =-2 . The point (I, l) is marked.
x
Answer (a)
y

(b) (i) On the diagram below, sketch the graph of y ==9-(x+l) 2 .

Answer (b)(i)
y

(ii) Write down the equation of the line of symmetry of y =9-(x+ t)2.

Answer (b)(ii)

20111 Preliminary Exam/ Elementary MathematicJ/ Sec 4E5N Paper I Dunman Secondary Schoo/

10
13

20. The points A(-5, 5), B(l, -3) and C(4, -3) are shown in the diagram below.
y
A(-5, 5)

B(l,-3) C(4,-3)

(a) Find the area of triangle ABC.

Answer (a) ...............................................units2 [1)


(b) Hence find the perpendicular distance from C to AB produced.

Answer (b) ................................................ units [2]


(c) Find the value of
(i) sin LABC,

Answer (c)(i) sin LABC= ...................... [l)


(ii) cos LABC.

Answer (c)(ii) cos LABC = . . ... ............ ..... [1]

20111 Preliminary Exam/ Elementary Mathematic.JI Sec 'E5N Paper I Dunman Secondary Schoo/
14

21. (a) The distance of the moon from the earth is 2.56x 10-3 Astronomical Units (AU). Given
that 1AU = 1.5 x l 0 11 m, find this distance in kilometres and in standard fonn.

Answer (a) ................................................... km [2]


(b) Given further that light travels at a speed of 1.8x1010 rn/min, calculate
(i) the speed of light in km/s.

Answer (b)(i) ............................................ krn/s [IJ


(ii) the time, in microseconds, light takes to travel from the moon to the earth.

Answer (b)(ii) ........................ microseconds [2]

22. On his birthday, Xin Tong put some money into an empty money box. Each week after that, he
puts a fixed amount of money into the box and does not take any money out. The following
table shows how much money is in the box after a certain period of time.
No. of weeks after the birthday 3 4 5 ... a . .. b . ..
Total amount of money($) 22 25 28 ... 58 ... c ...

(a) Find the value of a.

Answer (a) a= . ......... ... ... ........ .............. [lJ


(b) How much money did Xin Tong put into the box on his birthday?

Answer (b) $...................................................... [I)


(c) Express c in terms of b.

Answer (c) (l]


(d) How many weeks after his birthday does Xin Tong take to save at least $150?

Answer (d) .............................................. weeks (2]

201 JI Prtlimlnary Exam/ Elemen1ary Mathtma/lcsl Sec 4E5N Poper I Dunman &condary School

108
15

23. In the diagram below, QR= 7.5 cm, AB= 5 cm and QR II AB.
R

7.5 p

(a) Show that triangles ABC and RQC are similar.

Answer (a) In .6.s ABC and RQC .......................................................................................... ..

[2]
(b) Find
(i) Area of .6.ABC : Area of .6.RQC,

Answer (b)(i) ..... .. ............. .. (1]


(ii) Area of .6.ABC : Area of .6.RBC,

Answer (b)(ii) ..................... . [1]


(iii) Area of .6.A QC : Area of .6.RBC,

Answer (b)(iii) ... .. . .. .. .. ..... ..... : . .. . .. .. .. .. ..... [1]


(c) Dis a point on BP such that BD: DP= 3: 2. If the area oftrapeziumABRQ is 125 cm2 ,
find the area of triangle ADP.

Answer (c) .................................................. cm2 [2]

201 II Prelim111ary Exam/ Elementary Malhemallcsl &c 4E5N Paper 1 Dunman &condarySchool
16

24. (a) Draw a horizontal line segment AB 10 cm !ong in the answer space below. r11
(b) Construct the triangle ABC in vvhich LBAC = 50° and AC= 9 cm. [I]
(c) Construct the perpendicular bisector of AC. [I]
(d) Construct the bisector of L4CB. [I I

Answer (a), (b), (c) and (d)

(e) These two bisectors meet at M. Complete the statement below.

Answer (e) The point Mis equidistant from the points .................. and ................. .
and equidistant from the lines .................. and . . . . . .. . . . . . . . . . . . . . [I]

End of Paper

20111 Preliminary Exam/ Elementary.• Mathema11csl Sec 4E5N Poper I Dunman Secondary School
DUNMAN SECONDARY SCHOOL
PRELIMINARY EXAMINATION 2011
SECONDARY 4 EXPRESS I 5 NORMAL (ACADEMIC)
ELEMENTARY MATHEMATICS PAPER 1
Marking Scheme

On l'art Answers Mark


l (a) 10.0 107521 Bl
(b) 10.0 Al ft
2 (a) • • 2 I
./65' tr, 0.45' (-3)' 3-
7 Bl
(b) Bl
./65' tr
3 U + llx+6=0 Ml
-ll±.fi3
X= Al
4
4 (a) ,.,
i -3k= i-2 => k = .::
3 Bl
(b) 8 4x 2
32
-6x - = - Bl
x 1 x4
5 4 - (cl - 2ab + b") = 4 - (a - bt Ml
= (2 + (a-b)][2 - (a - b)]
= (2 + a - b)(2- a+ b) Al
6 7.5p = l l.2Sq Ml
p 11.25 3
- =--=-
q 7.5 2
p:q=3:2 Al
7 (a) (n -2)-(6-2) = n- 6 Bl
(b) (n+S)+(n-3+5)+{6+5) 2n+18
3 3 Bl
8 (a) 2nr 10
=- Bl
2n {1. lr) 11
(b) 2
n (1. lr ) - 7rr2
xl00%= 21% Bl
trr 2
9 (a) LCDE = LCDA +LADE= 50° + 90° = 140° Bl
(b)(i) LAED = LAEC + LCED = 50° + 20° = 70° Bl ft
(b)(ii) LABF= 40° + 2 = 20° Bl ft
10 (a) 21. x 31. x 11 Bl
(b)(i) 5940 + 270 = 22 Bl ft
(b)(ii) 2 x 3 x 11 2 = 726 Bl ft
11 (a) 6 5 4 5 .
- x - x - = - (or equivalent)
6 6 6 9 Bl
(b) 1 1 l 1 .
4x-·x-x- =-(or equivalent) Bl
6 6 6 54
(c) 1
tx.!.x.!.=- (orequivalent)
6 6 36 Bl
12 (a) 3 Bl
(b) {64. 81} Bl
(c) {50,51,52,53,54,55,64,81, 100} Bl
13 (a) k
0.5=-2 =>k=18 Ml
6
18
0.72 = -2 => d =5 cm Al
d
(b) 2.5 Newtons Bl
14 (a) 12.x - 31 ~ 24 - 13x => x ~ 2.2 and 24 - 13x < 50 => x > -2 Ml
Number line: from hollow dot at -2 to solid dot at 2.2 Al
(b) -1 Bl
15 (a)
*3 correct: 2 marks
[62]2.45 2 correct: 1 mark *B2
l.8 0-1 correct: 0 mark
(b) The total costs of producing a large-sized, medium-sized and
small-sized cake are $6.20, $2.45 and $1.80 respectively. Bl ft
(or equivalent)
16 (a) l (n+1)+2(n+3)+5(n+4)+10(n+3) =
5 Ml
n+1+n+3+n+4+n+3
n= l
Al
-
(b) 5 Bl
17 (a) 6c
- Bl
a
(b) l +x 3 2-x
+ Ml
(x-3)(x+2) 2(x-3) x(x + 2)
= 2x(l +x)+3x(x+ 2) -2(x-3)(2-x)
2x(x-3)(x+ 2) Ml
7x2 -2x+12
=
2x ( x - 3)( x + 2) Al
18 (a) speed 60
--=-=>speed= 37.5 mis Bl
25 40
(b) 0.5(130 + 60)(60 = 5700m Bl
(c) 1st Part: curve (increasing gradient) joining (0, 0) & (40, 1200)
2nd Part: straight line joining (40, 1200) & ( 100, 4800)
3rd Part: curve (decreasing gradient) joining (100, 4800) &
(130, 5700) *B2
*3 parts: 2 marks, 2 parts: l mark, 0-1 part: 0 mark
19 (a) ,..,,
2

•3 .z.s -z - 1.s ·l ·O S
l

0
D o. s

1 1. s z z.s )

•l

-z

-3

I *B2

*Correct shape: 1 mark


Correct position of ( 1, 1): 1 mark
(b)(i) Inverted U-shaped curve passing through (-4, 0), (-1, 9), (0, 8)
and (2, 0) *B2
*Correct shape+ 4 points: 2 marks
Correct shape + 2-3 points: 1 mark
Otherwise: 0 mark
(b)(ii) x =-1 Bl
20 (a)
_!_x 3x8=12 units2
2 Bl
(b)
~x~(-5-1) 2 +(5+3)2 x distance =12 Ml ft
distance= 2.4 units Al
(c)(i) 0.8 (or equivalent) Bl
(c)(ii) -0.6 (or equivalent) Bl
2.56x10-3 x 1.5x1011 = 384000000 m
21 (a)
= 3.84xl0s km
Ml
;\.l allow B2 I
(b)(i) l.8x 107 km
= 300000 krn/s or 3 x I 0 5 microseconds Bl
60 sec
(b)(ii) 3.84x 10s
300000
= 1.28 sec
6
Ml ft
Al
allow B2 I
= 1280000 microseconds or 1.28 x l0
22 (a) 58-22
+3=15
3 Bl
(b) 22 - 3(3) = $13 Bl
(c) c=l3+3b Bl
(d) 2
150=13+3x~x=45-
3 Ml ft
46 weeks Al
23 (a) LABC ~ LRQC (alt. Ls, AB II QR) } •any two:
L.BAC = L.QRC (alt. Ls, AB II QR) I mark each
L.ACB = L.RCQ (vert. opp. Ls) *B2
(b)(i) Area of MBC : Area of t::.RQC = (AB : R0)2 = 4 : 9 Bl
(b)(ii) Area of MBC: Area of l::.RBC =AC: RC= 2 : 3 Bl
(b)(iii) Area of MQC: Area of MBC= QC: BC= 3: 2
Area of M C : Area of AR.BC = 1 : 1 Bl
(c)
Area of MBP =(AB
Area of D.QRP QR;
y=.i ~
9
Area of MBP = 100 cm2 Ml

-MDP
- of
-Area - - =DP
- ~
" "DP -_ 40 cm2
Areao f LV1 Al
Area of MBP BP
24 (a)
c
(b)
(c)
(d) ,.

I *Construction lines must be clearly shown.


*B4
b A, C,AC,BC Bl
ICandidate Name: I Class: IIndex No:
DUNMAN SECONDARY SCHOOL
Where ...... discemment, d1scipli11e, dari11g. de1em1inat1011
& duty become a part nf life.

PRELIMINARY EXAMINATION 2011

SEC 4 EXPRESS/ 5 NORMAL (ACADEMIC)

MATHEMATICS 4016 I 2

2.5 H 11 AUGUST 2011


READ THESE INSTRUCTIONS FIRST

Wnte your name, index number and class on all the work you hand in.
Write in dark blue or black pen on the writing papers provided.
You may use a soft pencil for any diagrams or graphs.
Do not use staples, paper clips, highlighters, glue or correction fluid.

Answer all the ques1ions ..


If working is needed for any question, it must be shown with the answer.
Omission of essential working will result in loss of marks.
The use of a scientific calculator is expected, where appropriate.
If the degree of accuracy is not specified in the ques1ion, and if the answer is not exact, give the answer
to 3 significant figures. Give answers in degrees to I decimal place.
For 7t, use either your calculator value or 3· 142, unless the question requires the answer in terms
of Jt.

The number of marks is given in brackets [ Jat the end of each question or part question.
The total number of marks for this paper is I 00.

This question paper consists of!! printed pages including the cover page

MrD.Wong

Ms E.Thong [Tum over


2

Mathematical Formulae

Compound Interest

Total Amount== P(l + _r_)n . 100

Mensuration
Curved surface area of a cone = 1t rl .

2
Curved surface area of a sphere = 41t r

1 2
Volume of a cone= -7t r h
3
4 3
Volume of a sphere = -1t r
3
Area of triangle ABC= _!_be sin A
2
Arc length :::: re .where e is in radians .
1
Sector area= - r2e , where 0 is in radians.
2

Trigonometry

a b c
--=--=--
sinA sinB sinC
2
a 2 :::: b2 + c - 2bc cos A .

Statistics

"i.fx
Mean=
'L!

Standard Deviation = Ifx


"i.f
2
-("i.fx) "i.f
2
3

l. N
B

290m

c
A, B and C are three markers on a school field. Vanessa walks from point A on a bearing of 065°

to a point B which is 21 Om away. She then walks from point B on a bearing of 212° towards

point C which is 290m away.

(a) Calculate

(i) the bearing of B from C, [I]


(ii) the distance AC, [3]

(iii) the area of MBC, [2]


(iv) the distance BM along the path BA extended such that CM is the shortest [3]
distance from C to AB,

(b) Vanessa erected a flagpole P that stands vertically at the marker B. Given that the angle

of depression from the top of the flagpole to marker C is 35°,

(i) find the height of the flagpole P, [1]

(ii) find the angle of elevation of the top of the flagpole P from the point Mon AB. [1]

t (a) It is given that 3pr = 2r+q. [2}

Express r in terms of p and q.

(b) Solve the equation ~


x- l
- -
1 6
- - - -
x+l - x 2 -l.
[3]

(c) Seven men can paint a bridge in 12 days.

(i) How long would it take 3 men? [2]

(ii) Find the number of days it will require for 6 men to paint 4 bridges. [l]

20JJ/ Preliminary E.xoml Mathematics/ Ste 4EISN Poper 2 Dunman Secondary School
4

(d) Given that£ = {x: xis a positive integer such that 0~x~10},
P={x:x2 <30},
Q =(x: xis a prime number}.
(i) Draw a Venn diagram showing E , P and Q and place each of the members in [2]

the appropriate part of the diagram.

(ii) [ 1]
Find n{PnQ).

(iii) [ 1)
Find n{PUQ').

3. Kenneth and Samuel visited a car showroom together and each decided to get a car of the same

model. The list price of the car was $85,000.

(a) Kenneth paid a cash deposit of 20% and took a loan on the balance at a flat rate of 1.5% [3]

per annum. He paid the loan in equal monthly instalments over 5 years. Calculate his

monthly installment.

(b) Samuel was given a VIP discount of 0.5%. He then traded in his old car and paid the [2]

balance of $50,075 in cash. How much did he trade his old car for?

(c) How much more did Kenneth pay for the car than Samuel? Express your answer as a [2]

percentage of the list price

(d) After buying the car, Kenneth sent his car to a workshop in Malaysia for servicing. The

exchange rate between Singapore dollars and Malaysia ringgit was S$1 =RM $2.39.

(i) The total cost of servicing was RM $294, including a 5% government tax. [l]
How much did Kenneth pay in ringgit for the servicing?

(ii) Given that Kenneth changed S$200 initially, how much Singapore dollars did (2)

he have left?
5

4. Two taps A and Brun water at different speed. Tap A runs water atx litres per minute. Tap Bruns

water at a rate of 5 litres per minute faster than tap A. A rectangular tank with dimensions of 300

cm by 250 cm by 120 cm is to be filled with water. It takes 5 hours longer to fill the tank with

water using tap A as compared to using tap B.

(a) Find the volume of the tank in litres. [ 1]

(b) Write down an expression, in terms of x, the time taken to fill the tank by using

(i) Tap A, [1]

(ii) Tap B. [1]

(c) Form an equation in x and show that it reduces to x 2 + 5x- l 50 = 0 . [3]

(d) Solve the equation. [2]

(e) Hence find the time taken, in hours, to fill the rectangular tank if both taps A and B [2]

are turned on at the same time.

5.

A Q

The diagram shows two regular octagons that meet at points A and B.

(a) Find the size of each interior angle of the octagons. [1]
(b) Find LPAQ. (2]

(c) Show that AB: AC= 1.85 (3]

(d) Hence or otherwise, find the area of the smaller octagon if the area of the bigger [2]

octagon is 36.2 cm2 .

20111 Prd1mmary Exam/ Ma1hrmo1icsl Src 4EISN Paper 2 Dunman ~condary School
6

(a) C is the point (-8 , 5). The point D is such that DC= ( ~).
(i) Calculate I DC I. [1)

(ii) Find the coordinates of D. [2]

(b) In the diagram, F and Gare points on the lines AE and BE respectively. The lines BE

and CF intersect at D and AB is parallel to FC. It is also given that AB : FC =4 : 9


and EF: EA = BG : BE= 1 : 4, AB = 2q and AF= 3p.

(i) Express and simplify each of the following vectors in terms of p and q.

(a) [1]
EB
(b) [1]
AC

(c) (l]
EG

(d) (1)
AG

(ii) What can be said about the points A, G and Cl [ 1]

20111 Pnliminary Exam/ Mothtmatksl Ste 4EJ5N Poper 2 Dunmon Stcondory School
G

7. A crane stands on level ground.

It may be represented by a vertical tower AE

of height 6m and a jib AB of length l 2m. A

vertical cable hangs from B and 1s attached to


N
a load on the ground at the point D.

The jib is inclined at an angle of 20° to the


6
horizontal line AC.

(a) Calculate BD. E 5 F D [3]

(b) The load is lifted from D as the jib is rotated in a vertical plane about A. When the jib is in

the position AG, the load is lowered to the point Fon the ground, vertically below G. The

line AC cuts GF at N and EF = Sm. Calculate

(i) GN, (2]

(ii) the angle through which the jib has rotated. (2)

8. Answer the whole of this question on a sheet of graph paper.


The length of time taken by 80 drivers to complete a journey is given in the table below.
Time 60 < t:::; 70 70 < t:::; 80 80 < t ::;90 90<t:::;100 100<1:::;110 110<t:::;120
(t minutes)
Number of 4 10 14 20 24 8
drivers
(a) Using a scale of 2 cm to represent 10 minutes, draw a horizontal axis for times between 60 [3]

minutes and 120 minutes. Choose a suitable scale for the vertical axis and draw a histogram

to represent the information in the table.

(b) In which interval does the median of the distribution lie? (I]

(c) Calculate an estimate of the mean time taken to complete the journey. [2]

(d) Two drivers are chosen at random. Expressing each answer as a fraction in its lowest terms,

calculate the probability that

(i) both took more than J10 minutes for the journey, (2)

(ii) one took 80 minutes or less for the journey and the other took more than 110 minutes. (2]

20lll Preliminary Exam/ Mathematics/ Sec 4E/5N Paper 2 Du11man Secanllary St'hool
8

9.
I
I
I
I

A---------E\J-------;
I ,'
C
I ,'
I ,,,"'"' r
!.-'
0

E D
Diagram I Diagram II
Diagram I shows a barn and Diagram II shows the cross - section of its end.
A farmer needs to order a new roof for his barn. The roof is represented by ABC, the arc of a
circle of radius r, centre 0. ABCDE is a rectangle.
The farmer measures AC, CD, BF and the length of the barn.
(a) Given that AC= 8m and BF= 2m,
(i) write down an expression, in terms of r, for the length of OF, (1)
(ii) show that the radius, r, of the circle is 5 metres. [2]
(b) Show that angle AOC is approximately 1.85 radians. (2)
(c) Given that the length of the barn is 12 metres, calculate the curved area of the roof [3]
(shaded in Diagram I).
(d) Given also that CD= 7 metres, calculate the volume of the barn. [4]

10. Answer the whole of this question on a sheet of graph paper.


A particle was projected directly up a slope. Its distance, d metres, from the bottom of the slope,
t seconds after it was projected, is given in the table below.
t 0 0.5 1 2 3 4 5 6 6.5 7
d 0 1.58 3.02 5.48 7.38 8.72 9.5 9.72 9.72 9.72
(a) Using a horizontal scale of 2 cm to represent 1 second, and a vertical scale of 2 cm to [3]
represent 1 metre, draw a graph of d against t.
(b) Use your graph to find the distance of the particle from the bottom of the slope when [1]
t = 2.5.
(c) What happened to the particle after approximately 6 seconds? [1]
(d) (i) By drawing a tangent, find the gradient of your curve when t = 4. [2]
(ii) State briefly what this gradient represents. [ l]
(e) Calculate the average speed of the particle during the first 6 seconds. [I]
(f) At the instant the particle was projected, another particle was projected down the
slope from a point 10 metres away from the bottom of the slope. This particle moved
directly down the slope at a constant speed of 2 mis.
(i) On the same axes, draw the graph which shows the position of this particle. [2]
(ii) Use your graphs to find when the particles passed each other. [ 1]
END OF PAPER

20111 Prtlimiltary Emmi MatMmDJicsl Ste 4F.15N Paptr 2 Dwnman Secondary School
122
2011 PRELIMINARY EXAMINATION
SECONDARY 4 EXPRESS I 5 NORMAL (ACADEMIC)
MATHEMATICS PAPER 2 Marking Scheme

On Part Answers Mark


1 (ai) Bearing of B from C = 032° Bl
(ii) LABC =33° Ml
AC2 = 2102 + 2902 -2(210)(290) cos 33 Ml
AC=l61m Al
(iii) Ml
_!_(210)(290)(sin 33) = 16, 600m 2
2 Al
(iv) Ml
.!.ccM )(210) = 16584.26
2 BM
-=cos33 Ml
CM= 157.945m 290
157.945 BM = 290 cos 33 = 243m
tan33 =
BM
A 1 [deduct l mark
BM =243m for truncated error]
(bi) h
tan35=-
290
h=203m Bl
(ii) 203.06
cane=
243.21
a =39.9° Bl
2 (a) 3pr-2r = q
r(3p-2) = q
Ml
r=-q-
3p-2 Al
(b) x(x+ 1)-(x-1) 6 Ml
=
(x-l)(x+ 1) (x-l)(x+ l)
2 Ml
x =5
x=±2.24 Al
(ci) Bridge needs 7x12 = 84 man-days Ml
With 3 men, bridge needs 28 days Al
(cii) 56 days Bl
(di)
I r.
t> Correct values in Bl

·()
p
I 7
sets P and Q
3 5 Correct values in Bl
E set
~ -
6 8 9 10

(dii) n(PnQ)=4 Bl
(diii) n(PUQ') =9 Bl

123 2011 Preliminary Exam 401612 Dunman Secondary School


Qn Part Answers Mark
3 (a) 80% (85000) = $68000 Ml
5(1.5)(6800)
=5100 Ml
100
6800+5 100
=$1218 Al
60
(b) 99.5% (85000) = $84575 Ml
84575 - 50075 = 534500 Al
(c) Total Kenneth paid = $90 l 00
Total Samuel paid = $84575
Diff = $5525 Ml
5525
--x100%=6.5% Al
85000
(di) $280 Bl
(dii) Total cost= S$123.01 Ml
Amount left= $77 [accept $76.99] Al
4 (a) 9000 litres Bl
(bi) 9000 Bl
- -
x
9000
(bii) - - BJ
x+5
(c) 9000 + 300 = 9000 Ml
x+5 x
9000 + 300x + 1500 9000
=---- Ml
x+5 x
300x + l 500x - 4 5000 =0
2
Ml
2
x +5x-150=0 A.G
(d) (x-IO)(x+ 15) = 0 Ml
Al
x = 10 or -15
(e) 9000 Ml
=360 min =6 hrs Al
10 + (10+5)
5 (a) 135° Bl
(b) LBAC= 22.5° LPAQ;;;; 112.5° Ml Al
(c) l
(-/\B) AB Ml
sin67.5= 2 :::>2sin67.5=- Ml
AC AC
AB = 1.84776 -1.85 Al
AC
(d) Ml
(37r = 1369
20 400
Al
Area of smaller octagon= I 0.6 cm2
6 (ai) locj =5 units Bl

~ )-(~)
(aii) 8 Ml
OD=( D = (-12, 2) Al

2011 Preliminary Exam 401612 Dunman Secondary School


Part Answers Mark
(bi) EB = -4 p + 2q Bl
- 9 Bl
AC=3p+-q
- 2-
- 3 Bl
EG=-3p+-q
- 2-
- 3 Bl
AG=p+- q
- 2-
(bii) AC=3AG Bl
Points A, G and C lie on the same strai ht line.
7 (a) sin20° =BC Ml
12
BC=4.10m Bl
BD=10.1m Al
(bi) GN = .../122 -5 2 Ml
GN=l0.9m Al
(bii) A 5 Ml
GAN=cos-1 -
12
GAN=65.4° Bl
A

Al
8 (a)

L."I l I

Correct scale Bl
Accurate plotting of points Bl
Smooth curve + label axis Bl
6.5 m (±0.1) Al
It sto movin ./It landed on a int on the slo Al
Drawing of appropriate tangent at t = 4 Bl
Gradient= 1.13 (±0.1) Al
(dii) Al
(e) 9.72 -1 Al
Average speed =-- = 1.62 ms

125 2011 Preliminary Exam 401612 Dunman Secondary School


Qn Part Answers Mark
(fi) (Refer to line in above diagram)
Correct gradient of the line Bl
Accurate drawing of the line Bl
(fii) T= 2.1 s ±0.1) Al
9 (ai) OF=r-2 m Al
(aii) 2 2 2
r =(r-2) +4 r=5 m Ml, Al
(b) Bl
FOC = sin- 1(.±) = 53°
5
... AOC= 2(FOC) = 106° Al
(c) 106 53 Ml
Arc length = rl) = (5)(-1t) =-1t = 9.25 m
180 18 Bl
curve area= (9.25)(12) = 111 m2 Al
(d) 1 1 106
Area of segment OABC =1r26=2(5)2 ( 1t)= 23.13 m2
180 Bl
Area of triangle OAC = ~(8)(3) = 12 m2 Bl

Cross sectional area of barn= 23.13 + (8)(7)- 12 = 67.13 m2 Bl


Volume= 67.13)(121=805.56=806
,. m2 Al
10 (a) ..µ..
...... ~l
~ •.:..!. ~··
fi~:- +-!-
± .....
+,. + ~~·
. ;..t+.
..
. ~
~,-

., t-· i· t ITT1
.,.1 ~~
I
. ....... -rl+
...
·r
+
. I
- +
.. -iH
rq ;- -.. '
' !- ...t .. .
~i
.j.

....
. ~ +·
++.·
d
' r H·

:t r+ ·~ ·i+ .
-ti- . -
-1- 1-t+ j.. ,..;...·.;:
.--
I
. . 't""

I
i- ~
r-1:
. U:tt
. r
... #.l. ~· t: -r·
... .'t
'

~
j: -l . +n
.......
~.J

+ H.
. ~ I,. f.+. H·
'-1-
I

'"!- i.r-H
..
-
~
t '.i f ~i:
fJfi .
Lt.,.. -, rt
- • 't "'-!
. ,...
~

. l.:tr
'
ll I I
'
a:
rn·~ ~ ·id11 ! , , ~ * ·~~!!i "'- t ~wrr· trt ~ :i4:r t-rt- ·+
'..f
' I I
w • 4 I .... '• ...

Appropriate scale on axes Bl


Correct values plotted Bl
Histoe:ram drawn Bl
(b) Median= 95 < t $ 100 Al
(c) 65 x 4+ 75x10+85x14+95 x20+105 x24+115 x 8 = 7540 Ml
7540 .
So= 94.25 nunutes Al
(di) 8 7 7 Ml
-x-=-
Rn 79 790 Al
(dii) 14 8 8 14 14 Ml
-x-+-x-=-
80 79 80 79 395 Al

2011 Preliminary Exam 401612 Dunman Secondary School 1z.£


Class Index Number

METHODIST GIRLS' SCHOOL


Founded in 1887

PRELIMINARY EXAMINATION
Secondary 4

Thursday MATHEMATICS 4016/01


4 August 2011 Paper 1 2h

Write your name, class and index number on the question paper.
Write in dark blue or black ink on both sides of the paper.
You may use a soft pencil for any diagrams or graphs.
Do not use staples, paper clips, highlighters, glue or correction fluid.

INFORMATION FOR CANDIDATES

Answer all questions.


If working is needed for any question, it must be shown with the answer.
Omission of essential working will result in loss of marks.
Calculators should be used where appropriate.
If the degree of accuracy is not specified in the question, and if the answer is not exact, give the
answer to three significant figures. Give your answer in degrees to one decimal place.
For n, use either your calculator value or 3.142, unless the question requires the answer in
terms of 1t.
The number of marks is given in brackets [ ] at the end of each question or part question.
The total number of marks for this paper is 80.

173
Page 2 of 19 F"'
...Exam1iwr'1
US!e

Mathematical Formulae

Compound Interest
Total amount = Pr.~ + _r_)n
100

Mensuration
Curved surface area of a cone = 1trl

Surface area of a sphere = 4 nr


2

Volume of a cone = .!. nr 2 h


3

Volume of a sphere = ~ nr3


3

Area of a triangle = _!_ absin C


2

Arc length = rB, where Bis in radians

Sector area = ..!_ r 2 0 , where Bis in radians


2

Trigonometry
a b c
--=--=--
sin A sin B sin C

a 2 =b 2 +c 2 -2bccos A

Statistics
Mean= L ft
'Lf

Standard deviation = l:Jx 2-(L:1xJ2


2.J L I

174
Methodist Girls' School Mathematics Paper I Sec 4 Preliminary Exam 2011
Page 3of19 For
l!:::tamtner ·s
Use

Answer a ll the questions

1 Express 2x-3 - --
3 . Ie 1ractlon
as a smg " . m . its . Iest fiorm.
. sunp
2x 2 +7x-15 x+S .

Answer .................................. [2]

123456
2 Evaluate , giving your answer correct to 1 significant figure.
4
19.67
2
-( 2%)

Answer .................................. [2]

3 Simplify the following

.)
(I
~ ,
Ieavmg
. your answer m
. positive
'_ . .md.ices,
2
( 3c) x4c•

Answer (i) . . . . . . . . . . . . .. . . . . . . . . ....... f I]


(ii) ................. .......... .... r11
li5
Methodist Girls' School Malhemaucs Paper I Sec 4 Preliminary Exam 201 l
Page 4 of 19 For
C .T:amtntr ·
Use

4 (i) Find the sum of all prime numbers less than 10.

(ii) In a shop, the ratio of the number of televisions sets to MP3 players is 8:3. If there
are 35 more television sets than MP3 players in the shop, calculate the number of
MP3 players in the shop.

Answer (i) . . . . . . . . . . . . . . . . . . . . . . . . . . . . . . . [1]

(ii) ................ MP3 players [ 1]

5 If the exchange rate is S$1 = US$0.815,

(i) calculate the cost of US$ 3.3 trillion in Singapore dollars, leaving your answer in
standard form and to 3 significant figures.
(ii) Find the cost, in Singapore dollars, of an iPad 2 which is sold at US$499, giving
your answer correct to the nearest S$ l 0.

1\ nswcr (i) S$ .. .. .. . .. ....... . .. .... . .... [I]

(ii)S$ .. ......... . ..... . ......... [1] 11

Methodist Girls' School Mathematics Paper I Sec 4 Preliminary Exam 20 11


Page 5of19 For
£xamlnu 's
Us~

6 The capacity and height of the following two containers are the same. Wine is added at a
constant rate into each container until they are full. The total time to fill each container is
8 seconds. Sketch the graph for each of the containers.

(i)

Height (cm)
,,

cm

,,
,'

0 2 4 6 8
Time (sec)
[I)

Height (cm)
(ii)
2y

ycm

ycm

0 2 4 6 8
2xcm Time (sec)

[l]

1
Methodist Girls' School Mathematics Paper I Sec 4 Preliminary Exam 2011
Page 6of19 For
CXQmlMT
Ust

7 A train started its journey from Singapore at 15 26 and arrived at Gemas station,
Malaysia at 20 14 on the same day.

(i) What was the time taken for the journey from Singapore to Gemas station?

(ii) Another train leaves Gemas station for Singapore at 22 18 on Wednesday. If


the distance between the two stations is 220 km and th~ average speed of the
passenger train is 80 km/h, find the arrival time of the train at Singapore.

Answer (i) ............h ............. min (1]

(ii) .............................. (2]


8 In the diagram, ABCDE is a pentagon in which LBCD = LCDE = 84° and
LCBA = LBAE = LAED. [Diagram is not dra"vn to scale.]
(i) Find LBAE.
(ii) Given that LCBD = 5LBDC, find LCBD.

Answer (i) LBAE = ....... . .... ....... [1]


0

(ii) L.CBD =............. ..... [2] 178

Methodist Girls' School Mathematics Paper I Sec 4 Preliminary Exam 2011


Page 7 of 19 For
EXJJmiMr 's
Use

9 The table below shows the tax rates for resident individuals for year 2007 to 2011.
Chargeable Income Rate Tax (S)
First $20,000 0 0
Next $10,000 2 200
First $30,000 - 200
Next $10,000 3.50 350
First $40,000 - 550
Next $40,000 7 2 800
First $80,000 - 3 350
Next $40,000 11.5 4 600
First $120,000 - 7 950
Next $ 40,000 15 6 000

Use the above table to answes the following


(i) Mr Tan's chargeable income in 2010 was $100 000. What was his tax payable?

(ii) Mary paid an income tax of $855 in 201 1. What was her chargeable income correct
to the nearest dollar?

Answer (i) $ ............................ fl]

(ii) $ . . . . . . . . . . . . . . . . . . . . . . . . .. . . [2]

Methodist Girls' School Mathematics Paper I Sec 4 Preliminary Exam 2011


Page 8of19

10 Given that a 2 -b2 =60 and a-b =6, find the value of

(i) a+ b,

(ii) Q2 +b2.

Answer (i) . . . . . . . . . . . . . . . . . . . . . . . . . . . . . .. [ 1]

(ii) ............................... (2]

11 (i) Expand and simplify x(2x-4y)-2(x-y"f,

(ii) Factorise {3x + 2)2 -9x -6 completely.

Answer (i) ............................... [I]

(ii) . . . . . . . . . . . . . . . . . . . . . . . . . . . . . . (2] 180


Methodist Girls' School Mathematics Paper I Sec 4 Preliminary Exam 2011
Page 9of19

12 Given that x and y are integers, and -9 .S x ~ 8 and l s; y s; 7 , find


xi
(i) the smallest possible value of - ,
y
(ii) the largest possible value of y - 2x.

Answer (i) . . . . . . . . . . . . . . . . . . . . . . . . . . . . . . . [l]

(ii) ......... . .................... [1]

13 The marks in a Mathematics test of two groups of students are presented in the following
back to back stem-and-leaf diagram.

Leaves for Group A Stem Leaves for Group B


9 7 3 3 3
431104 0
9834115 5 4 7 8
8 5 3 2 6 6 0 l 2 4 4 5
4 l 7 4 3 5 6 7
8 2 l 3

Key (Group A) Key (Group B)


l 5 means 51 marks 4 8 means 48 marks

(i) Find the median mark for group B.

(ii) Find the ratio of the number of students who scored above 50 marks in group A to
those of group B.

Answer (i) .............. .. .. .... .. mark [2]

(ii) . . . .. . .. . . . . . . . .. . . . . . . .. .. . . . [ 1]

181
Methodist Girls' School Mathematics Paper l Sec 4 Preliminary Exam 20 J I
Page 10of19 For
ba,,,lncr~
Uu
14
A

c
In the figure, 0 is the centre of the circle. DOB is the diameter of the circle,
LCEB = 46° and LEAB = 132°. Stating your reasons clearly, calculate

(i) LBOC,
(ii) LDEC,
(iii) LEED.

Answer (i) LBOC ......... .......... [1]


0

(ii) LDEC = ................. [2)


0

(iii) LEED= ................. [2]


11
Methodist Girls' School Mathematics Paper 1 Sec 4 Preliminary Exam 2011
Page 11of19 For
.Examiner's
U#

15 {i) Dwing the Great Singapore Sale, the price of a Blue-Ray DVD player was
reduced from $240 to $212. If the player was sold at its original selling price,
the shopkeeper would make a profit of25%. Find the percentage profit when
it was sold at the reduced price.
(ii) The table shows the rate of interest offered by two different banks.
NameofBank Interest Rate
POSB 2% per annum compound interest
HSBC 4% per annum simple interest

Mr Tan deposited $1500 in POSB and $4500 in HSBC. Calculate the total
amount he would have in the banks from both the deposits at the end of two
years.

Answer (1) ............ ............... .% [2]

..,
(ii)$ .. ... . . .. ........... .. ...... [2]
1 f3
Methodist Girls' School Mathematics Paper I Sec 4 Preliminary Exam 2011
Page 12of19

16 (i) A developer estimates that he needs 96 men to build a house in 14 days. If he is


asked to complete the building in 12 days, how many more men must he hire,
assuming that the men work at the same rate?

(ii) If y is inversely proportional to x, what is the percentage decrease in y when the


value of x is tripled?

Page 13of19 f
Lan
(..
17 The diagram below shows the graph of the equation y = (x + pX3-x) where pis a
constant. The graph meets the y-axis at A(0, 12) and the x-axis at B and C.

Y=(x+ pX3-x)

Find
(i) the coordinates of point B,
(ii) the maximum point ofthe~graph y = (x+ pX3-x).
(iii) A point ( w, 6) lies on the curve. What are the possible values of w?
Page 14of19

18 ABCD is a cyclic quadrilateral and the diagonals AC and BD intersect at X.

(i) Prove, stating your reasons clearly, that 6.A.BX is similar to MJCX.
c
(ii) Given that AB = 5 cm, CD = 9 cm and BX= 2 cm, calculate the length of CX.

(iii) Calculate, leaving your answer as fraction, the value of Area of 6.A.BX
Area of M>CX

Answer

(i) ......................................................... ······· ... .................................. .

... . .. .. . .. . . . . ... .. . .. . ... . ... . . ....... ......... .. ·:· .................. ................... ........ .... [2]

(ii) ... ................. ... .. ..cm [1]

(iii) .................. ..... ........ [1]


Methodist Girls' School Mathematics Paper I Sec 4 Preliminary Exam 2011
Page 15of19 For
.Examiner 's

19 In the diagram, Mis the midpoint of XZ, OX =p + 2q, OZ = 7p - 2q, ZY


where k is a constant.
- - -- = 3kq - p,
Use

(a) Express as simply as possible in terms of p and/or q,

(i) --xz,
(ii) --
XM,

(iii) --
OM .
--+
(b) Express O y in terms of p, q and k.
v

2
(c) If k = - , what can you conclude about 0, Mand Y?
3

Answer (a)(i) -
xz =.. ........ ........ .. ..[I]
--+
0
(ii) XM =....... ............. [.l]

--+
(iii) OM = ... .......... ...... [1]

--
(b) OY =................ ..... .. [I)

(c) ................................................................. . .. . ... .. ................. ..... . .. . . .

... . .. .. . . . . .. .. .. ... . . . . . . . .. . ... . . . .. . . . . . . . . . . . . ... . . . . . . . . . . .. . . . .. . . . . . .. ... . . . . . . . .. .. . . .. . . . . .. .. [2]


1 7
Methodist Girls' School Mathematics Paper I Sec 4 Preliminary Exam 2011
Page 16of19

20 The diagram shows a pyramid with a horizontal rectangular base PQRS and a vertex V
which is vertically above the comer S.
Given that PS= 10 cm, PQ = 24 cm, tan LVPS = 1.7, find
(i) the length of VS,
(ii) the numerical value of tan LSVQ,
(iii) the volume of the pyramid VPQRS.

p 24

Answer (i) ........................... cm [I]

(ii) tan LSVQ = .............. [2)

(iii) .........................cm3 [ 1]
Methodist Girls' School Mathematics Paper I Sec 4 Preliminary Exam 20 l :
Page 17of19 For
Examiner·s
Use

21 When Elizabeth leaves for school in the morning, the probability that she takes her
umbrella to school is .!_ if it is a fine weather, and ~if it is a wet weather.
5 6

(a) Given that the probability that the weather of any particular day is fine is ~,
4
complete the tree diagram in the answer space.

Goes to school
with wnbrella
Fine
Weather Goes to school
without umbrella

Goes to school
with umbrella
Wet
Weather
Goes to school
( ) without umbrella

(2]

(b) Find, as a fraction in its simplest form, the probability that in a particular morning,
(i) the weather is fine and Elizabeth does not take her umbrella to school,
(ii) Elizabeth goes to school with her umbrella.

Answer (bi) .............................. [I]

1 9 (bii) . . . . . . . . . . . . . . . . . . . . . . . . . . . . [2]
Methodist Girls' School Mathematics Paper I Sec 4 Preliminary Exam 2011
Page 18of19
0

22 The diagram shows the speed-time graph of a particle A during a period of 60 seconds.
The particle A starts from a speed of 10 mis and accelerates uniformly until it reaches a
maximum speed of v mfs. It then decelerates and reaches l 0 m/s at 36 seconds and .
continues to travel at this constant speed. The distance travelled by this particle A in th~
first 20 seconds is 280 m. Calculate
(a) the maximum speed, v, of the particle A,
(b) the retardation of the particle A during the motion.
Speed (m/s
25

Particle A
10

20 36 60 Time (s)
0

Another particle B starts from rest 20 seconds later, and accelerates uniformly until it
reaches a maximum speed of25 m/s at 36 seconds. It then continues at this maximum
0
speed.
(c) On the same axes above, draw the graph to represent the journey of particle B. [1]
Calculate
(i) the acceleration of particle B during the first 10 seconds of its journey,
(ii) the distance between particle A and particle B at 60 seconds.

Answer (a) v = ............. ......... m/s[l]

(b) ......................... m/s 2 (1]

(c)(i) ....................... m/s2 [I] I


(ii) ..... ................... m [2]

Methodist Girls' School Mathematics Paper I Sec 4 Preliminary Exam 20 l I


0

Page 19 of1 9 For


L.:iom11wr 's
Ust

23 Jannie wants to hide her school treasure in the field so that she can share her memories
with her daughter in future. In order to remember where she has hidden her treasure,
she comes up with a map. Please help Jannie draw her map in the space below.

(i) Given that the field ABCD has sides AB of 120 m and AD = 80 m which meet at
right angle. LADC = 110° and LABC = 55°, use a scale of 1cm to represent 10 m
to construct an accurate drawing of the field. [2]

(ii) Construct a perpendicular bisector of the line AD. [1]

(iii) Construct an angle bisector of angle BCD. [1]

(iv) Jannie hides her treasure in the field at the equidistant from A and D as well as
equidistant from the two sides BC and CD. Mark this point T and measure the
distance CT.

(v) There is a plan to set up in the field , a fitness comer of 120 m 2 • Calculate its area
on the map.

Answer (iv) CT =... .............cm [l]

(v) .. ... ... ............ ..cm2 [1]

191 - End ol Paper -


Methodist Girls' School Mathematics Paper I Sec 4 Preliminary Exam 2011
MATHEMATICS Paper 1PRELIM2011

No. Answers No. Answers


~
1. -2 7(i) 4 h 48 min
(x+5)
(ii) 0 I 03 on Thursday
2. 10 8(i) 124

(ii) 80
3(i) I 9(i) 5, 650
--
12c 4
(ii) 44,357

x
(ii) -
3
4(i) 17 JO(i) JO

(ii) 21 (ii) 68
S(i) 12 11 (i) -2y2
4.05xl0

(ii) 610 (ii) (3x + 2X3x - 1)

6(i) He ght {cm) 12(i) 0


h .,
(ii) 25

13(i) 57

(ii) 4:3

I • • '
0 14(i)
2 4 6 8 92
Height (cm) Time (sec) (ii)
44
z .. (iii)
42

(ii) lS(i) 10.4 or 10 2.


12

6420.60
(ii)

16
16(1)
I I l I
• 2
0 2
I

4
I

6 8 (ii)
66- or 66.7
3
Time (sec)

J
No. Answers
0
No. Answers

20
l 7(i) ( -4 I 0 ) (i) 17

(ii) (ii) 1.2.


I 17
( -- 12 _!_ )
4
2 I

(iii) 1360
(iii) -3, 2

LABO= LACD (L in the same segment)


21
18(i)
I 4 5, l
(a) -
LBXA = LCXD (vert opp Ls)
4' 5'6 6
: . !::. ABX is similar!::. DCX
(b)
3
3 (i) -
3.6or3- 5
(ii) 5
43
(ii) -120
25
(iii) -
81
"

l 9(a)

6p-4q
(i)

3p- 2q
(ii)

4p
(iii)

(b) 6p+ {3k-2)q

(c) 2 - 3-
lf k = -, 0 Y =6p = -OM=> OYparallel to OM and O
3 2
is common. 0, Y and Mare collinear.
No. Answers

22(a) 18

(b) I
-
2
Speed (m/s)
(c) Particle B
25

Particle A
10

Time (s)
0 20 36 60

(i) I~
16

(ii) 56

23

CT= 6.2 cm

A B
(iv) 62

(v) 6
Class Index Number

METHODIST GIRLS' SCHOOL


Founded m 1887

PRELIMINARY EXAMINATION
Secondary 4

Tuesday MA-TH EMATICS 4016/02


16 Aug 2011 PAPER 2 2h 30 min

READ THESE INSTRUCTIONS FIRST


~

Write your class, index 9umber and name on all the work you hand tn .
Write in dark blue or bla'ck pen on both sides of the paper.
You may use a pencil for any diagrams or graphs.
Do not use staples, paper clips , highlighters, glue or correction fluid .

Answe r a ll questions.
If wo rking is needed for any question it must be shown with the answer.
Omission of essential working will result in loss of marks.
Calculators should be used where appropriate.
If the degree of accuracy is not specified rn the question, and if the answer is not exact,
give your answer to three significant figures. Give your answer in degrees to one decim al
place.
For n, use either your calculator value or 3 142. unless the question requires the answer
in terms of it .

At the end of the examination, fasten all your work secure ly together.
The number of marks is given in brackets [ ] at the end of each question or part
question.
The total number of marks for th ts paper s 100.
Marks

This question paper consists of 11 printed pages.


i'vlathematical Formulae

Compound 111reresr
11

Total amount;::; P (1 + _,_. )


100
Mensuralion
Curved surface area of a cone = nr/
~ '
Surface area of a sphere = 4nr~
v
1
Volume ofa cone = - m· 2h
3

Volume of a sphere

Arca of triangle .-lBC I a b sm


. ('
2
Arc length = rtJ, ''lier(: 0 is in radians
Sector area= _!_,. (). \\ Lcre 0 is in ra<lians
)

Trigonomefly

a h c
- - = -- = -
sin .·I sin H sin C

Statistics
>-1··
i\leun - - ·'
) .
-1

Standard Jc,·i:11iu11 L. /r ~ _ ( 'i tx) 2

\ !.., l 'if
Page 2of10

3 2 '
. .fy x +3x +2x
1. (a) S imp11 . [2)
2x 2 -4x-6
(b) Express as a single fraction in its simplest form,

_ 5y-x
2 [2]
y+3x

(c) (i) Express x 2 -9x-2 in the form (x-a)2 +b. [1]


(ii) Hence solve the equation x 2 -9x- 2 = 0, giving your answers to two

decimal places. . [3]

2. A finn rents out small and large buses. The cost of rental of each bus is shown in the table

below.

Average Distance for n days Average Distance for n days


Cost per day
:s 200 km > 200 km
Small bus $80 No extra charge 50 cents per km
Large bus $100 No extra charge 80 cents per km

(a) A small bus was rented out for 3 days and travelled 630 km during this time.

Calculate the total cost of rental. [l]

(b) A man who rented a large bus for -I days was charged $460.40. Calculate the total

distance he travelled. [2]

(c) On one occasion, a small bus and a large bus were each rented out for 5 days. Each
travelled the same distance, which is greater than 1000 km. The total charge for the
large bus was$ I 39 .27 more than the total charge for the small bus. Calculate the
distance that the large bus travelled. [3)

(d) On another occasion, a man rented a small bus for one day and travelled 178 km. He
spent a total of $124.50 on the i·ental charge and petrol. Given that a litre of petrol
costs $I .96, calculate the; petrol consumption of the small bus, giving you r answer in
kilometres rcr litre. [3]

i\kthodist Girls' School Mathematics Pap.:r 2 Sec 4 Prelirn Exam 20 I I


n
Page 3of10

3. Carrie bought some wallets for $892.50. There were 12 wallets in poor condition. The cost
price of each wallet was $x. She sold all rhe wallets in good condition. Each was sold at $7
more than the cost price for her0 to make a profit of $253.

(a) Find, in terms of x, an expression for

(i) the number of wallets bought, (1 )

(ii) the number of wallets sold. [1]

(b) Form an equation in x and show that it simplifies to

24x2 +674x-12495 = 0. [3]

(c) Solve the equation 24x 2 + 674x- I2495 = 0. [3)

(d) Hence find the number of\:vallcts sold. [2]

4. The figure shows the outline of a badge made up of three sectors with the same centre, 0.
AOB is a straight line. AOBCF forms a semicircle. The arcs AF, EPD and BC are each of
length 1.5 cm.

\ I
\ I
\ I
I

' \ I
\ I
\ I
\ I
\I

A 0

(a) Prove that triangle AFO is congruent to triangle BCO. [3]

(b) Given OE ~ ()/J - 2 cm. calculate

(i) LH )( ·, in rndinns, [I]

(i i) the p~l'llm.:tcr of the badge. [2)

(iii) thr ru11~1i11ing area or the hm.lge with the segment EPD rcmo\'ed. [41

Methodist Girls' School Malhcmu11cs i>apcr 2 Sec 4 Prelim Exam 201 J


Page 4 of 10
0

5. A large area is to be paved with blocks each one metre square. On Day 1, three
blocks are placed in a line, as shown in the diagram. Each followi ng day the paved
region is enlarged by adding blocks to surround the previous day's region, as shown
below

After Day I Afte.: Day 2 After Day 3

The perimeter and area of the total region CO\ ered by the blocks, after the completion
of each day's work arc calculated as in tht.: table below.

After Perimeter (m) Area (m-z)

Day 1 2(1+3)= ~ lx3 =3

Day 2 2(3+5)= 16 3x5=15

Day 3 2(5+7)=24 5x7 = 35

By considering the paved regions and the patterns developing in the table, answer the
questions below.

(a) (i) Find the perimeter after Da) 6. (1]

(ii) Write an express10!). in tenns of n. in its simplest form, for the perimeter
after Day n. [l)

(iii) Give a reason why I 002 cannnt be the perimeter after a certain day. [ lJ

(b) (i) Calculate the area after Day 6 [1]

(ii) 2
Write Jown an expression, in the l(inn an -b, for the area after Day n. ll J
0

(c) (i) Find an expression, in terms or 11. in its simplest form, for the number of
blocks added after Day n. [2]

(ii) Explain clearly "hY it is po:-;..;ihk IO add 2012 boxes in a certain day. [lj

M\!lhod isl <I ills' School Mathcm..t 1cs 1'.ipc1 2 Sec 4 Prclim Exam 2011
199
Page S of 10

6. P, Q, R and Sare four points of a field on level ground with R due east of Q.

It is given that PQ = 12.8 m, QS = 25 m, QR= 27.1 m, RS= 18 m and LPSQ =26°

27.1 Ill

(a) Calculate

(i) LSQR. [2)

(ii) obtuse L'QPS, [2]

(iii) the bearing of P from Q, [2)

(iv) the arl.!a of the field bounded by PQRS, (2)

(v) the ~hortcst distarn:e from (,} to NS [2]

(b) A girl, standing al a point inside the lii..:l<.l PQRS, spotted a kite on lop of a vertical
tree al R. I he angle of elevation ol the kite was 32°. She mo\ c<l forward a distance
of I 0 mctn.:s ll)war<.ls Rand found the angle of elevation of the kite to be 48°. Find
the height 111 thr tree. [2J

Methodist Girls' School Mathemat ics Paper 2 Sec 4 Prelim Exam 201 1
Page 6of 10

7. (a) Given that£ = {girls in MethodistGirls'School}, H ={ hockey players}

S {swimmers} and N ={netball players}

Express in set notation

(i) No girls in Methodist Girls' School are hockey players. []]


(ii) All girl s in Methodist Girls' School are either swimmers or
netball players or both. [1]

(b) Given that £ = {x: xis an integer, 2~x~15}, A= {x: xis a factor of 12}

and B={x:xisamultipleof3}

(i) find n( .·1 u B). [l]


(ii) list th~ clements .of the set A'nB'. [] J

(c) A compan: manufactures toy cars and dolls.

The follo,\ing table 1s used in calculating the cost of manufacturing each toy.

Labour Wood Paint


(hours) (blocks) (Tins)
Car 6 3 3
Doll 5 4 3

(i) Rcpres~nt this info!mation in a 2 by 3 matrix, named A. [I]

Labour costs $6.50 per hour, wood costs $1 per block and paint costs $1.20 per tin.
(ii) Represent this information in a 3 by I matrix. named B. [l)

(iii) Evaluat-.· C = AB [l J
F:.. p!,1111 what the clements in matrix C represent. [ l]

(iv) Givu1 J) = ( 150 225 ). evaluate DC and explain the significance of your
. [21

.\l~thodist Girls' School Mathcm;itks Paper 2 \c1: 4 Prelim txam 20 11


11
Page 7of 10

8. The volume of mixture required to make one cookie is 15 cm3 .

Before it is baked, the mixture 1s rolled into a sphere.

(a) Calculate the radius of the sphere. [2]

After it is baked. the cookie becomes a cylinder of radius 3 cm and height 0.6 cm.

The increase in volume is due to the air trapped in the cookie.


(b) (i) Calculate the volume of air contained in the cookie. [2]

(ii) Express this volume of air as a percentage of the volume of the mixture. [1)

The cookies are then packed in a box.

·1 he cross-section of the box is a regular hexagon. contaiPing 7 cookies as shown in


Diagram I.

Diagram J Diagram II
Three of the cookil.:s arc sh0\\11 in Diagram II.

0 is the centre or 1he hexagon in Diagram l :ind of the middle cookie in Diagram II.

Q is the point 'vhcre the two cookie<; 1ouch.


P and R are the centres of these cookies

(c) (i) ~1.11 ..: till' kng1h OP. [1 l


(i i) ( ·~1k11la1c the length ol <HJ. [2]

(iii) 13) w11 ~ id<.:ring similar 1riangks. or othcrn ise. calculat<.: Ilic length of a side or
[2J

Methodist Gtrls' School M;1thcmaltl'S Paper 2 Sec --l Prelim Exam 20 I I 202
Page 8of10

~. Answer the whole of this question on a sheet of graph paper.

The variables x and y are connected by the equation y =~ x 2


(3 - x).
2

Corresponding values of x and y, corrected to I decimal place where necessary, are given
in the following table .

x -1.5 -1.3 -1 -0.5 0 0.5 1 2 2.5 3 3.3 3.5


y 5.1 3.6 p 0.4 0 0.3 1 2 1.6 0 -1.6 -3.1

(a) Calculate the value of p. (1]

(b) Using a scale of 2 cm to represent 1 unit, draw a horizontal x-axis for -1.5 s x s 3.5.

Using a scale of 2 cm to represent I unit, draw a vertical y-axis for - 4 sys 6.

Draw the graph of y=~x 2 (3-x) for -1.5sxs3.5. (3)

(c) Use your graph to write down the range of values of x for which the gradient is

positive. [1]

(d) Use your graph to find the value of x for which ~x 2 (3-x) =3.
2 (1)

(e) By drawing a tangent, find the gradient of the curve at x = 2.5 . [2]

(f) (i) On the same axes, draw the graph of the straight line x + y = 1. [I]

(ii) Hence write down the three possible solutions to the equation

1 2
- x (3 - x) = 1 - x. [I)
2

11ethodist Girls' School Mathe ma I ic.:s Paper 2 Sec 4 Prelim Exam 201 I
a "5" 7 V& JV

10. 70 rurmers took part in a cross-country race. At various times after the start, one of the
officials recorded the number ofrurmers who had finished the race. The official's results
are given in the cumulative frequency curve below.

10 20 30 40 50 60

Time (minutes)

Methodist Girls' School Mathcm::tirs Paper 2 Sec 4 Prelim Exam 20 11 20 •


Page! Oof lO

10. (a) Use the curve to estimate

(i) the median time rakcn to nm the race, [1]

(ii) the interquartile range of tl1e distribution. [2]

(iii) The following frequency relates to the same results.

Time 20:::; I <30 30 ~I <40 40 .$ ( < 50 50 .$ ( < 60


(t minutes)
frequency 5 p q 11

(a) Find the value of p and the value of q. f2]

(b) Calculate
0

(i) an estimate of the mean time required to run the race, 11]

(ii) the standard deviation. [1]

(c) Another group of 70 runners have the same median but a sm a ller
standard deviation. Describe how the cumulative frequency will differ
[ 1]

(b) A bag contains 5 ickntical ball~ numbc.:r 1, 2, 3. 4 and 5.

Two balls arc.: drawn at random. one after the other, from the bag without replacement.

(i) Draw the possibility diagram to show the outcomes of the draw. ( 1)

(ii) find. as a fraction in its simplest fom1, the probability that

( ;\) both numbers arc odd, [ 1]

(b) lhe sum of the numbers drawn is 7, f 1)


{c) lhc product or th\.." nurnlwrs dnl\\n is 4, f1 I
(d ) in knst one or the numhc.:rs drawn is a prime number. fl l

l\1cthouist Girls· School ~1:c <I Prelim Exam 20 l I


PRELIM 2011 MATHEMATICS Paper 2 Answer Key

No. Answers No. Answers


l(a) x(x + 2) S(a)(i) 48
2(x - 3) (ii) 8n
(b) (iii) 1002 is not a multiple of 8.
7x -3y
3x+ y (b)(i) 143
' (ii) 4n 2 -1
(c)(i)
(x -4.5) 2 - 22.25
(c)(i) 4(2n-1)
(ii) (ii) 2012 is a multiple of 4
-0.22 or 9.22
2(a) $255 6(a)(i) 40.2°
(ii) 121.1°
(b) 875.S km (iii) 016.9°
(tv) 305 m2
(c) 1130.9 km (v) 24.3 m

(d) 7.84 km/I (b) 14.3 m

3{a) (i} 892.50 7(a)(1) n(H)= 0 or H=¢


x I (11) SuN=i·

(ii) 392.50 -1 2 I (b)(•) 7


x i (ii) A'n B '= { 5, 7, 8, l 0, 11 , 13, 14}

(b}
( 89 ~\".SO -1.2 }x r 7)-89'2.50 = 253
(c) (1) .1=(65 43 3)3
(c) (ii) 6.50]
x= l2.75 or t =-'I0-5 B= I
6 (
1.20
{d) 58 (iii)
1--~~1--~~~~~~~~~v~~J
4(a) AO = OB (radius of circle)
(:~:~~)
OF = QC (radius of circle) The cost to manufacture 1 toy car= $45.60
I
I
The cost to manufacture 1 doll = $40. l 0
. . (equal nrcs subtc11
c.AOJ· = L.JJ( J( 1 (iv) l15862.50)
1,yqual Ls
. !::u I FO = IBCO (SAS) Total cosl of manufacturing 150 toy cars
an<l 225 dolls is$ 15862.50 .
or AF· tu· (t:qt1al arcs have
equal chord-;)
(b}(i) :. &IFO = WCO (SSS)

(ii} 0 .75

(iii) 8.5 cm

1
3 .24 cm

206
No. Answers No. Answers
8.(a) 1.53 cm I lO(a) (i) 43 minutes
fV
(b)(i) 1.96 cm (ii) 9.5 minutes

(ii) 13.1 % (iii)( a) p =18 q = 36


(b )(i) 42.6 minutes
(c)(i) 6cm (ii) 8.01

(ii) 5.20 cm (c) The curve will be steeper .


Oall 2

(iii) 9.64 cm (b)(i)


B~ 2 3 4 5

0 (1 , 2) (l , 3) (I , 4) (I, 5)

2 (2, I) (2, 3) (2, 4) (2, 5)

I 3 (3, I) (3, 2) (3. 4) (3, 5)


{-l, I) (4, 2) (4, 3) (4, 5)
l 5 (5. I) (5, 2) (5, 3) (5, 4)

(ti) (a) 3
10
(b) I
-
5
(c) I
-
10
(d)
-109

9(a) 2
""""'"' :)\•
••••• •• · '·••-••• •• ·••••y.._..,, '''"''' ••••• •• • · •·•·••••·•• · ••·•• ••• ••••••••••o• •· •··••••••• · •••••••••••••
(b)

L:
r::(. ·· ::·:: :·i:.:·: ,: :'::.: c~i · c· :i::: : ::· t:· -~·· .

.\::~yi~'Ftj~ j.~:··::1.:.:. .r:. :. :t


: : : : . : ,j - 1 ; :

T-..::·}~....:; · :':' ): ,, ,~~:·f


·~··· ..... ' . , . • "I" . • .• "! ..... ! . .. : \ ... ,,......:_
........_...__'--'-'"'--~~~~~--1..___
I ~--- : : l '.\ .

(c) 0 <x <2

(d) x = -1.2±0 05

(e) Gradient = · 1.9 ±0.3

(f) x - 1, 0.55.f0.05. 34±0.05


Class Register No.

Candidate Name ........................................................ I._____.!.____ _ ____,

TANJONG KATONG GIRLS' SCHOOL

PRELIMINARY EXAMINATION 2011


SECONDARY FOUR

4016/1 MATHEMATICS
PAPER 1

Wednesday 24 August 20 11 2 hours


READ THESE INSTRUCTIONS FIRST
Write your name, class and register number on all the work you hand in.
Write in dark blue or black pen on both sides of the paper.
You may use a pencil for any diagrams or graphs.
Do not use staples, paper clips, highlighters or correction fluid.

Answer all questions.


If working is needed for any question, it must be shown with the answer.
Omission of essential working will result in loss of marks.
Calculators should be used where appropriate.
If the degree of accuracy is not specified in the question and if the answer is not exact,
give the answer to three significant figures. Give answers in degrees to one decimal place.
For Jr, use either your calculator value or 3.142, unless the question requires the answer
in terms of tr.
At the end of the examination , fasten all your work securely together.
The number of marks is given in brackets ( ] at the end of each question or
part question.
The total of the marks for this paper is 80.

For Ex aminer's Use

Marks I 80

Setter : Miss Grace Lee, Miss Sharifah, Mrs H Pang


Markers : Mr Ang WJ, Miss Grace Lee, Mrs H Pang, Mrs Diane Ang , Mrs M Loy, Miss Serena Lee

This Question Paper con sists of .1Q... print ed pag es, includ ing this p age.

127
3
For for
!xamtncr's
Answer all the questions. Exomincr"
Use Uot

I -0.00678 . .
1 (a) Calcu Iate ~ , correct your answer to I significant figure.
0.45x _.E_
1000
1
(b) Express 5- as a percentage.
3

Answer (a) [I]

(b) .................................. % (1]

2 Mr Lim's dinner bill in ABC restaurant was $170.665, before rorn1ding off to the nearest
cent. The bill includes I 0% service charge and 7% goods and services tax (GST).
Calculate

(a) the GST charged, and

(b) the amount incurred in the bill without the taxes.

Food and Beverage $-----


10% service charge $-----
sub total $-----
7%GST $-----
Total $170.665

Answer (a) $.................................... [l]

(b) $............ ......................... [1]

TKGS Preliminary Exam 20 I I 4()/6101 Mathematics Paper 1;


4

.:,~~er's 3 Write down an expression, in tenns of n, for the nrh tenn of the sequence I
Exa1
Use 1

(a) 2, 4, 6, 8, 10 ..... .

(b) 2, 5, 10, 17, 26 ..... .

Answer (a) [1]

(b) [1]

4 A helium-neon laser emits light of wavelength, A.= 6.328 x io-s cm for a certain
frequency, /(Hz).

(a) Express this wavelength in nanometre (nm).

(b) Given that the wavelength, :>.. (in cm) of light is inversely proportional to its
frequency, calculate the new wavelength of light, in nm, emitted by the
laser when its frequency is now increased by 250%.

Answer (a) ...............................nm [I]

(b) ................................. run [2]

TKGS Preliminary Exam 2011 4016101 Mathematics Paper


5
For
amrntt's Fe
Exam
use 5 A map is drawn using a scale of l : n. U:
On this map, a length of 5 cm represents an actual distance of 2 km.

(a) Find n.

(b) Calculate the area on the map, in cm2, which represents an actual
area of 3 km2 •

Answer (a) n =. .. ... ............. ............ [l]

{b) ................................ cm2 [2]

6 At noon on a particular day, the temperature at the bottom of a hill was l o·c
and the temperature at the top of the hill was -12'C.

(a) Calculate the difference between the two temperatures.

(b) The height of the hill is 440 m.


Given that the temperature changed uniformly with height,
find the height at which the temperature was -5°C.

Answer (a) .................................. °C [l]

(b) ................................... m [2]

TKGS Preliminary Exam 2011 4016101 Mathematics Paper l


6

For for
::Xammcr's 7 (a) Factorise 1- a 2 + 2ab- b 2 • Examine
Use Us•

(b) Given that - 6 $ x < 4_!._ and -1_!._$y$4, find the largest possible integer
2 3
value of (x + y)(x - y).

Answer (a) [2]

(b) [l]

8 Solve
2
(a) (3m + 3) = 9, where m < 0.

(b) 2 _t + .:_ =15 - 2x


12 3 4

Answer (a) [l]

(b) [2]
TKGS Preliminary Exam 2011 4016101 Mathematics Paper
7

For
Ex~~~•r'$ 9 One interior angle of a n-sided polygon is 84°. Ex.ami"'
Use Use

(a) Write down the size of its exterior augle.

The rest of the interior angles of the polygon are each equal to 136°.

(b) Find n.

(c) Given that (n - a) xl 80° is the sum of its interior angles, find the value of a.

(a) [l]

(b) (1)

(c) [1]

10 (a) Simplify( 2x
3x
~ ]_,, leaving your auswer as a positive index.

(b) Solve the equation 27 2x-J = 3 2 (3")

Answer (a) [1]

(b) [2)

TKGS Preliminary Exam 2011 4016101 Mathematics Paper l:


8
I'•
Exwn
The number of siblings that students in a class have is shown in the following table. u

Number of siblings 0 t 2 3 4 5

Frequency 1 3 x 6 4 5

(a) If there are 35 students in the class, calculate the mean number of siblings that
the students have.

(b) If the mode of the above distribution is 2, state the smallest possible value of x .

(c) Given that the median is 3, write down the largest possible value of x .

Answer (a) [1]

(b) [l]

(c) [t]

TKGS Preliminary Exam 2011 4016101 Mathematics Paper


9
For 2 The Lowest Common Multiple and the Highest Common Factor for two numbers, m For
miner's Examin1
Use Use
andnare 23 x3 2 x7xll 2 and 2 2 x3xll respectively.

(a) Find the value of m and of n given that 63 is a factor of n and not of m, m > n.

(b) Hence, find the smallest integer k, such that mk is a perfect cube.

Answer (a) m = ................ n = ... ... ... ... ... .... (2]

(b) k = ... ... ... ... ... ... ... ... ... ... ... ... ... .... [1]

TKGS Preliminary Exam 201 l 4016101 Mathematics Paper 1:


10
For
t.mancr's Exa.
Use

3 A bag contains 3 green balls and 5 yellow balls.


(a) Find the probability that a ball chosen at random from the bag is green,
yellow or red.
(b) Two balls are taken from the bag at random, without replacement.
Find the probability that
(i) the two balls taken are green,
(ii) at least one ball taken is yellow.

Answer
(a) [I]

(b)(i) [1]

(b)(ii} [2]

TKGS Preliminary Exam 2011 4016101 Mathematics Paper


11

For F'
1m1ner·s Exwr
Use 4 The diagram shows a triangle ABC, with AB parallel to the x-axis. u

A B

Given that A is ( - 2, 2), C is (7, -I 0) and the equation of the line BC is y =-2x + 4;
(a) find the gradient of AC.

(b) find the coordinates of point B.

(c) A point Dis added to the diagram to form a trapezium A BCD of area 60 units2 .

Find.
(i) the x-coordinate of D and

(ii) the equation of the line that passes through D and parallel to AC.

Answer (a) [l]

(b) [I]

(c)(i) (I]

(c)(U) [2]
TKGS Preliminary Exam 201 I 4016101 Mathematics Paper
12

~~ner's 15 (a) Expand 3(5p-q)(q + 2s). E~ar


I
:.Jse l

3x-2 6-9x
(b) Simplify - - + 2
.
2x-6 x -4x+3

Answer (a) (J]

(b) (2]

TKGS Preliminary Exam 2011 4016101 Mathematics Paper


13

For
Ex•mincr's 16 In a class of 40 international students, 23 students speak English, 8 students speak
Use German and 12 students speak French. 1t is given that

e = {students m the class},


E = {students who speak English},
F = {students who speak French},
G = {students who speak German}.

(a) Based on the Venn Diagram below and given that n((Fu E) 1

) =10, find the


number of students who can speak only French or only English, but not both.

F
G

0
(b) Express in set language the following statement:

Not all students who speak French speak English.

(c) Express in word the following set language :

GcE

Answer (a) [2)

(b) [I]

(c) ......... ..... ................ ..................................................................... [I]

TKGS Preliminary Exam 2011 4016101 Mathematics Paper


14
for Fe
uminer's 17 In the diagram, three points A, Band Care shown below. E~am
Use U:

(a) Measure and write down the reflex LABC.


(b) Construct a suitable line such that the points on the line are equidistant (1)
from the lines AB and BC.
(c) The point Dis such that A, B, C and D form vertices of a rhombus. [2]
Construct a possible position of D.

Answer (a), (b) and (c):

(a) [l]

9 TKGS Preliminary Exam 2011 4016101 Mathematics Paper


15
For For
laminer's E1<nmim
Use 18 (a) The minimum turning point of a quadratic curve is (1, -4). Write down the Use

equation of the curve in the form y = ax 2 +bx+ c where a, b, c, are integers


2
and the coefficient of x is l.

Answer (a) ....................................... [2]

y
(b) Sketch the graph of 18(a) on
the grid provided.
Indicate clearly where it cuts the
axes.

[2]

TKGS Preliminary Exam 2011 4016101 Mathematics Paper l •


16
:-or
for
n1nu's 19 In the diagram below, A, B, C and Dare points on the circumference of the Examiner's
Jsc Use
circle. AEC and DEB are straight lines.

It is also given that AE = 4 cm, BC= 3 cm and AD = 9 cm.

(a) Show that MED and MJEC are similar. [2]


(b) Find the length of BE.

Answer (b) [2]

TKGS Preliminary Exam 201 J 4016101 Mathematics Paper


18
For For
xaminer's EK.amine.r's
Use 21 The diagram below shows the speed-time graph of a particle moving for the first 30 Use

seconds.
Speed (mis)
w

0 10 15 30 Time(seconds)

(a) Given that the distance the particle covered for the last 15 seconds is
75 m, find the value of w.

(b) Find the speed of the particle at 17 s.

(c) On the axis below, sketch the distance-time graph of the particle's movement.

(2]
Distance
(m)

0 15 30 Time(seconds)

( d) Calculate the average speed, in km/h , of the panic le, for the first 30 seconds.

Answer
(a) [l]

(b) (1]

(d) [I]

TKGS Preliminary Exam 201 I 4016101 Mathematics Paper


19

For 22 (a) At 'QuickPrint' shop, the cost, C, in cents, of printing postcards consists of
miner'• E><m
Use a fixed charge of $1.00 and a charge of 6 cents per card. 1

(i) Write an equation to determine the cost, in cents, for printing n cards.

(ii) Draw and label the graph of the above equation on the grid below. [1)

(iii) Explain what the gradient of this line represents.

(b) At 'SuperPrint' shop, there is no fixed charge and it costs 10 cents to print a
card.
(i) On the same grid below, draw another graph to represent the above
information. [ 1]

(ii) State the range of values of n for which it will be cheaper to print cards
at 'Quic.k.Print'shop.

c.
cents

400

300

200

100

n
0 10 20 30 40

Answer
(ai) (1)

(a)(iii) .. .. . . ... ...... ...... .. .. . . . . ....... .. .... ... .. . ... . . .. ......... .. ... ......... ........... ... [l)

(b)(ii) [I]

TKGS Preliminary Exam 2011 4016101 Mathematics Paper


20

Fo
for 23 The diagram shows the cumulative frequency of the marks scored by 200 students in Ex.amil
E··~rnmer's the exams. Us-
Use

~
~ 160 -l-'-~·-------1-~~+--~-+-~-...~~--~-1-~--;-~~1-"----t
e
><
-+
••• i
.. . -
l...
0
i + ~

t
I
I

0 20 40 60 80 100 marks

(a) Complete the grouped frequency table based on the graph above. (I]

Marks (x) 0-20 20-40 40-60 60-80 80-100

Number of students 6

(b) Using the graph, estimate


(i) the interquartile range, and
(ii) the minimum mark needed to obtain a distinction, if only 20% of the students
attained a distinction for the exams.

(c ) Calculate the mean and standard deviation.

Answer
(b)(i) [I]

(b)(ii} [l]

(c) Mean = ............ , SD= . . .. . . . .. . . . (2)


END OF PAPER

TKGS Preliminary Exam 2011 4016101 Mathematics Paper


TKGS PRELIMINARY EXAM 2011 MA Pl SUGGESTED ANSWER KEYS

la 7 (correct to I sig fig}


lb 1
533 %
3
2a 170.665
- x 0.07 = $11.17 (to 2 d p)
1.07
2b 170.665 -11.165
- xl=$145
1.10
3a 2n
3b n 2 +1
4a 632.8 nm
,ib )/ =k, where k is a cons tan/
let original frequency be /.
:. new fi"equency=3 .5f
A. =_L
-
632.8 3.5/
A.= l 80.8nm
(or l.808 XI 0 2 nm)
Sa n-40 000
Sb Map : actual
= (S emf : (2 km)2

3
Map area= - X 25 .. 18.75 cm 2
4
6a HH-12) 22°C
6b 440
Height~ - x(l0-(-5)]
22
= 300 m
?a 2
l -a +2ab-b2
= 1-{a2 -2ab+b 2 )
= l-(a-b) 2
= (l + a-b x1 - a+ b)
7b (x+y)(x-y)
= x2 _ y2
= (-6)1 -(0)2
=36
Sa (3m+3) 2 =9
3m = 3 - 3 or 3m = -3 - 3
m - 0 (rej) or m=-2
:.m=-2

TKGS Preltmmary Exam 2011 4016101 MathematicsPaper 146


22
8b 1 x l5-2x
2-+-=---
12 3 4
25 + 4x =45 - 6x
x=2

9a 96°
9b 96° +(n-1X1so 0 -136°)= 360°
n-7
9c a-2
IOa 9 l
- xl
9 3
-x 2 = -x
4 4
IOb 33(2>'-t) =32+.r
Comparing powers.
6x-3 • 2+x
x=I
Ila x= 16
3 + 2(16)+ 3(6)+ 16+ 25
:. Mean=
25
- 2.69 (to 3 s.O
lib x- 7
I le x JO
l2a m=2 3 X3Xll 2
:. m = 2904
n=2 2 x3 2 x7xll
:.n 2772
12b For mk to be a perfect cube,
m=2 3 x3x11 2
:.k =3 2 x ll
k 99
13a
13bi
P(2 balls taken arc green)= 83 X 72
3
=-
28
13bii 3 5
"Y(afleast rball 1s yellow)= 8X 7 X -:.l +58X 47
25
=-
28
OR
P(at least I ball is yellow)
=I - P( none of the balls are yellow)

= t-(~x~)
25
=-
28

147 TKGS Preliminary Exam 2011


4016101 Mathematics Poper
23
14a gradient
-10-2
=--
7+2 I
-4 1
=-or-I-
3 3
14b 8( I, 2)

14ci I
60=-(3+/)xl0./=9
2
x-coordinate is 0.
14cii -4
y = - x + c. sub (0. -/ 0)
3
-4

15a
y=
3
x-10 or3y+4x+30=0

15pq +30ps-3q 2 -6qs


II
15bii 3x-2 (x-3)(x-1)
x
2(x - 3) 3(2 - 3x)
-(x-1)
=-
6
J
=-(1-x)
6
16a Lei x be 1hc no of sludents who speak bolh English nnd French.
12- x + x + 8+15 - x = 30
r-5
therefore no. of students who can speak onlv one lan1tuaRe = 7 + 10 =17
16b FnE,;;F
16c All studcnls who speak German nlso speak English. I
17a 298°± JO
17b Sec diagram below.
17c Sec diagram below
18a
y = (x-1)2-4
.
18b
- x 2 -2x-3
y-intcrccpt (0, - 3) y • (t - J)l.J ' ...
x-intcrcept (-1, 0) and (3, 0)
Minimum turning poinl (I, -4) I

I .
(-1, 0)
I ' . .'' I /1
(3,0)
' I
I

'

\.\ J I
'
''
I
I I
I
19a LAED = LBEC (vertically opposite Ls}
LADE = LBCE (Ls of the rnme segment)
~
I
I

'
(I, -4)
1
LEAD= LEBC (Ls of the same segment)

. . MED and MEC are similar. I

TKGS Preliminary Exam WI I 4016101 Mathematics Paper 1•


24
: 19b BE BC
- =-
AE AD
3
BE=-x4
9
l
=l-cm
I 3
::?Oa Correctly label Q (2, 3).
20b
.J8
20c
OC=(2.5)
I 2.5
pod -AB=2y-x
21a JO mis
21b Let the speed of the particle be v mis.
10-v 10
- - =-
2 15
2
v=8-mls
3
I 21c Sec Graph.

2ld 19.2 km h-1


22ai C= 100+ 6n
22aiii Gradient represent the rate at which the cost 1s increasing.
22bii n >25
I 23a 46,98,46,4
23bi 20-2 1 marks
23bii 62 marks
23c Mean = 49.6 marks, SD = 16.2 (to 3 s.f)
MATHEMATICS
PAPER 2
4

2 The table below shows the petrol prices ($) per litre for a particular grade of petrol
o ffieredb1y 3 petro l stat10ns.
Station A B c
Price per litre 2.040 1.938 2.035
($/litre)

(a) Represent the prices by a 3xl matrix D. (1]

Let E = 1001 where I is a 3x3 identity matrix.

(b) Find ED.


What do the elements in ED represent? [3]

Peter purchased $50, $80 and $60 worth of petrol from station A, B and C respectively.

1
2.040
1
(c) Evaluate (50 80 60 - - and describe what the matrix multiplication represents.
1.938
1
2.035
[2]

(d) Station A gives a 14% discount on its petrol price when paid with a bank's credit
card. Calculate the amount of petrol, in litres that can be purchased now with $50. [2]

L TKGS Secondary 4 Preliminary Examination 2011 4016102 Mathematics


5

3 A soda drink manufacturing company packs 8 cylindrical cans of soda in a crate as shown in
the diagram. Each cylinder has a radius of 6 cm and a height of 15 cm.

(a) Calculate

(i) the dimension of the crate required to contain the 8 cylindrical cans, (1)

(ii) the amount of empty space in the crate that is not occupied by the cylindrical [2]
cans.

The company decides to manufacture the soda in the form of smaller similar cylindrical cans
with radius x cm and height y cm instead. The volume of one such cylinder is I 000 cm 3 .

(b) Find

(i) the total volume, in m', for 2900 such cylindrical cans, [I]

(ii) x, [2]

(iii) y. [2]
6

4 (a) Consider the following sequence

Line I l+l--=- I
2
3
lx2

Line 2 l+---=--
I
2
l
3
7
2x3

Line 3 l+---=--
1
3
I
4
13
3x4

(i) Write down Line 4 and 5 of the sequence. [ 1]

(ii) Express the n'h line of the sequence in a similar pattern. (1]

(iii) Hence, show that

(_3-1) +(-7-1) +(_!1_-1) +. . +(~ -1) + (~ -1) + (_2!_-1)


Ix2 2x3 3x4 7x8 8x9 9x10
9
10 [2]
4 (b) A board game comprises a six-sided fair die, numbered 1, 2, 3, 5, 7 and 11 and a grid
of triangles, as shown in the diagram below.

In a game, a button is placed on triangle X (starting point) and the players throw the
die twice and add the two numbers obtained.

(i) Draw a possibility diagram to show all the possible sums of throwing the die
twice. [2)
If the sum of the two numbers is an even number, the button is moved two triangles
in an anti-clockwise manner.

If the sum of the two numbers is an odd number, the button is moved one triangle in
a clockwise manner.

(ii) Find the probability that the sum of the two numbers is an even number. [1]

The die is thrown 4 times.

(iii) Find, as a fraction in its simplest form. the probability that the button finishes
at Q. [I]

(iv) Find the probability that the button finishes at R. [ l]

15
TKGS Secondary 4 Preliminary Examination 2011 4016102 Mathematics
[Turn Over
8

5 (a) (i) Express as a single fraction in its simplest form


x 3-x
5 7 [2]

(ii) .
H ence, sol ve th e mequa 1·1ttes
. - 1$ -x - -
3--x <2.
5 7
Represent your solution clearly on a number line. [3]

(b) It is given thaty =2(x 2 -2x +4.5).

(i) Express y = 2(x 2 -2x + 4.5) in the form y = 2(x-h)2 + k where h and k are [2]
integers.

(ii) Write down the value of x for which the minimum value of y will occur. [1]

(iii) Make x the subject of y. [2]


9

6 (a) In June 2011, the price of a car was $56 000. Over the next two months, the price
increased by 25% and then decreased by 15%. Mr Tan, a potential car owner, said
that the overall increase in the price of the car was l 0%. Do you agree with Mr [ 2]
Tan? Justify your answer.

(b) Mr Goh bought a sports car from a car dealer. He made a down payment of 40% of
the selling price and paid the remaining amount by taking up a monthly instalment
payment scheme jointly offered by Bank A and Bank B:

From Payment Scheme


Simple interest of 3% per
50% of sum borrowed Bank A
annum for 3 years
Compound interest of2.4% per
50% of sum borrowed BankB annum compounded monthly
for 3 years

Given that Mr Goh made a down payment of S$4 l 832,


(i) Write down the loan amount taken from each bank. [2]

(ii) calculate his monthly instalment payable to Bank A and Bank B [4]
respectively.

After 3 years, Mr Goh sold his sports car to a Swiss friend at a loss of 15% of the
price he paid for it. The price he paid for includes the interests paid to the banks.

(iii) Find his selling price. [2]

(iv) Given that he received 56 900 CHF (Swiss Franc), calculate the rate of [I]
exchange, in Swiss Franc (CHF) to one Singapore dollar (S$), giving your
answer correct to the nearest cent.
10

7 ABCD is a trapezium. AB= 18 cm, DC= 8 cm, AD= BC and the circle touches all the
sides of the trapezium.

A 18 cm B

(a) Find the length of BC. (2)


Write down clearly, a property of circle used to find the length of BC.

(b) Show that the height of the trapezium is 12 cm. [2)

(c) XW and XY are two chords of the circle as shown below. XY = 7.2 cm.
WY is the height of the trapezium.

A
Find

(i) .LXWY in radians, [2]

(ii) the minor arc length XZY, [2]

(iii) the area of the shaded segment. [3)

TKGS Secondary 4 Preliminary Examination 201 I 4016102 Mathematics


11

8 In the diagram, J, Sand B represent three islands near a country.


JS= 4000 m, SB = 2600 m, JB = 6150 m.
The bearing of B from Sis 155°. LJSB is an obtuse angle.

(a) Find LSJB. [2]

(b) Calculate the bearing of B from J. (4]

(c) The angle of elevation from B to the top of a tower at Sis 53°.
Find the height of the tower. [2]

(d) A boat sails directly from J to B. It moves at an average speed of 43.2 km/h.

Calculate

(i) its shortest distance to S; [2]

(ii) the time taken for it to reach B, correct to the nearest minutes. [2]
12

9 In a factory, a frustum-shaped iron pail, of height 9 cm and radii 1 cm and 4 cm, was made by
cutting out a right circular cone from a larger right circular cone.
Diagram I shows the frustum-shaped iron pail.

Diagram I
(a) Find

(i) the height of the cone that was cut off, [2]

(ii) the volume of the pail, leaving your answer in terms of 1t. [2]

(b) The pail is filled with waste liquid to a depth of 6 cm and spherical ball bearings of
diameter 4 mm are dropped into the pail.

(i) Show that the amount of waste liquid in the pail is 26;r crn3 . [2]

(ii) Hence, determine the least number of ball bearings needed for the waste liquid
to overflow. [2]

(c) Another way of making the iron pail is by joining the edges AB to DC of an iron
sheet, as shown in Diagram II.

To protect the pail from rusting, a layer of paint is


coated over the entire exterior surface of the pail
(including the base).

(i) Show that the length of OC and OD are JlO ''


''
and 4M cm respectively. [2]
o'' . . ,
(ii) Given that 0.8 m/ of paint can coat a surface area
' ' ...
of l cm2 , determine the amount of paint, correct
to the nearest ml, required to coat the entire
exterior surface of the pail. [2]
Diagram II

159 TKGS Secondary 4 Preliminary Examination 201 I 4016102 Mathematics


13

10 Answer the w hole of this question on a sheet of graph paper .

The table below gives some values of x and the corresponding values of y, where
2400
y= 10+ -
. x

x 100 200 300 400 600 700 800 I 200


y 34 22 18 16 14 13 12

(a) Calculate the value of y when xis 700. [I]

(b) Using a scale of 2 cm to 200 units, draw a horizontal x-axis for 0s;xs;1200 .
Using a scale of 2 cm to 5 units, draw a vertical y-axis for 0 s; y s; 40.
On your axes, plot the points given in the table and join them with a smooth curve. [3]

(c) Use your graph to estimate

(i) the value of x when y is 30. [l]

(ii) the gradient of the curve at x = 220 by drawing a tangent. [2]

(d) (i) On the same axes, draw the graph of y =25 - - x for 0 $ x s; 1200 . [2]
60

(ii) The values of x where the two graphs intersect are the solutions of the
equation Ax 2 - 900x + B = 0. Find the value of A and of B. [2]

End of Paper 2

TKGS Secondary 4 Preltminary Examination 201 I


16
4016102 Mathematics O
[Tum Over
l "T

TKGS PRELIMINARY EXAM 2011 SUGGESTED ANSWER KEY FOR MA P2

l (a) 120
-
x
(b) 232
--
2x+8
(c) ~=120 _2.75
2x+8 x
232x = (2x + 8)(120-2.75x)
232x = 240x - 5.5x 2 + 960- 22x
5.5x2 + I4x-960 = 0
l lx 2 + 28x- 1920 = 0
(d) -28 ± ~28 2 -4(11)(-1920)
x=
2(11)
x = 12 or -14.5(3s/)
(e) 232
=$7.25
2(12) + 8

2 (a)
(2MO]
1.938
2.035
(b)

(193.8
2M]
203.5
The elements represent the amount payable in each
station for 100 litres of petrol.
(c) (95.3)
The matrix represents the total amount of petrol in
litres pu:chased by Peter from the 3 stations.
(d) = 28.5 litres
3 (ai) 48 cmx24 cmx15cm
(aii) Amount of empty space
2
= 48x24x15-8(1t)(6) (15)
= 3710 cmJ,(to 3 s.f)
(bi) 2.9 mJ
(bii)
x= r,! = 5.031 = 5.03 (to 3 s.f)
(biii) 5 031
y=.::x15= · xt5
6 6
y =12.57698 ..
y =12.6

TKGS Secondary 4 Preliminary Examination 2011 4016102 Mathematics


15
4 (ai) . I 1 21
Lme4: l+-- - = - -
4 5 4x5
. 1 I 31
Lme 5 : 1+- - - = --
5 6 5x6
(aii) 2
n +n+l
L.men: 1+----=
1 l
n n+l nx(n+I)
111111 l l I
( aiii)
1--+---+---+---+ ..... +---
2 2 3 3 4 4 5 9 10
I
= 1--
10
4 (bi) Possibility Diagram to show the sum of a die thrown
twice.

+ 1 2 3 5 7 11
l 2 3 4 6 8 12
2 3 4 5 7 9 13
3 4 5 6 8 1( 14
5 6 7 8 10 1~ 16
7 8 9 10 12 ]£ 18
11 12 13 14 16 H 22
.
(bii)
P(sum 1s even) = -26 = -13
36 18
(biii) 65
-
162
biv) P(at R) = 0
5 (ai) x 3-x
----
5 7
_ 7x-5(3-x)
35
12x-15
=
35
(aii) x 3-x
-1$----<2
5 7
.
-35$12x-15 and l 2x -15 < 70
2 1
x~-l- x<?-
3 12

2 l
:.-l-$x<7-
3 12
(bi) 2
y = 2[(x-1) -1+4.5)
= 2(x-1)2 + 7
(bii) The minimum value occurs when x = 1.
(biii)
y-7 =(x-1)2
2

4016102 Mathematics
162
TKGS Secondary 4 Preliminary Examination 201 I
[Tum Over
16

x=±~Y~ 7 +1
6 (a) Disagree with Mr Tan.
Or (1.25 x 0.85-1) x 100% = 6.25%
(bi) $31374
(bii) $949.94 (cor to 2 dp)
$936.49 (to 2 d p)
(biii) Selling price =
0.85 [41832+36(949.935 + 936.4947)]
= $93281.95
(biv) 0.61 CHF (nearest cents)

7 (a) BC = 4 + 9 = 13 cm (tangent from external point)


(b) Height= ~13 2 -(9-4) 2 =12cm(PythagorasTheorem)(Shown)

(ci) LWXY = 90° (rt angle in semicircle)

. LXWY =7-
sm ·2
12
LXWY = 0.6435 rad
(cii) Let the centre be 0.
LYOX = 0.6435x2 = 1.287 rad (Lat centre= 2 Lat circumference)
length of arc xzy = 6 x 1.287 = 7. 72 cm
(ciii) 16.1 cmi ( 3 s.f)
8 (a) - 1 1255
LSJB = cos ( - -) = 16.95° = 17 .0° ( I dp) or 0.296 rad.
1312
(b) BearingofB fromJ= 111.40+ 16.95= 128.4° ( 1 dp)
(c) h
tan 53°= - -
2600
Height= 2600 tan 53°
= 3450 m (3 sf)
(di) 1170 mz(to 3 sf)
(dii) 9 min ( nearest min)
9 (ai) Let the height of the cone that was cut off be x cm.
l x
-= - -
4 x+9
:. x= 3
Height of the cone that was cut off is 3 cm.

63 TKGS Secondary 4 Preliminary Examination 2011 4016102 Mathematics


17
(aii) Height of the larger cone = 3 + 9 = 12 cm
. -n
Volume of pail= 1()2
4 x12--n 1()21 x3
3 3
=637r=l98 cm2 (to 3 s.f.)

b(i) Let the new radius be y cm.


3 l
-=-
9 y
:.y=3
Amount of waste liquid in pail
= .!.n(3)2 x9-.!.n(l) 2 x3
3 3
= 26Jr (shown)

(bii)
Least number of ball bearings needed= ~ 16 ·254
-Ji(0.4f
3
= 433.59 ::::434 balls

c(i) oc = J32;l = .J10


OD= .J12 2 +4 2 = 4./10 (shown)
c(ii) 122 ml (to nearest ml)

10 (a) x = 13.4
(ci) 12.0
(cii) - 0.06 accept = 0.0587 [ ± 0.005]
(dii) x 2 -900x+14400 = 0
A = 1, B = 14400
Class Register Number

Name I l
4016/01 11/t1P2/EM/1

MATHEMATICS PAPER 1

Tuesday 16 AUGUST 2011 2 hours

VICTORIA SCHOOL

SECOND PRELIMINARY EXAMINATION


SECONDARY FOUR

Candidates answer on the Question Paper

READ THESE INSTRUCTIONS FIRST

Write your name, class and register number on all the work you hand in.
Write in dark blue or black pen.
You may use a pencil for any diagrams or graphs.
Do not use paper clips, highlighters, glue or correction fluid.

Answer all the questions.


If working is needed for any question it must be shown with the answer.
Omission of essential working will result in loss of marks.
You are expected to use a scientific calculator to evaluate explicit numerical
expressions.
If the degree of accuracy is not specified in the question, and if the answer is not exact,
give the answer to three significant figures. Give answers in degrees to one decimal
place.
For n, use either your calculator value or 3.142, unless the question requires the
answer in terms of n .

At the end of the examination, fasten all your work 5,ecurely together.
The number of marks is given in brackets [ ] at the end of each question or part
question.
The total number of marks for this paper is 80.

This paper consists of 13 printed pages, including the cover page.


[Turn over
2
Mathematica/ Formulae

Compound interest

Total amount = P(l +-2:._)n


100

Mensuration
Curved surface area of a cone = nrl

Surface area of a sphere = 4nr2

Volume of a cone= .!,.r2 h


3
4
Volume of a sphere = -nr 3
3

Area of triangle ABC= .!_ ab sin C


2
Arc length = rB, where B is in radians

Sector area = ~ r 20, where () is in radians


2

Trigonometry
a b c
--=--=--
sin A sinB sinC

a2 = b2 + c 2 - 2bccosA

Statistics

Mean = 'f.Jx.
If

Standard deviation = Ifx ('i.fx)


2
- - - --
L.! If
2

VICI OIUA ~(.'I JOO! 20 I I \ l/4f'2WM/ !

4
3
311003 - 26 x 311000 +I
1 Evaluate
311001 + 3

Answer ............................................ l2]

2 Light travels 1 metre in 3.3 nanoseconds. How far will it travel in 11.55 picoseconds?
Express your ans~er in standard form.

Answer ...................................... . m (2)

3 Factorise complete I) 2a2 .!_(b - a )2 •


.2.

Answer· ..........................:-................ (2 J

4 (a) Joseph sets off from his home to the Botanic Gardens in a taxi at 16 31 . Due to a
traffic jam, he did not manage to reach Botanic Gardens until 18 09. Calculate the
time taken, in hours, for Joseph to reach the Botanic Gardens. Give your answer in
exact form.

Answer(a) .............-.. ... .. hours [I}

(b) Find the distance, in kilometres, between his home and the Botanic Gardens if the
taxi Joseph took. travelled at an average speed of 40 km/h.

Annver (b) ............................. . km [ 11


-- - - - - - - -- - -- - - - -- - -- -----
5 The ratio of a man's expenditure to his savings is 2 : l. If his expenditure is reduced by
25% and his savings is increased by 25%, what is the new ratio of his expenditure to his
savings?

Answer .. .................... .. .. ...... 121


Vl\IOl<IA S1"HOOl 201 I I 1/4P2/EMl l
4
6 G iven that - 3 ~ x !>: -1 and 2 ~ y 5 5 , find the

(a) largest possible value of .l,


x

Answer(a) ......... . .................... [ 1]

x2 + y2
(b) smallest possible value of _ __:;__
y

Answer (b) ...................................... [I)

7 Written as the product of its prime factors, 480 = 2 s x 3 x 5 , 576 =2 6 x 3 2 and


112 =2~x7 .

(a) Find the lowest common multiple of 480. 576andl12.

An~wer (a) ................... .. ... ......... [I]

(b) Find the smallest positive integer /1 for which 480n is a muluple of 576.

Answer(b) n :::: ................................ [I]

8 Study the following sequence. The integers from row I to 5 are consecutive even numbers

Row Integers No. of even


I Total in a row Average in a row I
numbers (7) (A)
(N)
l 2 I 2 2
2 4 6 2 '
10 5
,__3 8 10 12 3 30 JO
4 14 16 18 20 4 I 68 q___
5 22 24 26 28 30 5 .' p
·-'----- - --
26

(a) Write down the value ofp and of CJ.

Answer(a)p 'q = .... JI I


(b) Write down a fonnula connecting N and T.

'1t1swer (b) ..... . . . Pl


VI( l ORl1\ SCllOOL 2011 1l/4P2/EM/l

6
5
9 The temperature at the top of a mountain, of height 4000 m is -q °C. The temperature at
sea level is p °C where p and q are positive integers. Assuming that the temperature
changes uniformly with height, write down an expression, in tenns ofp and of q for the

(a) difference between the two temperatures,

Answer (a) .............................. ° C [I]


(b) temperature at a height of 1600 m.

Answer (b) .............................. °C [2)

10 The coordinates of points A, Band Care ( 2, -1), ( 4, 1) and ( 5, a) respectively.

(a) [f A, Band Care collinear, find the value of a.

Answer(a)a== ............................. (!]

(b) Find the equation of the line passing through B which is parallel to the line
2x=44-y.

Answer (b) ...................................... f 1]

VICTORIA SCHOOL 2011 I 1/4P21EM/I

6
6
11 The diagram shows the major sector of a circle,
centre B and radius 15 cm. The reflex angle
ABC = 216°. A cone is formed by joining AB and
BC together.

Calculate the 15 c
(a) radius of the cone,
A

Answer (a) ................................. cm (2)


(b) height of the cone.

Answer (b) ................................ cm [ 1]

12 c = {x : x is an integer and 1<x::;20}


A = {x: x is a prime number}
B = {x: x is an odd number}
C = {x: x is a factor of 20}

List the elements in

(a) Ar1C,

Answer (a) ....................................... [ 1)

(b) (AvC)'nB.

Answer (b) ....................................... [2)

13 ABCDEF is a regular hexagon with centre 0.

Calculate the

(a) sum of the interior angles of the hexagon,

Answer (a) .. ................... . ........... (1]

(b) size of one of the exterior angles of the hexagon,

Answer (b) ......................................... [1 )

(c) reflex angle AOC.

Answer(c) ......................................... (1]

VICTORIA SCllOOL 2011 11141'2/EM/I


7
14 In the figure, mis parallel to PS and ZR and PH is parallel to RS. Angle PHT =136° and
angle PZR =124°.
H T

Calculate z R

(a) "
HPZ,

"
Answer (a) HPZ = ....................... [2]
,,
(b) PSR.

"
Answer (b) PSR = ........................ [I)

15 " = 2.4.
B lies on AC such that BC= CD. Angle ADC= 90°, BC = 5 cm and tan DCB
D
Find the exact value of

(a) AD,

A B 5 c

Answer (a) .................................cm [2]

(b) " .
cos DCB

Answer (b) .......... ~ . .. .. . ... f2]

V !Cl ORI/\ SCI IOOL 20 11 l l/4P2/F.M/ l


8

16 (a) Solve <240 Iy+' =3432.r


7

Answer (a) x = ......:........................ [2]

2a ~ )
2 3 )-2 + ( Sb-lcs
2a b c -S . . .. . .
(b) Simplify ( , leaving your answer m pos1t1ve md1ces.
5

4m ~er (h>.. ......... . ................ [2]

17 ". survey was done to find the average number of hours each student in a class spcn
surfing on Facebook per day. The results are sho,.,n in the table below.

- um_b_er_o_r_h_o_u_rs_ ____.._'__.__2___.._3__._
N
'-Number of children 6 8 x
49_ I. 45 I. IJ
n3

(a) If the average number of hours each student spent surfing on Facebook per dn\ is
3.15 hours. find x

Answer (a) x - . (2)

(u) If the mode is 4. find th'- grcatl!st possible value o l .c

r1nrner (h) x =- •• .••..•... ••• ••• • ••. [ 1J

(c) If the median number of hours each student spent on surfing Face book is 4. fin<l the
possible values oft.

1111.IW('f" (c) X -:-. ,,,


.f{ '(II IA\(IHJOI LOii 11 /'11'.!./l.;\1/i

10
9
18 Mrs Tan will either cook food or bake a cake for her family during the weekend. The
probability that she cooks food is i.
5
If she cooks food for her family, the probability that

she will overcook her food is ~ . If she bakes a cake, the probability that she will bum the
8
. I
Cake I S - .
IO

(a) Complete the tree diagram below. [2]

Answer(a)

Will overcook food

Cooks food

Will not overcook food

Wi 11 burn cake

Bakes cake

Will not bum cake

(b) Find the probability that she will not bum the cake or overcook the food during the
weekend.

Answer (b) ..................................... [2]

Solve the inequality 2 - x ~


3
19 (a) - x < x +3 .
2

Answer (a) ...................................... [2J

(b) A fruit seller bought a box of 142 apples for $35.50. lfhc sells each apple for $0.44.
what is the least number of apples that he must sell in order lo make a pro fit of not
less than $12?

Answer (b) ........... .............. apples [2)

VICTORIA SCHOOL 2011 I 1/4P2!EM/I


10
2
20 (a) Express -x2 +6x- 15 in the form lc-(x-h) •

Answer (a) ..................................... [2]

(b) Hence, sketch the graph of y = - x 2 + 6x -15 .


y
Answer (b)

0 x

[2]

21 (a) y is inversely proportional to~. It is known that y - 6 for a particular value of x.


Find the new value of y when this value of x is increased by 300%.

Answer (a) new y = ....................... [2]


(b) If Amos and Ben work together, they can complete a job in 12 days. Ben talces 5 days
to complete _!_ of the job alone. How many days will Amos take to complete the job
6
alone?

Answer (b) .................... days f31


VICTOR!/\ SC 11001, 2U 11 I J/4P2/EM/l

!3
- .L
11
22 The table below shows the number of cupcakes and muffins sold by two shops, shop A
and shopB.

Shop A ShopB
Cupcakes 150 139
Muffins 147 143

It costs $0.10 and $0.15 to bake a cupcake and a muffin respectively. Each cupcake was
then sold for $0.75 while each muffm was sold for $0.85.

(a) Write down a 2 x 2 matrix, Q, to represent the number of pastries sold by both shops.

Answer (a) . ................................. [ 1)

It is given that P = ( 0.10] and R =(0.75) .


0.15 0.85

(b) Calculate Q(R - P).

Answer (b) ...................................... [2]

(c) Explain what your answer to (b) represents.

Answer (c)............................................................................................................................ .

............... ... . ............. ..... -···· ................................................................. [ l]

VICTOR!'\ \Cl 1001. 211 l I


u.
23 In the diagram, ABCD is a parallelogram. AE, DE and BD are straight lines where AE
intersects BD at X and BC at Y respectively. AD = 6 cm and BY = 2 cm.
A B

D c E

(a) Show that triangles ABY and EDA are similar.

Answer (a) .................................................... .

.. ..................................................................................................................... [2]

(b) Find

EY
(i)
EA '

Answer (b)(i) ........................... [ I ]

Area of!:. BEY


(ii)
Area of !:. CEY '

Answer (b)(iz) ............................... [ I]

Area of~ BEY


(iii)
Arca ofAYCD

1lm111er (h)(iii) 121


VICI 0 1{1/\ SCllUOL :W 11 1l /41'21EM / I
13
24 TA and TB are tangents to a circle centre 0 and radius r cm. AT produced meets OB
produced at W. Given that BW= 12 cm and TW= 13 cm.

0 r B 12 w

(a) Write down the length of TA.

Answer (a) ............................... cm [2)

(b) Calculate the value of r.

Answer (b) .............................. cm [2]

(c) Calculate the perimeter of the shaded region ATB.

Answer (c) .............................. cm [2]

(d) Calculate the area of the shaded region ATB.

2
Answer (d) .............................. cm [2]

End of Paper

Tl11s document 1s 111tended for mternal circulation m l'tctoria School only No part of 1/11s document may be reproduced, stored in a retrieval
system or transmuted m an) form or l>y any mi!ans, electromc. mechamcul µhotocopymg or othern 1se, wtthout the pri11r perm1ss111n of the
Victoria School Jnrerna/ Exams Com111 111ee

VICTORIA SCHOOL 2011 1l/4P2/EMJI


14
1 1 15(a) 12cm
-
3
2 3.5xl0- 3 m l 5(b) 5
-
13
3 l 16(a) 1.5
-(3a-b)(a+b)
2
4(a) 16(b) 125b4 c 15
1!2.hrs
30 8
4(b) 17(a) 10
65.!. km
3
5 6:5 l 7(b) 8
6(a) 2 l 7(c) Oor 1
--
3
6(b) 18(b) 17
2.!. -
2 25
7(a) 20160 19(a) x~ I
7(b) 6 19(b) 108 apples
8(a) p = 130, q = 17 20(a) - 6 -(x-3)2
8(b) T =N +N
3 20(b) I

. .
l
0 '
·6

·IS 1"\
I
I \
9(a) p+q oc 2 l(a) newy=3
9(b) 3 2 2l(b) 20 days
-p--q
5 5
lO(a) a=2 22(a)
(150 147)
Q = 139 143
lO(b) y=-2x+9 22(b)
(R-P) = (200.40)
Q 190.45
1 l(a) 9cm 22(c) It represents the profit earned by
Shop A and B respectively from
selling c upcakes and muffins.
l l(b) 12cm 23(b)(i) 2
-
3
12(a) {2,5} 23(b)(ii) I
-
I 2
12(b) (A u C)' n B = {9, 15 ( 23(b)(iii)
I 3.
5
13(a) 720 24(a) Scm
13(b) 60' 24(b) 7.5 cm
13(c) 240 24(c) 18.8 cm
14(a) " 24(d) 4.43 cmi
HPZ= 100
14(b)
-
I
~

PSR =44"

VICTORIA SCHOOL 2011 1l/4P2/EM/1 16


Name

4016/02 11 /4P2/EM/2

MATHEMATICS PAPER 2
Friday 19 August 2011 2 hours 30 minutes

'IJC:~sc:HOOL. ~iere>A.. tcHool. VCn::lftti'SQ-OOt ~roiltfAICHOC:I(. Y'C:'°"1.AS'GHOOt. *'~ SO'fOOl VJICrQMt SCHOO(_ VK:~IC>IOOC. WC'°"'1tSOIOOl W:::7WUASc:HOOL WC:IOIMICHOot. YJC~SCHO«
W!CfOM& SCHOOi. VICM>ll:liot SOIOOC. \'JC.roRJA .SCHOOt 'l"CTOI™ KHOOC. \C~ SC>IOOt. W'IC'.(Olit£A IOIOCX. VCJ'Oil'tM SCl«>OL W:rOltlo" .so.oo&. YJCTOllUI' ICHOOt ~OlltM SCHOOL \'tCT°""'.c:HOQ&. WCTOR!'4iSCHOOl
MCTOftlA SCl«)Ot "9CTOIMSOfOOC. C'r~SOtOO&. WC'T°""""~ VIC~SCHCOL VIC~ to«Xlt YIC'°""' SO<JOt. ~1'04l!IA $O!OOl VIC~.SCHOOt. -.«:-TOl!ltM ~ '1C'"rOlllM.ICHOQl. C-7°""'.tCHOOI..
~r(lllltM .sQ<JiO&. WC:ro.4M SCHOOC. W'letoft".,. lGl-IOClc. \'tero.tM ~ ~"°"""' SQfOOc. l'ICl'OIM KHOO&. \l'JCrOlb& .sGHOOt. we~ SCHOOi. ~'°"""' .S:Cl«Xlt. W:roM&.IO.OC:.. '4C~ ~ v.:r~ ·~

VICTORIA SCHOOL

SECOND PRELIMINARY EXAMINATION


SECONDARY FOUR

Additional Materials: Answer Paper


Graph Paper (1 sheet)
Plain Paper (1 sheet)

READ THESE INSTRUCTIONS FIRST

Write your name, class and register number on all the work you hand in.
Write in dark blue or black pen.
You may use a pencil for any diagrams or graphs.
Do not use paper clips, highlighters, glue or correction fluid .

Answer all questions.


If working is needed for any question it must be shown with the answer.
Omission of essential working will result in loss of marks.
You are expected to use a scientific calcu lator to evaluate explicit numerical
expressions.
If the degree of accuracy is not specified in the question, and if the answer is not exact,
give the answer to three significant figures . Give answers in degrees to one decimal
place.
For ", use either your calcu lator value or 3.142, unless the question requires the
answer in terms of n .

At the end of the examination , fasten all your work securely together.
The number of marks is given in brackets [ ] at the end of each question or part
question
The total number of marks fo r this paper is 100.

This paper consists of ~ printed pages, including the cover page.


i [Turn over
Mathematical Formulae

Compound interest

Total amount= P(t +~)n


100

Mensuration
Curved surface a rea of a cone= ,u/

Surface area of a sphere = 4rrr 2


t 2
Volume of a cone -Hr h
3

4 '
Volume of a sphere - -1n·
3

/\rca of triangle A RC ..!. absinC


2
Arc length= rD. where 0 is in radians

Sector area= ..!.r 2B, where 0 is in radians


2

Trigonometry

-a- = -b - - -c-
sin A sin B sin C

a2 = b1 + c2 2bccos A

Statistics

Mean = l:fr:
I.f

Standard deviation = I.jx/2


L. - (~
L.fifx )2

VICTORIA SCHOOL 20 I I 1 l/4 P2/EM/2 18


3

Answer all the questions.

1 (a) It is given that T J


=21' x ~ y . Express yin terms of T, x, g and ". [2]

(b) Solve the equation (E_- 2)( +


x-1
x 1) =17 . [3]
1
(c) Express as a single fraction in its simplest form : + b [3]
4b -a 2 a-2b

2 (a) John decides to deposit $180 000 into a bank for 3 years. Orange Bank offers a special
rate of 1.8% per annum simple interest for the first 3 years. Blue Bank also has a special
package which offers interest compounded half yearly at 1.8% per annum. Calculate the
total interest John would receive after 3 years if he deposits all his money into

(i) Orange Bank, [1]

(ii) Blue Bank. [3]

(b) During the Great Singapore Sale, a table is sold at a 40% discount. A further
reduction of 20% 1s given on the discounted price if a discount coupon is used. A
customer buys a table using a discount coupon. Calculate the price she has to pay
if the original price of the table is $360. [2]

(c) The original price of a car is $70 500. A car can be bought under either one of the
following schemes:

Scheme A: Cash payment with a discount of 5%.


Scheme B: Hire purchase terms of a cash deposit of25% followed by monthly
instalments of $800 for 5 years 9 months.

Calculate the difference in the amount paid under the two schemes. (3]
4
3

T s R

In the diagram, PQT and RST are tangents to a circle, centre 0. Angle QTS = 66°,
angle SAB = 33° and angle ACQ = 83°.

Find

(a) angle SQB, [I]

(b) angle BSR, (I]

(c) angle TOQ, [2]


(d) angleABQ, [2]
(e) angle CSO. [2}

4 The following table gives the Math marks obtained by l 00 students.

Math marks (x) 50 $ x < 60 60 Sx <70 70s x <80 80Sx <90 9os x < ioo I
Number of
students
28 14 18 28 12
I
(a) Calculate the

(i) mean mark, [2]

(ii) standard deviation. [2]

(b) Two students are chosen at random. Find the probability that one of them
obtained less than 70 marks and the other more than or equal to 90 marks. f2)

VICTORIA SCHOOL 2011 I l/4P2/EM/2


5
S A group of x number of Victorians signed up for a computer course and shared the total
cost of $2070 equally among themselves.

(a) Write down an expression, in terms of x, the amount each member of the group
should pay. [1]

One week before the start of the course, 6 students withdrew, but 3 new members signed
up for it. The total cost now increased by $30.

(b) Write down an expression, in terms of x, the amount each member of the group
should now pay. [1]

(c) If the cost per student was $37 more than what was originally expected, form an
equation in x and show that it reduces to 37 x 2 - 14 lx - 6210 =0 . [3]

(d) Solvetheequation37x2 -141x-6210=0. [3]

(e) Hence, find the initial amount each member of the group should pay. [l]

v ---------------------------- sports car

Speed (m/s) 16 ---------------------- motorcycle

5 6 15
Time (t seconds)
The diagram shows the speed-time graph of a motorcycle and a sports car travelling on the
same stretch of road. Both vehicles started from the same point. The sports car started its
journey from rest. 5 seconds after the motorcycle left and travelled with the same acceleration
as the motorcycle· s initial journey of 6 seconds. The sports car reached the speed of v mis
when t = 15. Find the

(a) acceleration of the motorcycle in the first 6 seconds, [1]

(b) speed of the motorcycle when / =13 , [2]

(c) value of v, [2]

(d) time taken for the sports car, after it started its journey, to overtake the
motorcycle. [4]

VICTORIA SCHOOL 2011 I l/4P21E.M/2


0

7 Answer the whole of this question on a sheet of plain paper.

A field is in the shape of a quadrilateral JKLM. It is given that JK = 115 m, JM =92 m,


KL= 100 m, angle KJM =58° and angle JKL =90°.

(a) Using a scale of 1 cm to represent 10 m, make an accurate drawing of the field.


Measure the angle JML. (3]

(b) A signboard, S, is equidistant from JK and JM, and is such that JS= MS. Label
the point Sand measure the distance of the signboard from M. [ 4]

8
A

B p ()
b

In the diagram. OA = a and OB= h. The points Rand P lie on A8 and OB respectively such
- 2- - -
that OP= - PB and AR= kAB. The lines AP and OR intersect at the point Q.
3

(a) Express the following in tenns of a and/or b .

(i) AB, [I J

(ii) AP. [2]

(b) Show that PR=(l-k)a+(k-f)b. [2]

(c) If PR is parallel to OA, find the' alue of k. [2]

(d) Find the numerical value of

area of L1ABP
(i) ll J
area of t.1AOB

area of tJPRO
(ii) 12]
area of. IA OB

VICTORIA SCHOOL 2011 I l/4P2/EM/2 22)


7
9
North

The diagram shows 4 points A, B. C and D on level ground. A is due north of B and
ADC is a straight line. Angle BAD= 38°, angle ADB = 56°, BD = 6 km and CD= 10 km.

(a) Calculate

(i) BC. [2)

(ii) angle DBC, [2]

(iii) the bearing of B from C, (1]

(iv) the shortest distance from D to BC. [2]

(b) A vertical tower stands at D. The angle of elevation of the top of the tower, T, from
Bis 2°. Find

(i) the height of the tower in metres, [l]

(ii) the maximum angle of elevation of the top of the tower, T, when observed
along BC. [2]

VICTORIA SCHOOL 2011 I 1/4P2 l::.M/2


8
10

,.t,
,. ,. I' I
I
'
''
,. ,. ,. I
I ''
40 ' 60

Ix
Diagram I Diagram II

(a) Diagram I shows a frustum of height x cm formed when a smaller right circular cone
is cut off from a larger one. The height and the base radius of the larger cone are
40 cm and 35 cm respectively.

(i) Show that the radius of the smaller cone cut off is equal to 2(40- x) cm. [2]
8

(ii) Hence find the volume of the frustum in terms of x. [3]

(b) Diagram II shows a container which is made by fixing a hollow hemisphere of


radius 35 cm to the hollow frustum from Diagram I. The height of the container is
60 cm and its thickness is negligible.

(i) Calculate, to the nearest cm3 , the amount of water needed to fill the container to
its rim. [4]

(ii) A small circular hole of radius 0.4 cm is drilled through the bottom of the
hemisphere and a cylindrical pipe is attached to it. The water in the container
is drained through this pipe at a constant rate of 1 mis.
Calculate, to the nearest minute, the time taken for the water in the container to
be completely drained off. [3]

VICTORIA SCHOOL 20 11 11 /4P2/EM/2


9
11 Answer the whole of this question on a sheet of graph paper.

The variables x and y are connected by the equation y = x + _?_ _ 4. Some corresponding
x
values of x and y, correct to 2 decimal places, are given in the table below.

x 0.5 1 2 2.5 4 5 6 7

y 6.5 2 a 0.5 b 2 2.83 3.71

(a) Calculate the value of a and of b. [I]

(b) Using a scale of 2 cm to represent 1 unit on both axes, draw the graph of
5
y = x + - - 4 for 0 : : :; x : : :; 7 . [3]
x

(c) Use your graph to find the least value of y. [1]

(d) By drawing suitable straight lines, solve the following graphically.

(i) x+_?_-7=0 [2]


x

(ii) 2x+~-9 ?: 0 [3]


x

(e) By drawing a tangent, find the coordinates of the point on the curve such that
the gradient of the tangent at the point is -1. [2]

End of Paper

This document is intended for internal circulatwn m Vic:trmu School only t>o par/ of this document may be reproduced. stored ma relrleval
system or transmllled m any form or by any mectn~'. el1:ctro111c, mechamcal pho1ocopymg or 01hen1 ise. w11ho111 the prior permtss1011 of the
Victoria School Internal t:xams Comm111ee.

VICTORIA SCHOOL 20 l I I l /4P2/EM/2


Answer Key 2011Prelim2 E Math Paper 2
gT2 ii)
la) y=x--
4Jr2

b) x = 2.88 or -5.38 b)
c) (;b:ba) c)

2ai) $9720 AreaMBP 3


d) =-
ii) $9941.34 AreaMOB 5
b) $172.80 Area M'RO 6
e) =-
c) $5850 AreaMOB 25

3a) LSQB=33°
b) LBSR =33° 9ai) BC= 14.3 km
c) LTOQ = 57° ii) LDBC =35.6°
d) LABQ = 50° iii) Bearing of B from C = 058.4°
e) LCSO = 17° iv) Shortest distance= 3..t9 km
bi) Height of tower = 210 m
4ai) Mean= 73.2 marks ii) 3.40
ii) SD= 14. l marks

b) Probabilitv = ~
, 275
r1 ( 40-x )3
.i9n 1000--
IOaii) - J cm,
3 6-l
L

bi) 138404 cm 3
$ 2070 ii) 46 minutes
Sa)
x
$ 2100
b)
x-3 lla) a= 0.5,b = 1.25
7 c) Least value of y = 0.4 5
d) x =15.-11-
37 di) x = 0.8 or x = 6.2
e) $138 ii) x $ 0.7 or x ~ 3.9
e) Coordinates (1.5,0.8)

6a) Acc.= 2 ms- 2

b) s= 15 -l ms- I
9
c) v=20
d) 16.1 seconds

7a)
,VJ L

J K

LJ}v/L = 1-l0°
b) \fS=53 m

8ai) AB = b-a

VICTOR1A SCHOOL 2011 I I /4P2/EMl2


__
XMs;/Jji
- --..
......
. .._
XTh~~SECONDARYSCHOOL

Preliminary Examination 2010

Mathematical Formulae
Compound interest

Total amount= P(l +-r-)n


100

Mensuration
Curved surface area of a cone = ;rrl

Surface area of a sphere= 4;rr 2

Volume of a cone= .!. nr 2h


3
4
Volumeofasphere= -nr3
3

Area of triangle ABC= _!_ ab sin C


2
Arc length = rB, where (} is in radians

Sector area= ..!.r


2
2
B, where 0 is in radians

Trigonometry
a b c
--=--=--
sin A sin B sin C

a2 = b2 + c2 - 2bccosA

Statistics

Mean= l./x
l,f

Standard deviation = I.fx2


- -- -- (l./x)2
L.f L.f
2
Answer all the questions.

1 (a) A farmer has l 500 tomato plants. He estimates that each plant will produce 6.5 kg
of tomatoes. Calculate the total mass of tomatoes, in tonnes, that he can expect to
be produced. (1 tonne= I 000 kg] [2]

(b) He proposes to apply 220 ml of liquid fertilizer which is imported from America to
each plant.
The fertilizer is sold in containers each holding 50 litres and costing US$165.
Calculate
(i) the number of containers he must buy, [2]
(ii) the total cost of the fertilizer in Singapore dollars at the exchange rate of
US$I=S$1.43. [2]

(c) The farmer has 0.3 hectares ofland available for the l 500 plants.
Calculate the average area, in square metres, available for each plant.
(1 hectare= 10 000 m2] [2]

In the figure, FC = kp where k is a scalar.


(a) Express r -q in termsofkand p. [I]

(b) Express ED in terms of k and p. Hence, show that ED is parallel to AB. [2]

(c) If ED= 4AB, find the vahie of k. [l]

(d) If the area of the triangle FEB is 10 square units, find the area of the quadrilateral
ABDE. [3]

Xinmin Secondary School 4016/02/20 I0

25:
3

3 (a)

PQRS is a cyclic quadrilateral. PR and QS meet at X RST touches the circle XPS
at S. If angle RQS =65° and angle PST= 110°, find, by stating the reasons
clearly,
(i) angle RPS, [1]
(ii) angle RSX, [l]
(iii) angle PQS. [l]

(b)

•I


22: I
I

33

The two containers shown in the diagram are geometrically similar.


Their heights are 22 cm and 33 cm.
(i) The diameter of the base of the larger container is 12 cm. Calculate the
diameter of the smaller container. [I]
(ii) Every part of the surface of the container is painted orange. Given that it
costs $350 to paint the larger container, find the cost of painting the smaller
container, correct to the nearest cents. [2]
(iii) The containers are completely filled with oil. Given that the smaller container .
holds 5 litres of oil, find the amount of oil the larger container holds. [2]

[Turn Over
4

4(a) Simplify each of the following.

(i) (: : l +(!-'. r, leave your answer in positive index. [3]

2a
(ii) - -x-a
-- - - -2- [3]
(x - 2a) 2 x2- 5ax+6a2 3a - x

(b) Solve the following equation.


7x-4 1 3x + 8 7+x
[3]
~-3=~-IOx

I
I
I
A B•
-------------~·--
8

A piece of wire consists of a straight part AB of length 8 cm and a semi-circular part


whose radius OD is of length 3 cm. Two beads X and Y move along the wire at the
same speed such that when X is at A, Y is at B and when X is at C, Y is at D.
(a) State the length of the arc CD. [l)
(b) Find, in radians, the angle D~OC. [2)
(c) Find the area of the shaded segment. [3]
5
6
Speed in mis
40

20

75 Time in seconds
0 10 35 55

The diagram shows the speed - time graph of a car over a period of 75 seconds. The car
decelerated uniformly from u rn/s to 20 mis during the first l 0 seconds and continued to
move at constant speed of 20 rn/s for 25 seconds. It accelerated uniformly to a speed of
40 mis in 20 seconds and then decelerated uniformly at a different rate for a further 20
seconds.
Calculate
(a) the speed of the car after 50 seconds, [2]
(b) the deceleration during the last 20 seconds of its motion, [ 1]
(c) the value of u, if the distance travelled in the first 10 seconds is 275 metres, [2]
(d) the distance travelled by the car in the first 55 seconds of its motion. [2]

v v
Figure l
Figure2
A conical funnel of depth 13 cm and vertical angle 84°, is held upright with its vertex V
downwards, as shown in Figure 1.

(a) Find the radius of the base of the funnel. [2]

A freshly painted wooden ball is carefully placed in the funnel. It is observed that the
highest point of the ball, centre 0 and radius r cm, is at the same level as the rim of the
funnel, as shown by the vertical cross-section view in Figure 2. The ball is now
removed and a circular line of paint is found adhering to the funnel.
(b) Show that the length OV is approximately 1.494r cm. [2]
(c) Hence, calculate the radius, r, of the wooden ball. [2]
(d) Find the length of the circular line of paint. [4]

[Turn Over
255
6
8 The table below shows the results of compressive strength tests on a batch of 100
concrete cubes.

Compressive Streneth (x k2/cm 2) Number of Cubes


160<x~180 l
180 < x ~ 200 7
200 < x ~220 26
220 < x ~ 240 38
240<x ~ 260 22
260< x ~280 6

(a) State the modal class. [1]

(b) Calculate
(i) the mean compressive strength, [2]
(ii) the standard deviation of the compressive strength. [2]
2
(c) If the mean compressive strength of another batch of 100 concrete cubes is 240 kg/cm
and its standard deviation is 18.5 kg/cm 2, compare and comment on these two
batches of concrete cubes. [2]

9
..
North
I

R
North
+
I
I
8
: 35°

p
A ship sails 8 km from P to Qon a bearing of 035°. It then sails 5 km from Q to R on a
bearing of 075°.

(a) Calculate
(i) the angle PQR, (1]
(ii) the distance PR, [3]
(iii) the bearing of R from P. (3)

(b) The ship finally sails from R to a position T, which is due North of Q. Given that

QTR" = 40°, calculate


(i) the distance RT, (2]
(ii) the shortest distance between the point Q and the ship as it sails from R to T. [2]
7
10 The whole of this question is to be done on a piece ofgraph paper.
The table below shows some values of x and the corresponding values of y for
1
y =3x+-.
x

x O. l 0.2 0.5 1 1.25 1.5

y 10.3 5.6 a 4 4.55 5.2

(a) Calculate the value of a. [I]


(b) Using a scale of 10 cm to represent l unit on the x-axis and 2 cm to represent
1 unit on the y-axis, draw the graph of y = 3x + _!_ for 0 .1 ~ x:::; 1.5 . [3]
x
(c) By drawing a tangent, find the gradient of the curve at the point when x = 1. [2]
(d) Estimate the range of values of x for which 3x+..!. $ 4.5. [2]
x
( e) On the same axes, draw the graph of y = 6- x and hence use your graphs to solve
the equation 4x2 -6x + l =0. [3]

11 A business makes toy buses and toy lorries. The following table is used to calculate the
selling price of each toy.

Labour (hours) Wood (blocks) Paint (tins) o/e Profit made


Bus 6 4 3 5
Lorry 3 4 2 10

Labour costs $8 per hour, wood costs $1 per block and paint costs $2 per tin. The
percentage profit made for a bus is 5% of its cost price and for a lorry is 10% of its cost price.

It is given that A =(: : D and B = rn


(a) Find AB. [1 ]
(b) Write down a matrix C, such that the elements of the product CAB will give the
selling price of a toy bus and a toy lorry respectively. Evaluate this prodµct CAB. [2]
(c) If a toy bus and a toy lorry are packed in a bag and sold at a discount of5% of their
total selling price, find , by writing down another matrix D, such that the elements of
the product DCAB will give the discounted selling price of this bag. Hence evaluate
this product. [2]
8

12 (a)

x 480
X t I I I I I
-·-..a--~---·
I I I ·--- ... --'--•-
' I I
- IL - JI - -'-
I - - - . • - - - _1 _ - t.. - .i -
I I I

-T--.--r---·
I

I
I

I
I

I
----.,--r-r- I I I

' I I
80

Diagram I

Square tiles, of side~ x cm, are to be stuck to a wall so that they fill a rectangular
space of 480 cm by 80 cm. Some of the tiles are shown in Diagram t.
(i) Write down an expression, in terms of x, for the number of tiles that will fit
across the top row. [I]
(ii) Given that 600 tiles are required to fill the whole space, find the value of x. [2]

(b)
480

y-5

Diagram 2

Diagram 2 shows another rectangular space which is also 480 cm by 80 cm.


This is to have one row of rectangular tiles stuck inside each edge so that they cover
the unshaded area only.
The tile measures y cm by (y- 5) cm. Each tile is placed so that its longer side is
vertical. Some of the tiles are shown in the diagram.
(i) Write down an expression, in terms ofy, for the number of tiles that will fit
across the top row. (I]
(ii) Given that 68 tiles are required to fill the whole unshaded area, form an
equation and show tharit reduces to 9 y 2 -185y+100 = O. [3]
(iii) Solve this equation and hence find the dimensions of a tile. [ 4)

Answers:

The End]
9
l(a) 9.75 tonnes (b )(i) 7 containers (ii) $1651.65 (c) 2 m2
--+
2(a) r-q =(k-l)p (b) ED= (2k-l)p (c) 2.!. (d) I 00 unit2
2
3(a)(i) 65° (ii) 65° (iii) 45°
(b)(i) 8cm (ii) $155.56 (iii) 1621
8
S I
x 2
4(a)(i) a2b2 (ii) 2
(b) 3 or -2-
(x-2a) 7

5(a) 8cm (b) 2~ radians (c) 9.94 cm2


3
6(a) 35 mis (b) 2 m/s2 (c) 35 (d) 1375 m
7(a) 11.7 cm (c) 5.21 cm (d) 24.3 cm
8(a) 220 < x s 240 (b)(i) 228.2 kg/cm 2 (ii) 20.8 kg/cm 2
9(a)(i) 140° (ii) 12.3 km (iii) 050.2°
(b)(i) 7.51 km (ii) 4.53 km
lO(a) 3.5 (c) gradient::::: 2 (d) 0.28 s x s 1.22 (e) x::::: 0.2 or 1.31

ll(a)
G~) (b) C = (l.05
0
0) ( 60.9)
1.1 ' 35.2
(c) (0.95 0.95); (91.295)
480
12(a)(i) (ii) 8
x
480 5
(b)(i) (iii) 20 or - ; 20 cm by 15 cm
y-5 9
ANDERSON SECONDARY SCHOOL
2011 Preliminary Examination
Secondary Four Express I Four Normal I Five Normal

CANDIDATE
NAME
CENTRE INDEX
NUMBER IsI I I I I NUMBER II I I I
MA TH EMA TICS 4016/02
Paper 2 13 September 2011
2 hours 30 minutes
Additional Materials: Writing paper (10 sheets)
Graph paper (1 sheet)

READ THESE INSTRUCTIONS FIRST

Write your name, centre number and index number on all the work you hand in.
Write in dark blue or black pen both sides of the paper.
You may use a pencil for any diagrams or graphs.
Do not use staples, paper clips, highlighters, glue or correction fluid.

Answer all the questions.


If working is needed for any question it must be neatly and clearly shown in the
space below the question.
Omission of essential working will result in loss of marks.
Calculators should be used where appropriate.
If the degree of accuracy is not specified in the question, and if the answer is not exact,
give the answer to three significant figures. Give answers in degrees to one decimal
place.
For n, use either your calculator value or 3.142, unless the question requires the
answer in terms of n.

At the end of the examination, fasten all your work securely together.
The number of marks is given in brackets [ ) at the end of each question or part
question.
The total of the marks for this paper is 100.

This document consists of 120 printed pages.


ANDSS 4E5N Prelim 2011 4016/02 [Turn over
Mathematical Formulae

Compound Interest

Total amount= Jl 1+-'100-)n


~

Mensuration

Curved surface area of a cone = rcrl

Surface area of a sphere= 41tr 2

1
Volume of a cone= -nr 2h
3

4
Volume of a sphere= -nr 3
3

Area of triangle ABC= ..!... ab sin C


2

Arc length = re , where e is in radians


Sector area = _!_ r 2 0, where 0 is in radians
2

Trigonometry

a b c
--=--=
sinA sinB sinC

a2 =b 2 + c 2 - 2bc cos A

Statistics

Mean = 'Lj'(
'Lf

Standard deviation = 2:~~2


1.J; - -(~]2
,;.,
1

ANDSS 4E5N Prelim 2011 Math (4016f02) (Turn over


3

1 The original price of a bag was US$358. After a discount, the sale price of the bag
became US$149.

(a) Calculate the percentage discount of the bag. (1]

On a particular day, there was a factory promotion that gave a further 30% discount
on the sale price for the bag.

(b) Calculate the new sale price of the bag. [1]


(c) Calculate the total percentage discount for the bag. [l]

A tax of 8.75% was imposed on the new sale price of the bag.

(d) Calculate the amount of tax payable. [1]


(e) Calculate the price of the bag in Singapore dollars, if the exchange rate
on that day is US$1=S$1.24. [1]

6m - 2mn + n 2 - 3n
2 {a) Simplify 2 2
[4]
8m -2mn-n
l lx-3 2x2 -8x
(b) Solve the equation - =0. [4]
x2 +x-12 16-x2

3 The diagram shows a paper weight in the form of a prism where ABCD, ABFE and
CDEF are rectangular faces and ADE and BCF are triangular faces. AB -= 6.5 cm,
A

AD= (12 - 3x) cm, AE = (2.x-r 3.2) cm and DAE= 30· .


E

6.5 cm
F
,,
,
____ ,,,'

B ,' C
(12 - 3x) cm
(a) Given that the difference in the area between the rectangle ABCD and the
triangle ADE is 21.6 cm2 .
Form an equation in x and show that it reduces to l 5x 2 - 23 lx + 468 =0. (2]
(b) Solve the equation 15x 1 - 23 lx + 468 = 0. (2]
(c) Calculate the volume of the paper weight. [2]

ANDSS 4E5N Prehm 2011 Math (4016/02) [Turn over


4

4 The diagrams below show the fust three triangles in a sequence of triangles of
increasing size.
Each triangle is formed by joining the neatly arranged dots such that the smaller
right-angled triangles are exactly identical.

Triangle I Triangle 2 Triangle 3

The table below shows the number of dots and the number of small right-angled
triangles in each of the larger triangles.

Triangle Total number of dots Number of small right-angled triangles


I 4 2
2 9 8
.,
.> 16 18
4 a h

n y z

(a) Find the value of a and of b. [2]


(b) Find the total number of dots needed to form Triangle 16. [I]
(c) Find the number of small right-angled triangles in Triangle 6. [I]
(d) (i) Express y in terms of n. [1]
(ii) Express z in terms of n. [1]
(e) State the triangle which has 450 small right-angled triangles. [I]

5 In the diagram below, 0 is the centre of a semicircle and TC is a tangent to the


semicircle at C. Given that OC = 12 cm and BT = 25 cm, calculate

(a) BOC in radians, [2]


(b) the area of the shaded region . [3)

ANDSS 4E5N Prehm 2011 Math (4016/02) [Turn over 2


5
6 The table below shows the average number of visitors per day in July 2011 (rounded
off to the nearest hundred) and the price of admission ticket for each adult and each
child in three theme parks, Alpha Land, Beta Land and Gamma Land respectively.

Average number of visitors Price of tickets


Theme Park per day in July 2011 {US$)
Adults Children Adults Children
Alpha Land 10 200 6 800 80 74
Beta Land 8 500 4 200 85 79
Gamma Land 2400 l 400 69 59

(a) Write down two matrices such that the elements of their product, under matrix
multiplication, will give the average total amount of ticket sales in each theme
park. Evaluate the product of these l\VO matrices and label the matrix T. [2]
I
(b) Evaluate - (1 I l)T. [1]
3

(c) State what the element in .!. (l I I) T represents. [I]


3
During the school holidays, each theme park reduced the price of an adult ticket by
I 0% and the child ticket by I 5%.
(d) Using matrix multiplication, find the discounted price for each t) pe of ticket
at each theme park. [2]

7 Jn the diagram, LABC = 90 • , AB= 16 cm, BC = 12 cm, BP= x cm and the point Q
on the line AC is the image point of B under tl1e reflection in the line AP.
Calculate, without the use of a calculator, the exact value of
(a) x. [5]
A

(b) tan CAP. [1]

16

\' p c
12

ANDSS 4E5N Prehm 2011 Math (4016/02) [Turn over


6

8 The distribution below shows the number of SMS messages 20 students sent in a
particular month.

78 74 84 78 85 94 88 72 68 76
25 84 78 82 78 87 80 92 95 69

(a) Illustrate the data using a stem-and-leaf diagram. [2]

Using the stem-and-leaf diagram, calculate

(b) (i) the mode, [l]

(ii) the mean, [1]

(ii) the standard deviation of the distribution. (1]

(c)

'
I

-----+------+-----+-----t-------+------
I
I
Number of SMS
mcssagcs
25 a 95

The distribution is represented using a box-and-whisker diagram.


(i) Find the value of a. [ 1]
(ii) Calculate x2 - x1 and explain what x2 - x1 represents. f2]

(d) Another student sent p SMS messages in that same month.


Write the range of values of p such that the median is 80. [1]

9 In the diagram, PQRS is a rhombus where QS is one of the diagonal.


RS is produced to point T such that Sis the midpoint of RT. PS and QT intersect at 0.

Q p

(a) (i) ShO\.\ that triangle QRS and triangle PST are congruent. [3]

(ii) Explain why PQST is a parallelogram. [21

(b ) Show that triangle POQ is similar to triangle RQT. [2]

ANDSS 4E5N Prelim 2-011 Math (4016102) [Turn over 26~


7

10

- - - 4 l
In the diagram above, OA =a, OB= b and CA =- a - - b.
3 3
Mis the midpoint of AB and AC cuts OB at D.

(a) Express, as simply as possible, in terms of a and b,


(i) OM, (1]
(ii) oc. [l]

(b) What can you conclude about OC and AB ? [1)

(c) Find the numerical value of


(i)
oc [1]
AB'
area of D.OCD
(ii) [I]
area of MBD

(d) Express, as simply as possible, CD, in terms of a and b . [l)

(e) Find the numerical value of


area of D.OCD
(i) [1]
area of D.OCA '
area of tlOAM
(ii) [l J
area of tlOCB '
area of t.BCD
(iii) [l]
area of OABC

263 ANDSS 4ESN Prehm 2011 Math (4016/02) [Tum over


8

11

A D

B c
The diagram above shows a playground in the shape of a regular hexagon ABCDEF
with centre 0 A ci rcular fountain with centre X lies in the hexagon such that ll

touches AB and BC. It is given that AB= 50 m and BX= 12 m.

(a) Calculate
(i) LABC, [I]
(ii) the area of triangle AOB in kilometers square. leaving your ans\ver
in standard form. l2J
(iii) the radius of the fountain. [3]

(b) A boy walks directly from A to E. Calculate the distance he walks. (2]

(c) A lamp post of height 8 m stands vertically at 0.


A bird sits on top of the lamp post.

(i) Calculate the angle of elevation of the bird on top of the lamp post
from A. [ l]

(ii) The boy decides to walk directly back along EA until he reaches a
point T from which the angle of depression of the boy from the bird
is the greatest. Calculate the distance OT. [2 j

ANDSS 4E5N Prehm 2011 M ath (4016/02) [Tum over 264


9

12 A closed container, A, of negligible thickness is made up of a cylinder of diameter


22 cm and height 2 cm, and a right cone of height 42 cm as shown.

22
Container A
(a) Calculate

(i) the volume of Container A, leaving your answer in terms of 7t. [2]

(ii) the surface area of Container A, giving your answer to 2 decimal place. [2]

Container B

A closed hollow Container B which has the same volume as Container A, is made up
of a right pyramid of height 42 cm and a cuboid of height 2 cm. Both the pyramid and
the cuboid have the same square base.
(b) Show that the length of the square base, correct to l decimal place, is 19 .5 cm. [2]

(c) If Container B is filled with water up to ~of its volume, find the depth of water
4
in the container. [4]

Container B

ANDSS 4E5N Prehm 2011 Math (4016102) (Turn over


10

13 Answer the whole of this question on a sheet ofgraph paper.

The table below gives the x- and y-coordinates of some points which lie on a curve
y = -x3 + x 2 + 5x + 3.

-3 -1 0 1 2 3 4

I: I 24 0 3 8 9 0 -25

(a) Using a scale of 2 cm to represent 1 unit on a horizontal axis for -3 ~ x ~ 4


and 2 cm to represent 5 units on a vertical axis for- 25 5 y S 25, plot the points

given in the table and join them with a smooth curve. [2]

(b) Using your graph, solve the equation - x 3 + x 2 + 5x + 8 = 0 . [2]

(c) By drawing a suitable tangent, find the gradient of the curve at the point where
x = l. (2]

(d) The x-coordinates of the points where the line y = -3x + 3 intersects the curve

are the solutions to the equation -x 3 + x 2 +ax+ b = 0.


Find the value of a and of b. [2)

(e) Using the same axes, draw the line y =-3x + 3. From your graphs, determine
the range of values of x for - 3 ~ x ~ 4 for which - x 3 + x 2 + 5x + 3 > -3x + 3.
[2)

(f) The line y =-~ x + k intersects the curve y =-x3 + x2 + 5x + 3 at least two
2
points. Find the largest value of k. [l]

ANDSS 4E5N Prehm 2011 Math (4016/02) [End Of Paper 266


ANDERSON SECONDARY SCHOOL
2011 Preliminary Examination
Secondary Four Express I Four Normal I Five Normal
MATHEMATICS 4016/02

ANSWER SCHEME

l(a) 58.4% 6(a)


(1319 200)
1054 300
248 200
l(b) $104.30 6(b) (873 900)
The element represents the
mean ticket sales collected
l(c) 70.9% 6(c)
from the three theme parks in
July.
( 72 76.5 62.1)
l(d) $9.13 6(d)
62.9 67.15 50.15
The discounted price for adult
ticket and children ticket at
Alpha Land is $72 and $62.90
l (e) $140.65 respectively, and at Beta Land,
$76.50 and $67.15 respectively
and at Gamma Land, $62.10
and $50.15 respectively.
3-n l
2(a) 7(a) x=5-
4m+n 3
2(b) x=-
l
7(b) -1
2 3
3(b) x =13 or x = 2 .4 8(b)(i) 78
3(c) 62.4 cmJ 8(b)(ii) 78.35
4(a) a=25 b = 32 8(b)(iii) 14.3
4(b) 289 8(c)(i) 79
4(c) 72 8(c )(ii) 11
x 2 - x1 represents interquartile
4(d)(i) y=(n+l)2 8(c)(iii)
range of the distribution.
4(d)(ii) z = 2n 2 8(d) p~80
4(e) Triangle 15
5(a) 2. 7 5 radians
5(b) 172 cm2

267 ANDSS 4E5N Prelim 2011 Math (4016/02) [Answer Scheme


1
lO(a)(i) -(a+ b) 12(a)(i) 1936n cm3
2
1
1O(a)(ii) -(a- b) 12(a)(ii) 2018.73 cm2 ·
3
lO(b) AB is parallel to OC . 12(c) 16.3 cm

lO(c)(i)
oc 1
-=- 13(b) x=3.3
AB 3
area of l:!.OCD 1
1O(c)(ii) =- J3(c) 4
area of MED 9
1
lO(d) -(4a-b) 13(d) a= 8, b=O
12
area of l:!.OCD l
lO(e)(i) =- 13(e) -3~x<-2.4 0<x<3.3
area of 60CA 4 '
3
lO(e)(ii) - l 3(t) k= 14
2
area of ll.BCD 3
10(e)(iii) =-
area of l:!.OABC 16
ll(a)(i) 120°
l l(a)(ii) l .08x io-3 km2
1 l(a)(iii) 10.4 m
1 l(b) 86.6m
ll(c)(i) 25m
1 l(c)(ii) 10.5°

:
j ......... ........ 2.9
)'

........................ !. . .....'.""""""'.· ·. ·. ..:. . . . . . .:... . .. . !. . '""'"]'"


: : :

...............................................i ............T..........................,...........................,.............

------·-·-.:"""''
:. . . ....:. ...........~............ l.............;.............

-J -2. s -i r
Oo00••••••• • :• •-•0 •••••• • ~ •••••• ••• • - : ·••OOo.••°'• • :- --• 00 •• 00 •!- 0 000000- o oo~oo ooU oo oUoo 0 • 00 O• U 0_.,0 - : • O>T0 . . •0• ~.·
. 0• 0 0 0••0 O OOroOO O OOOOO oOO~ooOooooooooO•o

~ . ; : ~ ~
··• ···• !· • ·,:··- ··•·• i: .. . ... i. • • -;.I ~ . .. : .; .
~ : ~
. ...... .T...
.
,:.............ti . .
1" . r... . - . .
-~,. . . ····· H •• • • • • •••••

-.... .....:: . ... ..................... i,.. .. . : .. .... :.i


; ;- ..... ,:.
•'l tJ .. . .. ... .... ... . .......... - ....•
~
'
-~ -

~ l i ' . ;
............ ·1· ............i............. ,.............;--.......... ·1·............:. .. ....... ... ........... "! .............,........... . ~ .............,.... ...... .. ..........·\·...........:·

ANDSS 4E5N Prelim 2011 Math (4016/02) [Answer Scheme 2 6a

You might also like

pFad - Phonifier reborn

Pfad - The Proxy pFad of © 2024 Garber Painting. All rights reserved.

Note: This service is not intended for secure transactions such as banking, social media, email, or purchasing. Use at your own risk. We assume no liability whatsoever for broken pages.


Alternative Proxies:

Alternative Proxy

pFad Proxy

pFad v3 Proxy

pFad v4 Proxy